Sei sulla pagina 1di 277

Mecnica y Ondas.

Planteamiento y resolucin
de problemas tipo

Alvaro Perea Covarrubias


Doctor en Ciencias Fsicas
Universidad Nacional de Educacin a Distancia
Madrid, Enero 2005

ndice
Captulo 1. Leyes de Newton
Primera Parte: Leyes de conservacin
Introduccin
1. Conservacin de la energa
2. Conservacin del momento lineal
3. Conservacin del momento angular
Problemas resueltos
Problemas propuestos
Segunda Parte: Fuerzas disipativas
Introduccin
1. Amortiguamiento de Coulomb
2. Amortiguamiento de Stokes
3. Amortiguamiento de Newton
Problemas resueltos
Problemas propuestos
Tercera Parte: Sistemas de masa variable
Introduccin
Teora
Problemas resueltos
Problemas propuestos

Captulo 2. Sistemas conservativos


Primera Parte: Dinmica de la partcula en una recta
Introduccin
1. Movimiento bajo la accin del potencial V(x)
2. Diagramas de energa
3. Puntos de equilibrio. Movimientos de oscilacin
Problemas resueltos
Problemas propuestos
Segunda Parte: Potenciales centrales
Introduccin
1. Conservacin del momento angular
2. Ley de las reas
3. Ecuaciones del movimiento
4. Conservacin de la energa
5. Potencial centrfugo. Potencial efectivo
6. Tipos de movimiento
7. Clculo de rbitas. Frmulas de Binet
Problemas resueltos
Problemas propuestos

Tercera Parte: Fuerza gravitatoria


Introduccin
1. Campo gravitatorio
2. Fuerza gravitatoria
3. Energa potencial gravitatoria
4. Energa total
5. Cuerpos continuos
6. Autoenerga gravitatoria
7. Ley de Gauss
Problemas resueltos
Problemas propuestos
Cuarta Parte: Movimiento planetario. Satlites
Introduccin
1. Ecuaciones del movimiento
2. rbitas planetarias. Descripcin analtica
3. rbitas planetarias. Descripcin grfica
4. Leyes de Kepler
Problemas resueltos
Problemas propuestos

Captulo 3. Dinmica del slido rgido


Primera Parte: Movimiento plano
Introduccin
1. Grados de libertad del movimiento
2. Campo de velocidades
3. Centro instantneo de rotacin
4. Momento lineal
5. Momento angular y momento de inercia
6. Teorema de Steiner
7. Energa cintica
8. Ecuaciones del movimiento
9. Condicin de deslizamiento
10. Colisin con una masa puntual
Problemas resueltos
Problemas propuestos
Segunda Parte: Movimiento general
Introduccin
1. Grados de libertad del movimiento
2. Eje instantneo de rotacin
3. ngulos de Euler
4. Velocidad angular en funcin de los ngulos de Euler
5. Matriz de inercia. Ejes principales. Momento angular
6. Energa cintica
7. Ecuaciones del movimiento
8. Utilidad de la ecuacin de Euler
Problemas resueltos
Problemas propuestos

Captulo 4. Relatividad especial


Primera Parte: Relatividad de Galileo
Introduccin
1. Principio de relatividad
2. Transformacin de Galileo
3. Efecto Doppler no relativista. Corrimiento de
4. Velocidad relativa de las galaxias. Edad del Universo
Problemas resueltos
Problemas propuestos
Segunda Parte: Cinemtica relativista
Introduccin
1. Principio de relatividad especial
2. Proporcionalidad entre intervalos de tiempo
3. Transformacin de Lorentz
4. Contraccin de la longitud
5. Dilatacin del tiempo
6. Adicin relativista de velocidades
7. Sucesos propios e impropios
Problemas resueltos
Problemas propuestos
Tercera Parte: Dinmica relativista
Introduccin
1. Masa relativista
2. Leyes de conservacin
3. Fenmenos de colisin
Problemas resueltos
Problemas propuestos

Captulo 5. Oscilaciones de sistemas con un grado de libertad


Introduccin
Clasificacin de las oscilaciones
Primera Parte: Oscilaciones libres
1. Movimiento armnico simple (MAS)
2. Energa en un MAS
3. Determinacin de para un MAS
4. Potencia desarrollada en un MAS
5. Sistemas sencillos de oscilaciones simples
I.
Muelle simple
II.
Pndulo simple
III.
Pndulo fsico
IV.
Objeto flotante
V.
Lquido en tubo en U
VI.
Torsin de hilos
VII. Muelle de aire
VIII. Asociacin de muelles
IX.
Asociacin de hilos
Problemas resueltos
Problemas propuestos
Apndice: Oscilaciones armnicas en dos dimensiones

Segunda Parte: Oscilaciones amortiguadas


Introduccin
Fenmenos de amortiguamiento
1. Oscilaciones amortiguadas
2. Movimiento subamortiguado
3. Movimiento amortiguado crtico
4. Movimiento sobreamortiguado
5. Parmetros del movimiento subamortiguado
6. Movimiento con amortiguamiento seco
Problemas resueltos
Problemas propuestos
Tercera Parte: Oscilaciones forzadas
Introduccin
1. Oscilaciones forzadas no amortiguadas
2. Resonancia
3. Rgimen transitorio y permanente
4. Desfase respecto a la fuerza exterior
5. Potencia absorbida
6. Movimiento generado por fuerzas de inercia
7. Movimiento forzado amortiguado
Problemas resueltos
Problemas propuestos

Captulo 6. Oscilaciones de sistemas con varios grados de libertad


Introduccin
Primera Parte: Sistema de dos masas y un muelle
1. Ecuaciones del movimiento
2. Movimiento del centro de masa
3. Movimiento relativo
4. Conclusiones
Problemas resueltos
Problemas propuestos
Segunda Parte: Modos de vibracin
Introduccin
1. Metodologa
2. Movimiento del sistema
3. Coordenadas normales
4. Movimiento forzado
Problemas resueltos
Problemas propuestos

Captulo 7. Vibraciones en sistemas continuos. Anlisis de Fourier


Introduccin
Primera Parte: Oscilaciones transversales
Introduccin
1. Oscilaciones acopladas en una cuerda con masas
2. Lmite continuo. Ecuacin de ondas
3. Soluciones armnicas simples
4. Ondas estacionarias: cuerda con dos extremos fijos
5. Energa mecnica en una cuerda vibrante
6. Ejemplo adicional: cuerda con un extremo libre
7. Pulsos de forma constante
Problemas resueltos
Problemas propuestos
Segunda Parte: Ondas de sonido
Introduccin
1. Ondas sonoras
2. Modos normales. Ondas estacionarias
Problemas resueltos
Problemas propuestos

Captulo 8. Ondas progresivas


Introduccin
1. Solucin general de la ecuacin de ondas
2. Soluciones armnicas simples
3. Velocidad de fase. Dispersin
4. Propagacin de un grupo de ondas. Velocidad de grupo
Problemas resueltos
Problemas propuestos
Caso prctico: Ondas de gravedad

Captulo 9. Formulacin lagrangiana


Introduccin
1. Lagrangiano
2. Integral de accin S
3. Principio de mnima accin
4. Ecuaciones de Lagrange
5. Ecuaciones del movimiento
6. Evaluacin de la energa cintica
7. Evaluacin de la energa potencial
8. Fuerzas de ligadura
9. Metodologa
Problemas resueltos
Problemas propuestos
Apndice: Formulacin hamiltoniana
A) Principio de mnima accin
B) Teorema de E. Noether
C) Formulacin hamiltoniana

Tema 1. Leyes de Newton


Primera parte: Leyes de conservacin
1. Conservacin de la energa
Supongamos que tenemos una masa m sometida al efecto de una fuerza F , que
sigue una trayectoria del punto A hasta el punto B. Definimos s como la coordenada
espacial a lo largo de la trayectoria. La ley de Newton define el movimiento

d 2s
F=m 2
dt
Si calculamos el trabajo WF de la fuerza a lo largo del recorrido, definido por

WF =

Fds

comprobamos lo siguiente

dv
1
1
WF =
m ds =
mvdv = mv2B mv 2A
dt
2
2
A
A
Por tanto, el trabajo de F sobre m es igual a la energa cintica ganada por la
partcula.
Cuando F es una fuerza conservativa, es decir, deriva de un potencial V (s ) ,
que por ahora supondremos que depende slo de las coordenadas del sistema, a travs
de la frmula

F=

dV
ds

el trabajo de la fuerza es

dV
= V A VB
ds
A
Comparando las dos expresiones obtenidas para WF se comprueba que
1 2
1
mvB + VB = mv2A + VA
2
2
WF =

Esto es, la energa definida como la suma

1
E = mv 2 + V
2
es constante.

Cuando existan fuerzas de rozamiento, contrarias al movimiento, obtenemos la


siguiente ecuacin para la fuerza neta que acta sobre el sistema

F =

dV
Fr
ds

y de aqu, evaluando WFr , vemos que

WFr = Fr ds = E A E B
A

La prdida de energa es igual al trabajo realizado por la fuerza de rozamiento. Si no


existe rozamiento, pero s existe prdida de energa, necesariamente debe existir una
fuerza, exterior al sistema, tal que WF = E A E B .
Cuando el sistema realice un giro con velocidad angular constante respecto a
un eje situado a una distancia d , la ley del movimiento debe incluir la fuerza centrfuga

Fc = m 2 d
en la forma

F =

dV
Fc
ds

El signo - delante de Fc indica que es una fuerza de inercia, como se ver en un tema
posterior. Podemos definir un potencial centrfugo Vc tal que

Fc =

dVc
ds

llegando a

Vc =

1
m 2 d 2
2

En estos casos de giro con velocidad angular constante , se conserva la energa como
la suma

E=

1
1
mv 2 + V + m 2 d 2
2
2

Hay que destacar que si Fc es perpendicular a la trayectoria no existe la contribucin de

Vc a la energa.
Por ltimo, es importante recordar que la definicin de trabajo es

WF =

r r
F dr

A
r
por lo que si F es perpendicular a la trayectoria no realiza trabajo, y no debe
contabilizarse en la ecuacin de la energa. Por ejemplo, la tensin T del hilo en un

pndulo no realiza trabajo, y la energa no depende de T .

2. Conservacin del momento lineal


La segunda ley de Newton puede expresarse en la forma

r
r
dp
Fext =
dt

Si no hay fuerza exterior Fext , el momento lineal del sistema p es constante. Se


aplicar este principio cuando se estudien los choques de partculas, en los que actan
slo fuerzas interiores al sistema, y en movimientos de traslacin sobre plataformas
libres.

3. Conservacin del momento angular


r

Cuando sobre un sistema acta una fuerza exterior Fext , aplicada en el punto con
r
vector de posicin r , la rotacin del sistema respecto del punto origen O se describe
por la ley

r
r r
dL
M O = r Fext = O
dt
r
r
siendo M O el momento de la fuerza respecto a O y LO el momento angular respecto a
r
O . En el caso de que la fuerza exterior sea nula, Fext = 0 que sea paralela al vector
r
r
de posicin r de su punto de aplicacin, el momento de fuerza M O es nulo, as y el
r
momento angular LO se conserva constante. Esta ley de conservacin del momento
angular determina la ley de movimiento en la rotacin libre de plataformas, en
movimientos de partculas dentro de superficies de revolucin y en la dinmica de los
slidos rgidos.

Problemas Resueltos
1.1

Una partcula de masa m se mueve en el interior de una superficie


semiesfrica de radio r bajo la accin de la gravedad. Si la partcula parte del
reposo en la posicin = 0 , determinar, en funcin del ngulo , la fuerza
ejercida por la partcula sobre la superficie en los distintos puntos de su
trayectoria.

r
m

Las fuerzas que actan sobre la masa m son el peso, la normal N y la fuerza
centrfuga debido a su giro respecto al punto O . Tomando coordenadas polares con
centro en O , la ecuacin de equilibrio de fuerzas en la direccin radial es

N = mg sin + Fc

N
Fc

90-

mg

La velocidad del movimiento (giro respecto a O) es v = r& , con lo cual

Fc = mr& 2 , y la normal satisface

N = mg sin + mr& 2

Las variables y & no son independientes. En este sistema se conserva la


energa, como suma de la energa cintica, que depende de la variacin temporal del
ngulo de giro, y la energa potencial, que depende del propio ngulo de giro. La
energa del sistema es

1
m v 2 + mgh
2
donde h es la altura de la masa m respecto del plano horizontal definido por la
ecuacin = 0 ,
h = r sin
E=

Con esto, la energa toma la expresin

1 2 &2
mr mgr sin
2
Conocidos los datos iniciales, & = 0 cuando = 0 , obtenemos el valor de la energa
E =0
E=

y, de su conservacin, deducimos la relacin

mr& 2 = 2mg sin


Con ayuda de este resultado, la fuerza ejercida por la partcula sobre la superficie
en funcin de su posicin angular es

N = 3mg sin

1.2

Sobre un estanque flota una tabla rectangular y homognea de masa m1 y

longitud L . Sobre uno de sus extremos descansa un gato de masa m2 . Con mucho
cuidado para no caerse, el gato se desplaza al otro extremo de la tabla. Cunto ha
avanzado el gato en relacin al agua? Qu retroceso ha sufrido el extremo de la
tabla?
Como no hay fuerza exterior al sistema, el momento lineal del sistema permanece
constante. Inicialmente tanto el gato como la tabla estn en reposo, y el momento lineal
del sistema es cero. De aqu se deduce que el centro de masas del sistema permanece
fijo. Tomamos un sistema de referencia ligado al agua (en reposo), cuyo origen coincida
con el centro de la tabla en el momento inicial. Respecto a este sistema, la posicin del
centro de masas xcm es

(m1 + m2 )xcm = m1 x1 + m2 x2 = m1 L
2

Sea dt la distancia positiva respecto al agua recorrida por la tabla y d g la


distancia recorrida por el gato respecto al agua en sentido contrario. Como L es la
distancia recorrida por el gato sobre la tabla, se cumple

dt + d g = L
Adems, el centro de masas debe mantener su posicin respecto al agua por lo que

m1x1 + m2 x2 = 0

En funcin de las distancias positivas recorridas por el gato y la tabla esta ecuacin se
escribe

m1dt = m2 d g

Resolviendo el sistema de ecuaciones, encontramos

m1
L
m1 + m2
m2
dt =
L
m1 + m2
dg =

1.3

Un hombre de masa m se encuentra en un punto P de la periferia de una


plataforma circular de radio R y momento de inercia IO respecto a su centro. La
plataforma puede girar libremente respecto a O. En un instante dado, el hombre

comienza a caminar con velocidad constante V0 , en una direccin que forma un


ngulo constante con el radio vector inicial OP. Calcular la velocidad angular
del sistema cuando el hombre alcance el borde de la plataforma.

V0

El sistema formado por el hombre y la plataforma no sufre ninguna fuerza


exterior. Su movimiento conjunto se debe slo a las fuerzas de accin y reaccin
creadas cuando el hombre camina. Por tanto, la ley de movimiento es en este caso la ley
de conservacin del momento angular respecto a O. Inicialmente es nulo puesto que
tanto la plataforma como el hombre estn reposo, por lo que LO = 0 . Para mantener
constante este momento angular la plataforma siempre deber girar en sentido contrario
al movimiento del hombre.
Si d es la distancia recorrida por el hombre, al ser su movimiento uniforme

d = V0t

y su momento angular respecto al origen es

LO ,h = mrV0 sin
donde r es la distancia del hombre al origen O y es el ngulo formado por el vector
de posicin OQ del hombre y su velocidad. En el tringulo OPQ, se cumple

Q
r
O

r
R
=
sin sin

R
Con esto,

LO ,h = mRV0 sin = cte

Se comprueba as que el momento angular del hombre tambin es constante.


El momento de la plataforma es, entonces,

LO , p = LO,h = mRV0 sin

Al ser un cuerpo continuo, su momento angular se define por

LO, p = I O

siendo I O su momento de inercia respecto a O, y la velocidad de giro. Por tanto,

mRV0 sin
= cte
IO

en sentido contrario al movimiento del hombre.

1.4

Una cadena flexible, de longitud L y masa M se encuentra sobre una


superficie lisa de forma que cuelga libremente una longitud L0 . Si el sistema en
reposo se abandona a s mismo, determinar la velocidad de la cadena en el instante
en el que el ltimo eslabn deja la superficie.

L L0
L0

Utilizamos la conservacin de la energa para calcular la velocidad final de la


cadena. Tomando la energa potencial igual a cero en la superficie lisa, la energa
potencial de la cadena, cuando cuelga una longitud l es

E p = mgycm
siendo m la masa de los eslabones de longitud l , situados en promedio a una distancia
ycm del borde de la superficie lisa. Si la cadena es homognea, con su masa igualmente
l
l
distribuida por todos los eslabones, tenemos m = M . Adems, y cm = . Con esto,
L
2
la energa potencial de la cadena en cualquier instante es

1
l2
E p = Mg
2
L
Por conservacin de la energa

1
1
l2
2
E = Ec + E p = MV Mg
2
2
L
Inicialmente, l = L0 y V = 0 , por lo que el valor de la energa es
1
L20
E = Mg
2
L
Introduciendo este dato en la ecuacin de la energa, hallamos la relacin existente entre
la velocidad de cada de la cadena y la longitud de los eslabones que cuelgan en
cualquier instante

V=

g 2
(
l L20 )
L

Cuando la cadena se separa de la superficie lisa, l = L , su velocidad es

V=

g 2
(L L20 )
L

1.5

En la colisin entre dos cuerpos, el coeficiente de restitucin e se define


como el cociente de los mdulos de las velocidades relativas para los estados final e
inicial. Particularizando para una colisin en una dimensin, probar que en una
colisin elstica e = 1 , y para una colisin inelstica, calcular la energa cintica
perdida en funcin de e .
En cualquier colisin, las fuerzas que actan son interiores al sistema, por lo que
siempre est asegurada la conservacin del momento lineal total, suma de los momentos
lineales de las partculas implicadas en la colisin. Por definicin, una colisin elstica
es aquella en la que se produce la conservacin de la energa total, mientras que en una
colisin inelstica una parte de la energa cintica inicial se transforma en calor por
friccin, constriccin, etc. Sin tener en cuenta los mecanismos que producen esta
transformacin, en toda colisin inelstica se observa una prdida de energa cintica
del estado final respecto al inicial.
En la colisin elstica, la conservacin de momento lineal y energa se escribe

m1V1 + m2V2 = m1V1 + m2V2

1
1
1
m1V12 + m2V22 = m1V12 + m2V22
2
2
2
donde los valores correspondientes al estado final llevan una prima. Estas ecuaciones
pueden reducirse a las expresiones

m1 (V1 V1) = m2 (V2 V2 )

m1 (V12 V12 ) = m2 (V22 V22 )


Dividiendo la segunda por la primera, obtenemos

V1 + V1 = V2 + V2

(a)

La velocidad relativa final e inicial se definen por

Vrel = V2 V1
Vrel = V2 V1
por lo que la ecuacin (a) anterior establece que en la colisin elstica

= Vrel
Vrel

con el valor del coeficiente de restitucin igual a 1

e=

Vrel
=1
Vrel

Desde el punto de vista del sistema centro de masa, que se mueve con velocidad
constante Vcm , la colisin elstica corresponde a un proceso en el que la velocidad
relativa cambia de signo.

Para tratar la colisin inelstica, vamos a tomar como referencia la velocidad del
centro de masa Vcm , que se conserva constante antes y despus del choque, y la
velocidad relativa Vrel antes del choque. Por definicin

(m1 + m2 )Vcm = m1V1 + m2V2

Vrel = V2 V1
con lo cual, en funcin de las variables Vcm , Vrel , las velocidades iniciales son
m2
V1 = Vcm
V
m1 + m2 rel

V2 = Vcm +

m1
V
m1 + m2 rel

Las velocidades finales satisfacen la ley de conservacin del momento

(m1 + m2 )Vcm = m1V1 + m2V2

y, por definicin del coeficiente de restitucin,

eVrel = V2 V1

Con ayuda de estas dos ecuaciones quedan determinadas las velocidades finales en la
forma

m2
V
m1 + m2 rel
m1
V2 = Vcm + e
V
m1 + m2 rel
V1 = Vcm e

La energa cintica perdida durante la colisin es

1
1
1
1
m1V12 + m2V22 m1V12 m2V22
2
2
2
2
1
mm
= (1 e 2 ) 1 2 Vrel2
2
m1 + m2

Q = Ei E f =

De una forma ms directa se puede resolver la colisin inelstica. Sabemos que


en ausencia de fuerzas exteriores el movimiento de un sistema de masas puede
estudiarse como la combinacin del movimiento del centro de masa y del movimiento
relativo. El centro de masa tiene asociada la masa total del sistema (m1 + m2 ) , y el
movimiento relativo tiene asociada la masa reducida =

m1m2
.
m1 + m2

Por tanto, la energa cintica ser la suma de la energa cintica del centro de masa, y de
la energa cintica del movimiento relativo, en la forma

Ei =

1
(m1 + m2 )Vcm2 + 1 m1m2 Vrel2
2
2 m1 + m2

y la prdida de energa durante la colisin ser

Q=

1 m1m2
1
mm
(
Vrel2 Vrel2 ) = (1 e 2 ) 1 2 Vrel2
2 m1 + m2
2
m1 + m2

1.6

Dos masas A y B chocan frontalmente con velocidades en sentidos opuestos,


segn muestra la figura. Si la colisin es inelstica y el coeficiente de restitucin es
e , calcular las velocidades finales y la energa perdida.

3m

2m

2V0

3V0

Se conserva el momento lineal del sistema. Inicialmente

pi = 2m(2V0 ) 3m(3V0 ) = 5mV0

Si VA , VB son las velocidades finales, el momento lineal final es

p f = 2mVA + 3mVB
Aplicando la conservacin del momento lineal pi = p f , obtenemos la relacin entre
las velocidades finales

2VA + 3VB = 5V0


Introducimos ahora el coeficiente de restitucin, cociente de las velocidades
relativas final e inicial

e=

VA VB
V VB
= A
2V0 ( 3V0 )
5V0

VA VB = 5eV0
La solucin al sistema de ecuaciones para las velocidades finales es

VA = V0 (3e 1)

VB = V0 (1 + 2e )
Segn un problema anterior, la prdida de energa en un choque inelstico es

Q=

1
(1 e 2 )Vrel2
2

En este sistema, la masa reducida es

2m 3m 6
= m
2m + 3m 5

y la velocidad relativa antes de la colisin

Vrel = 2V0 ( 3V0 ) = 5V0


con lo cual, la energa perdida resulta ser

Q=

16
3
m(1 e 2 )25V02 = m(1 e 2 )V02
25
5

1.7

Dos discos de radio r se mueven sobre un tapete. El disco B est en reposo


y el disco A se acerca con velocidad V . Calcular el parmetro de impacto b para
que la componente de la velocidad de B perpendicular a V , despus del choque,
sea mxima. Hallar tambin la direccin del movimiento de B. La colisin es
elstica.

V
r

En el momento de la colisin, V est dirigida hacia la derecha, y la distancia


vertical entre los centros de las esferas es b . Sean VA , VB las velocidades finales de los
discos, formando un ngulo y con el eje horizontal, respectivamente. La
velocidad de B despus de la colisin est dirigida en la perpendicular al punto de
contacto de los dos discos. De la figura

VB

B
r

VA
vemos que

sin =

b
2r

Ya que las masas son iguales, la conservacin del momento lineal (en forma
vectorial) y la conservacin de la energa se escriben

V = VA + VB
V 2 = V A2 + VB2
Elevando al cuadrado la primera ecuacin, y comparndola con la segunda vemos que

VA VB = 0
Es decir, el ngulo formado por las velocidades finales de A y B debe ser 90. Segn la
figura anterior, esto equivale a

+ = 90

Con este dato, para hallar los mdulos de las velocidades, resolvemos la ley de
conservacin del momento lineal sobre cada uno de los ejes. Obtenemos

V = VA cos + VB sin
0 = VB cos VA sin
con la solucin

VA = V cos
VB = V sin
La componente de la velocidad de B perpendicular a la velocidad inicial V es

(VB ) = VB sin = V sin cos = 1 V sin 2


2

y se hace mxima cuando sin 2 = 1 , = 45 . El parmetro de impacto para ese


ngulo es

b = 2r sin 45 = 2r
1.8

Se deja caer una bola de masa m desde una altura H 0 sobre un plano

horizontal. Despus de la colisin la bola sube hasta una altura H1 . Calcular el


coeficiente de restitucin e y el tiempo que tarda la bola en quedarse inmvil en el
suelo.
Si V0 es la velocidad antes del choque con el suelo y V1 la velocidad despus del
choque, el coeficiente de restitucin viene dado por

e=

V1
V0

La velocidad con que llega al suelo, al caer desde una altura H 0 , es

V0 = 2gH0
y la velocidad con que despega del suelo, para poder subir hasta la altura H1 , es

V1 = 2gH1
Por tanto, el coeficiente de restitucin es

e=
De esta frmula se concluye adems que

H1
H0

H1 = e 2 H 0
2
Es decir, la altura alcanzada despus del choque es igual a e multiplicado por la altura
antes del choque.

En el n-simo bote, la bola alcanzar una altura

H n = e2 H n1 = e4 H n 2 = ... = e 2n H 0

y por tanto, se parar slo despus de infinitos botes, ya que al ser e < 1, H n 0 si
n.
Calculamos el tiempo Tn que tarda la bola en recorrer la distancia de bajada y de
subida en el n-simo bote. El tiempo de bajada es

tb = 2

H n1
g

y el tiempo de subida es

ts = 2

Hn
g

con lo cual

Tn = t b + t s = 2

H n1
H
+ 2 n
g
g

Introduciendo el coeficiente de restitucin

e 2 n2 H 0
e2 n H 0
2H 0 n1
Tn = 2
+ 2
=
e (1 + e )
g
g
g
Como la bola sufre infinitos botes antes de pararse, el tiempo que tarda en
quedarse inmvil en el suelo es

T=

n =1

1.9

2H 0
Tn =
(1 + e )
g

n =0

en =

2H 0 1 + e
g 1 e

Una partcula de masa m , sujeta a una cuerda inextensible de masa


despreciable, describe una trayectoria circular sobre una superficie horizontal sin
rozamiento. Se tira hacia abajo de la cuerda con una fuerza F , a partir de un
instante en el que la partcula est a una distancia r0 del orificio y lleva una
velocidad V0 . Si como resultado la cuerda se desplaza hacia abajo con una
velocidad constante U , determinar la tensin de la cuerda en todo instante.


r
m

F
Como la cuerda se desplaza radialmente con velocidad constante, la masa no
tiene aceleracin radial. La fuerza neta radial es cero, y la partcula describe una
trayectoria circular cuyo radio disminuye con el tiempo debido al desplazamiento de la
cuerda. La ecuacin de equilibrio de fuerzas en la direccin radial es

F = Fc
donde Fc es la fuerza centrfuga debido al giro de m respecto al centro de la
trayectoria circular. Por tanto, la velocidad de la partcula y el radio de su trayectoria
estn relacionados segn la frmula

F=m

V2
r

Al ser radial la fuerza exterior aplicada F , su momento respecto a O es cero, y


as, el momento angular respecto a O se conserva constante. En el instante inicial,

LO ,i = mr0V0
y en un instante arbitrario

LO , f = mrV

Entonces, de la conservacin del momento angular obtenemos la relacin

V =

r0
V
r 0

y de aqu, la fuerza aplicada en funcin de la coordenada radial

F=m

r03V02
r3

Nos queda encontrar la variacin temporal de r . Si la cuerda se desplaza hacia


abajo con velocidad constante U , el radio de la trayectoria de m disminuye de acuerdo
con

U =

dr
dt

Teniendo en cuenta la condicin inicial r = r0 en t = 0 , esta ecuacin puede integrarse


con el resultado

r = r0 Ut

De aqu, finalmente, la variacin temporal de la fuerza aplicada es

r03V02
F=m
(r0 Ut )3
Ya que la cuerda es inextensible, la fuerza neta sobre cada porcin de ella debe ser nula.
Con esto concluimos que la tensin de la cuerda debe ser igual a la fuerza aplicada F .

Problemas Propuestos
1.10 Una rana de masa m est situada en el extremo de una tabla recta de masa
M y longitud L . La tabla se encuentra en reposo flotando en un estanque. La
rana da un salto a lo largo de la tabla con un ngulo de elevacin sobre la
horizontal. Calcular la velocidad inicial mxima V0 de la rana para que en el salto
no caiga al agua.

1
M
gL
sin 2 M + m

Solucin: V0, max =

1.11 Sobre una esfera fija se coloca una masa puntual en la posicin ms alta. Si
se desplaza ligeramente de su posicin de equilibrio, su movimiento se realiza
sobre la superficie esfrica. Determinar la posicin donde la masa abandona la
superficie esfrica. Dicha posicin queda definida por el ngulo que forma la
vertical con la direccin radial desde el centro de la esfera a la masa puntual.
Solucin: cos =

2
3

1.12 Se colocan n masas M a lo largo de una recta, de modo que cada una toca
a la siguiente. Desde la izquierda inciden dos masas M con la misma velocidad V
y chocan sucesivamente con la fila de n masas. Probar que es imposible que como
consecuencia de la colisin elstica una sola masa M sea expulsada por la derecha
o que lo sean dos masas con velocidades diferentes.

1.13 Una masa puntual m est ligada a una cuerda inextensible que a medida
que va girando se enrolla sobre un eje fijo de radio a . Si la velocidad angular de
giro es 0 cuando la distancia de la partcula al punto de tangencia es r0 ,
determinar la velocidad angular de giro y la tensin de la cuerda, cuando el
sistema ha girado un ngulo .

r0
r0
= 0

r0 a

Solucin:

r04
T = m
(r0 a )3
2
0

1.14 Una masa puntual m se ve obligada a moverse sobre un aro de radio R . El


aro se sita verticalmente y se le comunica una velocidad angular constante de
giro respecto al eje vertical OO . Utilizando el concepto de trabajo de las fuerzas
implicadas en el movimiento, determinar la expresin de la conservacin de la
energa en funcin de la variable angular definida en la figura.

Tema 1. Leyes de Newton


Segunda parte: Fuerzas disipativas
En el mundo real, la energa no se conserva. Una piedra lanzada hacia arriba no
llega a la altura deseada, la amplitud del pndulo se amortigua, una bola que rueda
sobre el suelo horizontal acaba parndose. As ocurre porque existen fuerzas de
rozamiento o friccin que se oponen al movimiento. Podramos decir, no obstante, que
la energa total se conserva si incluimos la energa trmica. que se acumula en los
grados de libertad internos de los cuerpos en cuestin.

La consecuencia directa de las fuerzas de rozamiento es la disipacin de energa.


En general, la fuerza de rozamiento es paralela y de sentido contrario a la velocidad y
puede expresarse como la suma

Fr (V ) = N V V 2

donde N es la normal o reaccin de la superficie en la que se apoya el cuerpo.

1. Amortiguamiento de Coulomb
Es el rozamiento seco. Cuando un objeto est apoyado sobre una superficie, se
produce una fuerza que se opone al movimiento y que cancela cualquier fuerza aplicada
que sea menor que N . Si la fuerza aplicada es mayor que N , la fuerza de
rozamiento toma un valor constante e igual a N . Escribimos, entonces

Fr = N

si

V >0

Fr = F

si

V =0

1.16 Calcular el tiempo que tarda en pararse una partcula por efecto del
rozamiento de Coulomb si su velocidad inicial es V0 , y la distancia recorrida.
La ley del movimiento es

ma = N = mg

y la deceleracin que sufre la partcula por el rozamiento seco es

a = g

Integrando con las condiciones iniciales x = x0 , V = V0 en t = 0 , obtenemos

V = V0 gt
1
x = x0 + V0t gt 2
2

Con ayuda de estas frmulas, obtenemos el tiempo de parada

t=

V0
g

y la distancia recorrida

V02
x x0 =
2 g

2. Amortiguamiento de Stokes
Tambin llamado viscoso. Cuando un cuerpo se mueve en el seno de un fluido, el
fluido se opone a su movimiento intentando contrarrestar la deformacin que provoca el
movimiento del cuerpo. La razn de este comportamiento radica en la viscosidad del
medio fluido, efecto macroscpico originado por el rozamiento entre las capas
moleculares del fluido. El amortiguamiento de Stokes tiene la expresin

FS = V

1.17 Calcular la ley de movimiento para una partcula que sufre el


amortiguamiento de Stokes si su velocidad inicial es V0 .
La ley del movimiento es

dV
= V
dt

Definiendo la constante de tiempo

la ecuacin del movimiento

dV V
+ =0
dt
tiene la solucin

t
V = V0 exp

Integrando una vez ms, encontramos la posicin de la partcula en funcin del tiempo

t
x = V0 1 exp

El cuerpo se para cuando V = 0 , al cabo de un tiempo infinito, pero recorre una


distancia finita V0 . Esto no se corresponde con lo observado (el tiempo de parada
finito), ya que cuando la velocidad se hace pequea el rozamiento no sigue la forma de
Stokes, y se hace ms intenso.

3. Amortiguamiento de Newton
Tipo de rozamiento en un fluido viscoso, cuando el movimiento adquiere
velocidades mayores que el caso de Stokes, como puede ser el movimiento de un cohete
en la atmsfera. Tiene la expresin

FN (V ) = V 2

1.18 Calcular la ley de movimiento para una partcula que sufre el rozamiento
de Newton si su velocidad inicial es V0
La ley del movimiento es

dV
= V 2
dt

que escribimos en la forma

dV 2
+ V =0
dt m
La solucin que satisface las condiciones iniciales es

V = V0

1
V
1+ 0 t
m

De aqu, obtenemos la posicin de la partcula

x=

m
V
ln 1 + 0 t

m

El cuerpo se para cuando V = 0 , al cabo de un tiempo infinito, y una distancia


infinita. Esto no se corresponde con lo observado, ya que cuando la velocidad se hace
ms pequea el rozamiento sigue la frmula de Stokes.

Tema 1. Leyes de Newton


Tercera parte: Sistemas de masa variable
Los sistemas de masa variable, es decir, sistemas en los que la masa que se
encuentra en movimiento depende del tiempo, no conservan la energa. Para estudiar el
movimiento utilizamos la 2 ley de Newton, escrita en la forma

Fext =

dp
dt

y referida a un sistema de ejes fijo en el espacio.


Estudiamos el caso ms general posible. Supondremos que entre el instante t y el
instante t + dt , el sistema gana una masa dmg , y expulsa una masa dme con una
velocidad U respecto del sistema, en sentido contrario a la velocidad V del sistema.
Tomamos como referencia un sistema de ejes fijos.
El momento lineal del sistema, en el instante t , es
y en el instante t + dt

pi = mV

p f = m + dmg (V + dV ) + dme (V + dV U )

Restando, el incremento de momento lineal durante el intervalo de tiempo dt es

dp = mdV + Vdmg + (V U )dme

donde hemos despreciado los trminos de orden cuadrado en los diferenciales. De aqu,
la 2 ley de Newton del movimiento tiene la expresin

Fext = m

dmg
dV
dm
+V
+ (V U ) e
dt
dt
dt

Como casos particulares, si no hay expulsin de masa, como el ejemplo del trineo
que acumula nieve en su movimiento o de la gota en condensacin,

dme
=0
dt
dmg dm
=
dt
dt
con lo cual

Fext =

d (mV )
dt

En el caso del cohete, siendo m la masa del cohete,

dmg dm
=
<0
dt
dt
dme
dm
=
>0
dt
dt
obtenemos

Fext = m

dV
dm
+U
dt
dt

Problemas Resueltos
1.19 Calcular la velocidad y aceleracin de ascensin vertical de un cohete que
expulsa gases a ritmo constante, con una velocidad U respecto al cohete.
La masa del cohete depende del tiempo, segn la ecuacin

dm
= k
dt
m = m0 kt
siendo k la razn de expulsin de los gases quemados, medida en kg/seg.
La ecuacin de movimiento adecuada es

Fext = m

dV
dm
+U
dt
dt

Ya que el movimiento del cohete es de ascensin vertical, la fuerza exterior


corresponde a la fuerza de la gravedad, dirigida hacia abajo. Obtenemos

mg = m

dV
kU
dt

El estado dinmico del cohete queda caracterizado completamente por el valor de


su masa en cada instante, y la dependencia temporal de las variables de movimiento del
cohete, ( x, V ) , tiene la forma funcional

x(t ) = x[m(t )]

V (t ) = V [m(t )]
Como consecuencia, al resolver el problema es aconsejable, mediante la regla de la
cadena, sustituir en la ecuacin del movimiento las derivadas temporales por derivadas
respecto de m , eliminando as la variable temporal. Obtenemos

d dm d
d
=
= k
dt dt dm
dm
con lo cual la ecuacin de movimiento es

mg = km

dV
kU
dm

dV g U
=
dm k m
Ahora podemos integrar directamente, con el resultado

V=

g
g U
dm = m U ln m + cte
k
k m

La condicin inicial establece que el cohete parte del reposo, con masa inicial m0 . Es
decir, V = 0 cuando m = m0 . Aplicando esta condicin inicial, obtenemos el valor de
la constante

cte = U ln m0

g
m
k 0

y la velocidad de ascensin en funcin de la masa del cohete

V=

g
(m m0 ) + U ln m0
k
m

Introduciendo m(t ) , obtenemos dicha velocidad de ascensin vertical en funcin


del tiempo

V = gt + U ln

m0
m0 kt

Finalmente, derivando respecto del tiempo, obtenemos la aceleracin de subida en


funcin del tiempo,

dV
Uk
= g +
dt
m0 kt
Ya que el cohete parte del reposo en el instante inicial, para que ascienda
inicialmente es necesario que

dV
>0

dt 0
Uk
>g
m0
relacin que se debe satisfacer entre la masa inicial del cohete, la razn y velocidad de
expulsin de los gases para que sea posible la ascensin vertical del cohete.

1.20 Calcular la velocidad y aceleracin de un cohete, que se mueve


horizontalmente, y que expulsa gases a ritmo constante, con una velocidad U
respecto al cohete.
De la misma forma que en el problema anterior

dm
= k
dt
m = m0 kt
La ecuacin de movimiento adecuada es

Fext = m

dV
dm
+U
dt
dt

Ahora, al moverse horizontalmente, el cohete sufre el amortiguamiento de Newton


(velocidades grandes) por lo que Fext = V 2 y

V 2 = m

dV
kU
dt

Utilizando de nuevo la regla de la cadena, eliminamos la variable t , con lo cual


la ecuacin de movimiento se escribe en la forma

V 2 = km

dV
kU
dm

dV
dm
+
=0
2 m
U V
k
La integracin es directa, resultando

Uk
+V

k
ln
+ ln m = cte
4U
Uk
V

Con la condicin inicial, V = 0 cuando m = m0 , obtenemos la constante

cte = ln m0 , y la velocidad horizontal en funcin de la masa del cohete

V=
donde hemos definido el parmetro

Uk m0 m
m0 + m

4U
k

Con este resultado, la aceleracin puede obtenerse a partir de la ecuacin del


movimiento. Llegamos a la expresin

dV kU V 2 2k m0 m
=
=
dt
m
m (m + m )2
0
1.21 Calcular la aceleracin de cada de una cadena agrupada situada sobre un
plano horizontal, debido a su propio peso.

x
Tomamos x como la longitud de la cadena que est cayendo en el instante t . Por
tanto, la longitud de cadena en movimiento en el instante t es x . Existen fuerzas de
rozamiento entre los eslabones y el plano, que evitan que toda la cadena est en
movimiento en un momento dado. Es un sistema que no conserva la energa.
Al no conservar la energa, y no existir expulsin de masa, debemos utilizar la ley
de Newton en la forma

Fext =

d
(mV )
dt

En el instante t , la masa en movimiento es

m = x

donde es la densidad de masa de la cadena (masa por unidad de longitud), y cae con

dx
, bajo la accin de su peso
dt
Fext = mg = xg
Por tanto, en el instante t , la ley de movimiento del sistema es
d
xg = (xV )
dt
una velocidad V =

De la misma forma que en el problema del movimiento de un cohete, el estado


dinmico dependa exclusivamente del valor de la masa del cohete, en este problema, el

estado dinmico depende exclusivamente del valor de la masa en movimiento, funcin


de la variable x . Por tanto, es recomendable sustituir la derivada temporal por una
derivada respecto a x , utilizando la regla de la cadena

d dx d
d
=
=V
dt dt dx
dx
Con este cambio, la ecuacin de movimiento pasa a ser

xg = V

d
( xV )
dx

Para integrar esta ecuacin, multiplicamos por x los dos miembros,


resultado

x 2 g = xV

con el

d
1 d
2
(
xV ) =
(
xV )
dx
2 dx

Ahora, la integral es directa

x 2V 2 =

2 3
x g + cte
3

Inicialmente, la cadena est en reposo totalmente agrupada, por lo que la condicin


inicial es V = 0 cuando x = 0 . Aplicando esta condicin, obtenemos el valor de la
constante, cte = 0 , y el perfil de la velocidad de cada en funcin de la longitud de la
cadena en movimiento

V =

2
xg
3

Para obtener la aceleracin de cada, derivamos respecto al tiempo, con el


resultado

a=

dV dx dV
dV 1 dV 2 1
=
=V
=
= g
dt dt dx
dx 2 dx 3

1.22 Calcular la velocidad y aceleracin de cada de una cadena, que


inicialmente se encuentra en posicin vertical, con uno de sus extremos tocando
justamente el suelo. Calcular la fuerza que ejerce sobre el suelo en su cada.

x
N

Al existir una fuerza que frena el movimiento, la fuerza normal del suelo sobre la
cadena, no se conserva la energa del sistema. Sea x la longitud de la cadena en el aire,
y L x la longitud de la cadena depositada en el suelo. La ecuacin de movimiento
para la cadena de longitud x que est cayendo es

dV
=g
dt
puesto que slo acta la fuerza de la gravedad. De aqu obtenemos la velocidad de cada

V = 2 gs
siendo s la distancia vertical recorrida por la cadena desde su posicin inicial. En este
caso

s = Lx

por lo que la velocidad de cada tiene la expresin

V = 2 g (L x )

La fuerza normal N debe contrarrestar el peso de la cadena de longitud L x ,


depositada en el suelo, y la fuerza que ejerce la cadena de longitud x sobre el suelo,
debida a su movimiento. Calculamos dicha fuerza segn la expresin general

F=

dp
dt

siendo dp el momento lineal que comunica la cadena al suelo en el tiempo dt . Sea


la densidad de masa de la cadena. Dicha cadena lleva una velocidad

V =

dx
dt

(el signo menos tiene en cuenta que la coordenada x disminuye con el tiempo).
Durante el intervalo de tiempo dt , la longitud de cadena que se deposita en el suelo es
dx = Vdt , comunicando un momento lineal dp = dxV = V 2dt al suelo. Por
tanto, la fuerza que ejerce sobre el suelo la cadena que cae es

F = V 2 = 2g (L x )

Con este resultado, la fuerza normal que ejerce el suelo sobre la cadena ser la suma

N = g (L x ) F = 3g (L x )

igual a tres veces el peso de la parte de la cadena depositada. En virtud de la ley de


accin y reaccin, la fuerza que ejerce la cadena sobre el suelo tiene el mismo valor.

Problemas Propuestos
1.23 Los dos bloques de una mquina de Atwood simple tienen inicialmente la
misma masa m0 . El bloque de la izquierda es un recipiente que contiene agua, que
comienza a expulsar a razn constante k con velocidad U respecto del recipiente.
Si el contenido del agua es de 8 / 9 m0 , calcular la velocidad y aceleracin del
bloque de la derecha cuando el bloque izquierdo expulse todo el agua.

8
m
9 0

U
a=
Solucin:

m0

4
9 kU
g+
5
10 m0

V =

8 m0 g m0 g
9
+ 2
+ U ln
9 k
k
5

1.24 Calcular la velocidad y aceleracin de cada de una gota de agua que, en


presencia de un ambiente saturado de vapor, aumenta su masa a ritmo constante
k , debido a la condensacin de vapor sobre su superficie.

g
(m 2 m02 )
2km
Solucin:
g m2 + m02
a=
2 m2
V=

Tema 2. Sistemas conservativos


Primera parte: Dinmica de la partcula en una recta
Estudiamos el movimiento de una partcula puntual de masa m a lo largo de una
recta bajo la accin del potencial V (x ) . La fuerza que acta sobre la partcula es

F=

dV
dx

con lo que la 2 ley de Newton se expresa

mx&& =

dV
dx

La energa total del sistema se define como la suma de la energa cintica y la


energa potencial (o simplemente potencial), en la forma
2

1 dx
E = m + V (x )
2 dt
y se conserva constante

dE
=0
dt
Los sistemas conservativos de un grado de libertad son integrables. A partir de la
ecuacin de la energa, y en virtud de su conservacin, la solucin es expresable
mediante una integral. Para la solucin que satisface la condicin inicial, x = x0 en
t = 0 , la integral del movimiento es

()
x

x0

dx
2
(E V (x ))
m

=t

El signo debe estar de acuerdo con la direccin del movimiento. Ser positivo para una
velocidad inicial positiva y negativo para una velocidad inicial negativa. Como
conclusin, la solucin queda fijada conocidos los datos iniciales ( x0 , x&0 ) y el valor de
la energa del sistema

E=

1 2
mx& + V (x0 )
2 0

En principio, la nica dificultad nos la plantea la resolucin de esta integral. Cuando el


potencial sea una funcin complicada de x , dicha integral no ser fcil de evaluar. En
tales casos, y en todos en general, es conveniente anticipar el tipo de movimiento de
forma cualitativa. Para ello utilizamos los llamados diagramas de energa, que
explicamos a continuacin.

El caso que ms nos interesa corresponde a un potencial V (x ) con puntos de


equilibrio. Los puntos de equilibrio corresponden a puntos donde se anula la primera
derivada del potencial. En ese caso, la fuerza sobre la partcula es nula, lo que permite
que la partcula se mantenga indefinidamente en reposo. Cuando se desplaza del punto
de equilibrio, el movimiento subsiguiente depender del valor de la energa. Nuestro
objetivo es describir cualitativamente la relacin existente entre el valor de la energa y
los distintos tipos de movimiento. Para ello, dibujamos el perfil de un potencial
arbitrario con distintos puntos de equilibrio.

V(x)
E
1 2
mx&
2

Vb
Va
Vc
a

En primer lugar dibujamos una lnea horizontal que corresponda al nivel de


energa E . La distancia vertical entre el nivel de energa y la grfica de V (x )
corresponde a la cantidad

1 2
mx& , siempre mayor o igual que cero. Por tanto, el
2

movimiento se produce en la regin donde la energa es mayor que el potencial. Es la


llamada regin permitida. Los puntos de retroceso son aquellos que satisfacen
E = V ( x ) . Separan la regin permitida de la regin prohibida, y a ellos llega la
partcula con velocidad nula, invirtiendo el sentido de su movimiento.
Una vez delimitada la regin de movimiento, estudiamos el comportamiento en
su interior, principalmente en la vecindad de los mximos y mnimos del potencial. Al
acercarnos a un mnimo, la pendiente del potencial es negativa

dV
< 0 , con lo cual la
dx

fuerza ejercida sobre la partcula es positiva. El potencial acelera la partcula hacia el


mnimo. Cuando nos alejamos del mnimo, la pendiente es positiva y la fuerza ejercida
es negativa. El potencial frena a la partcula desde el mnimo. Por tanto, la partcula se

ve obligada a oscilar respecto de los puntos mnimos del potencial. Los puntos mnimos
son puntos de equilibrio estable. Opuestamente, en la vecindad de un punto mximo,
el potencial frena a la partcula que se acerca al mximo, y acelera a la partcula que se
aleja de l. Por tanto, la partcula se ve expulsada de los puntos mximos del potencial.
Son puntos de equilibrio inestable. Cuando existen varios puntos de equilibrio en su
trayectoria, la partcula aumenta su velocidad al acercarse a un mnimo, por donde pasa
con su velocidad mxima, y disminuye su velocidad al acercarse a un mximo, por
donde pasa con su velocidad mnima. Y en cualquier caso, la partcula invierte su
movimiento al llegar a un punto de retroceso.
De la integral del movimiento, obtenemos el perodo de la oscilacin entre dos
puntos de retroceso ( x1 , x2 )

T =2

x2

x1

dx
2
(E V (x))
m

Cuando uno de los puntos de retroceso se encuentre en el infinito, uno de los


datos a tener en cuenta es la velocidad con que llega la partcula al infinito. Por
definicin, dicha velocidad es

v =

2
(E V ( ))
m

Podemos tener tres casos, una velocidad nula en el infinito, una velocidad finita o una
velocidad infinita. Grficamente corresponden al perfil de potencial
E

v = 0

V(x)

E
1 2
mv
2

V(x)
x
1 2
mv
2

E
V(x)

Problemas Resueltos
2.1

Una partcula de masa m se mueve a lo largo del eje x en un campo de

fuerza que deriva del potencial V =

1 2
kx . Determinar el movimiento en torno al
2

punto de equilibrio, y calcular el perodo de oscilacin.


El potencial V (x ) tiene un mnimo en x = 0 . Este es el punto de equilibrio
estable. De la conservacin de la energa

E=

1 2 1 2
m x& + kx
2
2

despejamos la velocidad de la partcula

dx
2E k 2
=
x
dt
m m
donde arbitrariamente hemos escogido el signo positivo de la raz (movimiento hacia la
derecha). Integrando

t=

m
2E

dx
k
x
1
E
2

m
k
x +C
arcsin
k
2E

x=

2E k
sin
t +
k
m

movimiento armnico simple de frecuencia

k
m

A=

2E
k

y amplitud

Para hallar el perodo del movimiento, buscamos los puntos de retorno que
satisfacen

E = V (x )

con la solucin

x=

2E
k

Ya que el tiempo empleado para ir de un punto de retroceso al otro es independiente del


sentido del movimiento, el perodo queda determinado por la integral

T = 2 dt = 2

2E
k

dt
dx = 2
dx

2E
k

m
k
arcsin
=2
x
2E
k

2E
k

= 2

2E
k

2E
k
2E
k

dx
2E k 2
x
k m

m
k

equivalente a la relacin conocida entre el perodo y la frecuencia para un movimiento


armnico simple

T=
2.2

Una partcula de masa m se mueve a lo largo del eje x bajo una fuerza de

atraccin hacia el origen O dada por F =

k r
i . Si la partcula parte del reposo
x2

en la posicin x = a , determinar el tiempo que tarda en llegar al origen.


A partir de la expresin

r
dV r
F =
i
dx

calculamos el potencial por integracin

r v
V = F dx = Fdx

con el resultado

V=

k
k
+C
2 dx =
x
x

Elegimos la constante de manera que el potencial sea nulo en el infinito, V ( ) = 0 . Por


lo tanto, C = 0 .
Ya que la fuerza que acta sobre la partcula deriva de un potencial, la energa se
conserva siendo la suma de la energa cintica y la energa potencial

1
k
E = mx& 2
2
x

En el momento inicial, la partcula est en reposo en la posicin x = a , por lo que el


valor de la energa es

E =

k
a

A partir de la integral de movimiento, el tiempo de cada al origen est dado por

t=

dx
2
E V (x )
m

donde el signo menos indica que la velocidad es negativa, ya que el movimiento est
dirigido hacia el eje x negativo. Desarrollando la integral, con el valor dado para la
energa total

m
t=
2k

dx
ma
=
2k
1 1

x a

ma
=
a=
2k 2
2 2

x
dx
ax

ma3
k

2.3

Verificar que una partcula de masa m siempre efecta oscilaciones


armnicas respecto de los puntos de equilibrio estables del potencial V (x ) .
Sea x0 un punto de equilibrio estable para el potencial V (x ) . Por tanto, se
satisface

V (x0 ) = 0
V (x0 ) > 0

Colocamos la partcula en un punto x , cercano a x0 y estudiamos su movimiento. Si la


distancia al punto de equilibrio, x x0 , es pequea, podemos desarrollar el potencial en
el punto x , en serie alrededor de x 0 , en la forma

1
2
V (x ) V ( x0 ) + V (x0 )(x x0 ) + V (x0 )(x x0 )
2
y ya que x 0 es un mnimo de V (x ) , este desarrollo se reduce a
1
2
V (x ) V ( x0 ) + V (x0 )(x x0 )
2
Con este resultado, la fuerza que acta sobre la partcula en un punto cercano al
punto de equilibrio es

F =

dV
= V (x0 )(x x0 )
dx

y la ecuacin de movimiento correspondiente se escribe

d 2x
m 2 = V ( x0 )(x x0 )
dt

Definimos la distancia al punto de equilibrio, segn la variable s = x x0 , su


ecuacin de movimiento

d 2 s V (x0 )
+
s=0
m
dt 2

corresponde a un movimiento armnico simple de frecuencia

V (x0 )
m

Por tanto, la ley de movimiento es

x = x0 + A cos(t + )

La partcula oscila en torno al punto de equilibrio con una frecuencia y amplitud A .


Dicha amplitud debe ser pequea para que la oscilacin sea armnica. Cuando no es
pequea, los trminos de orden tres y superior en el desarrollo en serie ya no pueden
despreciarse, y la oscilacin resultante es anarmnica.

Problema Propuesto
Una partcula se mueve en el potencial V (x ) = x (6 x ). Hallar los puntos
de equilibrio y determinar su estabilidad. Estudiar el movimiento de la partcula
segn los valores de la energa total. Discutir las regiones accesibles y prohibidas a
la partcula.

2.4

Tema 2. Sistemas conservativos


Segunda parte: Potenciales centrales
Un potencial U se denomina central cuando depende solamente de la distancia
a un punto fijo O. Tomando un sistema de referencia centrado en O, el potencial slo
depende de la coordenada radial U = U (r ). La fuerza generada por la presencia de este
potencial es

r
r
r
dU r
F = U (r ) =
ur = F (r ) ur
dr

Consecuentemente, una fuerza central es aquella que est dirigida hacia, o desde, un
punto fijo O, y cuyo mdulo slo depende de la distancia a O. A continuacin
enumeramos las principales caractersticas del movimiento bajo un potencial central.

A) Se conserva el momento angular respecto a O


El momento de la fuerza respecto a O es

r r r
r r
M = r F = F (r ) r u r = 0

y de la ley de Newton para el movimiento de rotacin respecto a O

r
r dL
M=
=0
dt

se concluye que el momento angular respecto al centro de fuerza O es constante

r r
r
L = r mv = cte

Como
consecuencia, r y v deben estar contenidos en el plano perpendicular al vector
r
L , y la posicin de dicho plano no vara en el tiempo. La rbita est contenida en ese
plano. Dada la simetra del problema respecto a la rotacin segn el eje definido por el

vector L , es conveniente tomar un sistema de coordenadas polares (r , ) en el plano de


la rbita, centrado en el punto O.

B) El movimiento cumple la ley de las reas


Respecto al sistema de coordenadas polares, la posicin y velocidad tienen la
expresin

r
r
r = r ur
r
r
r
v = r& ur + r& u

El momento angular respecto a O es

v
r
r
r
r
L = r u r m r& u r + r& u = mr 2& u z

y la condicin de conservacin de su mdulo nos lleva a la constante del movimiento

L = mr 2&

Definimos la velocidad areolar como la derivada respecto al tiempo, del rea S


barrida por el radio vector al moverse la partcula en el plano de la rbita.

v areolar =

dS
dt

La ley de las reas establece que la velocidad areolar es constante, es decir, se recorren
reas iguales en tiempos iguales. Equivale a la conservacin del mdulo L del momento
angular. Como demostracin, supongamos que una partcula se encuentra en el instante
t en la posicin A, y en el instante t+dt en la posicin B.

B(t+dt)
r+
dr

dS

A(t)
x

La partcula recorre en el intervalo dt la distancia ds r d . El rea barrida es igual al


rea del tringulo AOB

1
1
dS = r (rd ) = r 2d
2
2
con lo cual la velocidad areolar es

dS 1 2 d 1 2 &
= r
= r
dt 2 dt 2
Introduciendo la conservacin del momento angular, obtenemos la frmula matemtica
de la ley de la reas

dS
1
L
=
mr 2& =
dt 2 m
2m

C) Ecuaciones del movimiento


Escribimos la ley de Newton en coordenadas polares

r
r
F = ma

donde la aceleracin del cuerpo tiene la expresin

r
r
r
a = &r& r& 2 ur + r&& + 2 r&& u

con lo cual, la ley del movimiento es

F (r ) = m &r& r& 2

d
mr 2& = 0
dt
La ltima ecuacin expresa la conservacin del momento angular.

D) Conservacin de la energa
Se define la energa de la partcula como la suma de la energa cintica y la
energa potencial

1 r
E = mv2 + U (r )
2

Derivando respecto al tiempo, vemos que la energa se conserva, es una constante del
movimiento

r r r r
r
r r
dE
= mv a + v U (r ) = v ma + U (r ) = 0
dt

E)

Potencial centrfugo. Potencial efectivo

Utilizamos la conservacin de la energa para integrar el sistema, y encontrar la


solucin que defina su movimiento. En coordenadas polares

v 2 = r&2 + r 2&2

Con ayuda del teorema de conservacin del momento angular, podemos eliminar la
variable angular , resultando

v 2 = r& 2 + r 2

L2
L2
2
&
=
r
+
m 2r 4
m2r 2

As, la energa del sistema queda definida como funcin exclusivamente de la distancia
radial r,

1 2
L2
1 2
L2
E = m r& + 2 2 + U (r ) = m r& +
+ U (r )
2
2
mr
2 mr 2

Se comprueba entonces que el problema es similar al estudiado en la primera parte de


este tema (dinmica de la partcula en una recta), ya que la partcula se ve sometida al
potencial efectivo en la direccin radial

L2
+ U (r )
2mr 2
funcin exclusiva de la variable radial r. El primer trmino se llama potencial
centrfugo y da lugar a una fuerza repulsiva radial, debida a la rotacin respecto a O
L2
Uc =
2mr 2
U ef =

F) Tipos de movimiento
La regin accesible para la partcula es aquella en la que E U ef (r ). Si E es
igual a un mnimo del potencial efectivo U ef tenemos estados de equilibrio en los
cuales r es constante (r& = 0 ). Es decir, son trayectorias circulares centradas en O. Para
energas ligeramente superiores a estos mnimos de potencial se producen oscilaciones
en torno a las rbitas circulares, que para algunos potenciales toman la forma de
trayectorias elpticas con uno de los focos en O.
Los puntos de retroceso del movimiento radial son las soluciones de
E = U ef (r ) . La trayectoria se mantiene acotada entre dos circunferencias si existen dos
puntos de retroceso radial. Si no, la partcula llega al infinito, o bien, cae al centro de
fuerzas O.

G) Clculo de rbitas. Frmulas de Binet


Partimos de las ecuaciones del movimiento

F (r ) = m &r& r& 2

L = mr 2&
Suponemos que la dependencia temporal de r es de la forma
r (t ) = r[ (t )]
con lo cual, las derivadas temporales adquieren la forma

dr d dr & dr
=
=
dt dt d
d
y

d 2 r d 2 dr d d 2 r && dr & 2 d 2 r
= 2
+
=
+
d
dt 2
dt d dt d 2
d 2
2

Derivando respecto al tiempo la ecuacin que define el mdulo del momento


angular L, encontramos

r 2&& + 2rr&& = 0

con lo cual, la derivada segunda adquiere la expresin


2
d 2 r & 2 d 2 r
1 dr
=
2
d 2
r d
dt 2

Introduciendo estas derivadas temporales en la ley del movimiento en la


direccin radial, vemos que se satisface
2
F (r ) & 2 d 2 r
1 dr
=
2 r& 2
2

m
r d
d

2
Eliminando & con ayuda de la conservacin del momento angular, llegamos a la

conclusin de que la rbita de la partcula r = r ( ) es solucin de la ecuacin


diferencial

mr 4 F (r ) d 2r
1 dr
= 2 2 r
2
r d
L
d
2

Esta ecuacin se simplifica con el cambio de variable

u=

1
r

con el resultado

d 2u
m 1
2 + u = 2 2 F
d
u L u

que se conoce como la 2 frmula de Binet, para la rbita bajo una fuerza central F (r ) .
Nos sirve para calcular la trayectoria de la partcula si conocemos la fuerza central, o
para calcular la ley de fuerza si conocemos la trayectoria de la partcula.
La 1 frmula de Binet relaciona el campo de velocidades con la rbita de la
partcula. Es decir, nos permite conocer el mdulo de la velocidad en los distintos
puntos de la rbita recorrida por la partcula. Con los mismos pasos anteriores,
2
2

L2
L2
L2 dr
2 dr
&
v = r& + 2 2 = + 2 2 = 2 4 + r 2
mr
m r d
d m r

en funcin de la variable u
2

L2 du
v = 2 + u 2
m d

Problemas Resueltos
2.5

Una partcula de masa m describe una trayectoria circular de radio R que


pasa por el centro de fuerzas O. Determinar en funcin de m, R, y de la velocidad
mnima de la partcula en su trayectoria,
a) la ley de velocidades,
b) la ley de la fuerza
c) la energa potencial de la partcula.

y
m
r
O

La trayectoria en coordenadas polares es

r = 2R cos

como se puede deducir de la siguiente grfica

90

2R

Respecto a la variable u obtenemos

u=

1
2 R cos

y de aqu,

du
sin
=
= u tan
d 2R cos2
u2
du
2
2
2
= 4R 2u 4
d + u = u (1 + tan ) =
2
cos

2

Utilizando la 1 frmula de Binet, hallamos la ley de velocidad de la partcula en


funcin de la posicin en la rbita
2

L2 du
L2 2 4 4 R 2 L2
2
v = 2 + u = 4 2 R u = 2 4
m d
m
mr

v=

2 RL
mr 2

Debemos expresar el resultado en funcin de la velocidad mnima de la masa m en la


trayectoria. La velocidad alcanza su valor mnimo cuando la coordenada radial r sea
mxima, es decir, en r = 2 R . Dicho valor mnimo es

L
2mR

v min =

con lo cual, la velocidad de la partcula satisface

2R
v = vmin

Para determinar la fuerza central F(r) que genera este tipo de rbita utilizamos la
2 frmula de Binet. Ahora

d 2u d
2u
u
2
=

+
=

+
= u + 8R 2u 3
u
tan
u
tan
u
(
)
2
2
2
d
d
cos
cos
con lo cual

Lu
1
F =
m
u
2

d 2u

8 R 2 L2u 5
8 R 2 L2
2 + u =
=
= F (r )
m
mr 5
d

En funcin de la velocidad mnima, la fuerza central tiene la expresin

mv2min 2 R
F (r ) =

R r

fuerza atractiva, inversamente proporcional a la quinta potencia de r.


La energa potencial satisface

F (r ) =

dU
dr

Integrando esta ecuacin, con la condicin de que la energa potencial se anule en el


infinito, obtenemos

mv2min
U (r ) = F (r ) dr =
R

mv 2min 2R
2R
dr
=

r
2 r

2.6

Una partcula de masa m describe una espiral bajo la accin de una fuerza

central. Deducir la ley de fuerza, si la espiral est descrita por la ecuacin r =

a
.

Utilizamos la 2 frmula de Binet. En funcin de la variable u, la rbita se


describe por la ecuacin

1
u=
a
La fuerza central que genera este tipo de movimiento satisface

L2u 2
L2u 3
L2
1

F =
(u + u ) = m = 3 = F (r )
m
mr
u
Por tanto, es una fuerza atractiva inversamente proporcional al cubo de la distancia al
centro de fuerzas.

2.7

Una partcula de masa m se mueve en una regin donde existe un potencial

central de la forma U (r ) = kr , k > 0 . Para qu valores de la energa y del


momento angular la rbita ser una circunferencia de radio a con centro en O?
Cul es el perodo de revolucin en esa rbita? Si el movimiento de la partcula
se perturba separndolo de dicho movimiento circular, cul ser el perodo de las
oscilaciones radiales alrededor de r = a .
4

Por ser el potencial central se conserva la energa y el momento angular respecto


a O. En una rbita circular de radio a, el equilibrio de las fuerzas en la direccin radial
se escribe

Fc = F (a )
siendo Fc la fuerza centrfuga, y F (a ) la fuerza central evaluada en r = a . Por
definicin, la fuerza central deriva del potencial en la forma

F (r ) =

dU
= 4kr 3
dr
y es una fuerza atractiva, dirigida hacia el centro O.

La fuerza centrfuga, generada por la rotacin respecto al centro O es

Fc = m

v2
a

con lo cual, del equilibrio de fuerzas, obtenemos la velocidad en la rbita circular

v2
= 4ka 3
a
k 2
v=2
a
m
m

La energa del sistema es suma de la energa cintica y la energa potencial

E =m

v2
4 ka 4
+ U (a ) = m
+ ka 4 = 3ka 4
2
2m

y el momento angular para la trayectoria circular es

L = mav = 2 mk a 3
El hecho de que la energa de la partcula sea menor que la energa potencial en el
infinito, en este caso equivale a que sea finita, indica que la partcula no puede escapar
al infinito. La partcula est atrapada en el campo de fuerza.
La trayectoria circular se describe con velocidad constante v, siendo el perodo
de revolucin igual a

T=

2a m
=
v
a k

Cuando la partcula se ve desplazada ligeramente de esta rbita circular, su


energa potencial U(r) admite el desarrollo en serie

1
2
U (r ) U (a ) + U (a )(r a ) + U (a )(r a )
2
y de aqu, la fuerza central que sufre la partcula, en las inmediaciones de la rbita
circular, es

F (r ) U (a ) U (a )(r a )

Suponemos que la perturbacin no afecta al momento angular de la partcula, que se


mantiene constante. La fuerza centrfuga para posiciones cercanas al movimiento
circular tambin admite el desarrollo

Fc (r ) Fc (a ) + Fc(a )(r a )

La ley de movimiento para el movimiento radial es

d 2r
m 2 = F (r ) + Fc (r )
dt
y sustituyendo los desarrollos anteriores, obtenemos la ecuacin del movimiento en la
vecindad de la rbita circular

d 2r
m 2 = U (a ) U (a )(r a ) + Fc (a ) + Fc(a )(r a )
dt
Definimos la variable para el movimiento perturbado como la distancia radial a la
rbita circular

s = r a

La ley de movimiento para la variable s es

d 2s
m 2 + [U (a ) Fc (a )]s = U (a ) + Fc (a )
dt
El lado derecho es nulo en virtud de la condicin equilibrio en la rbita circular, por lo
que obtenemos finalmente

d 2s
m 2 + [U (a ) Fc (a )]s = 0
dt
que corresponde a una oscilacin armnica cuya frecuencia de movimiento satisface

2 =

U (a ) Fc(a )
m

Evaluando las derivadas espaciales

U (a ) = 12ka 2

teniendo en cuenta el valor constante del momento angular L

Fc(a ) = 3

L2
4mka 6
=

3
= 12ka 2
4
4
ma
ma

obtenemos la frecuencia de oscilacin del movimiento perturbado alrededor de la rbita


circular

p = 2 6

k
a
m

y el perodo de las oscilaciones radiales

Tp =

m
a 6 k

lo que indica que el movimiento de oscilacin radial es ms lento que el movimiento


orbital en un factor

6.

2
, dirigida perpendicularmente
3a 3
al radio vector, a una distancia a del centro O de una fuerza central atractiva
m
4 . Calcular el tiempo de cada al centro de fuerzas.
r
2.8

Una partcula se lanza con velocidad

Utilizamos el concepto de potencial efectivo. La ley del movimiento para la


coordenada radial r se obtiene de la conservacin de la energa

1 2
L2
&
E = mr +
+ U (r )
2
2mr 2
donde U(r) es la energa potencial de la partcula. Por definicin,

U (r ) =

F (r ) dr =

m
3r 3

tomando como referencia una energa potencial nula en el infinito. Por tanto, la energa
queda expresada en la forma

1 2
L2
m
E = mr& +
3
2
2
2mr
3r
Para determinar la trayectoria radial de la partcula, despejamos la velocidad
radial

dr
2E
L2
2
r& =
=
2 2+ 3
dt
m mr
3r
tomando el signo menos por ser la velocidad radial negativa (el radio vector decrece en
el tiempo). Integrando, el tiempo de cada sobre O es

T =

dr
=
2E
2
L2

+
m m 2r 2 3r 3

dr
2E
2
L2
2 2+ 3
m mr
3r

Este tiempo puede calcularse una vez determinado el momento angular respecto a O y
el valor de la energa de la partcula.
Por ser la fuerza una fuerza central, se conserva el momento angular respecto a
O. Calculamos su valor tomando como referencia el momento inicial del movimiento

L = ma

2
2
3 =m
3a
3a

Adems, se conserva la energa. Inicialmente

1
m 2 2 m
2
E = m0 +

=0
2
2ma 2 3a 3a 3
Por tanto, el tiempo de cada est dado por la integral

T=

dr
2
2
3
3r
3ar 2

Con el cambio de variable

3
2

r dr
1 1

r a

r = a

obtenemos

T =a

3a
2

La integral indefinida tiene la solucin

con lo cual

3 / 2 d
1
0
1

3 / 2 d
3 3
= (1 ) + + arcsin
1
4 2 4

3 / 2 d 3
3
= arcsin 1 =
4
8
1
0
1

Finalmente, el tiempo de cada resulta ser

27 a 5
T =
128
2.9 Se somete a una partcula de masa m a una fuerza central
r
k r
F = 2 ur , k > 0 . Calcular su trayectoria. Cul debe ser su velocidad inicial
r
para que dicha trayectoria sea una circunferencia de radio r0 y centro O?
En funcin de la variable u, la fuerza que sufre la partcula es

1
F = ku 2
u
y la 2 frmula de Binet nos dice que la rbita

r=

1
u ( )

es solucin de la ecuacin diferencial

u + u =

m 1 mk
F =
L u 2 u L2
2

La solucin es suma de la solucin general de la ecuacin homognea


u + u = 0 , esto es, u g = A cos( 0 ) , y una solucin particular de la ecuacin

completa. Elegimos esta solucin particular como la solucin constante, u p =

mk
. Por
L2

tanto,

u = ug + u p =

mk
+ A cos( 0 )
L2

De aqu, la rbita queda definida por

r=

p
1 + e cos( 0 )

ecuacin de una cnica de parmetro p

p=

mk
L2

y excentricidad e

AL2
mk
El ngulo 0 es el formado por el eje polar = 0 , y el eje x.
e=

La trayectoria ser un circunferencia de radio r0 si e = 0 , y si p = r0 . Por tanto

r0 =

mk
L2

Por ser la fuerza central, el momento angular respecto a O se conserva. Inicialmente la


partcula lleva una velocidad V0 perpendicular al radio vector r0 , por lo que

L = mr0V0

La velocidad necesaria para la rbita circular es

m 2r02V02
r0 =
mk
k
V0 =
mr0
Adems se comprueba que en la rbita circular hay un equilibrio de las fuerzas
radiales. La fuerza central debe anularse con la fuerza centrfuga debida al giro respecto
aO

k
V02
=m
r0
r02
2.10 Una partcula se mueve en una rbita circular de radio a, bajo la accin de
una fuerza central dirigida hacia un punto O. Sean Vmax ,Vmin los valores mximo y
mnimo de su velocidad. Calcular el perodo de la rbita en funcin de estos datos.

m
Vmax

rmin

rmax

Vmin

a
En el caso ms general, la rbita circular no tiene por qu estar centrada en O.
Tomando coordenadas polares con centro en O, la rbita es una elipse. Al ser la fuerza
central, el momento angular respecto a O es constante. En los puntos donde la
velocidad es mxima o mnima, el vector velocidad no tiene componente radial, y es
perpendicular al radio vector. Por tanto, el momento angular satisface

L = mrmaxVmin = mrminVmax

Adems, ya que los puntos de velocidad mxima y mnima se encuentran en


posiciones opuestas sobre la circunferencia de radio a

rmin + rmax = 2a

Utilizando esta expresin junto con la ecuacin anterior obtenemos el momento angular
en funcin de los datos del problema

L = 2ma

VmaxVmin
Vmax + Vmin

Como la fuerza es central, se satisface la ley de las reas. Si T es el perodo de


2
revolucin en la rbita circular de radio a, cuyo radio vector barre una superficie a
tenemos

a 2
L
=
T
2m
De aqu, podemos calcular, conocido el momento angular, el perodo de la rbita

T = a

Vmax + Vmin
VmaxVmin

Problemas Propuestos
2.11 Una partcula de masa m se mueve bajo el efecto de una fuerza central de
modo que el ngulo que forma el vector velocidad y el radio vector es constante.
Calcular su trayectoria y el tiempo que tarda en caer al origen si parte de la
r r
r
posicin r = r0 en t = 0 , con velocidad v 0 .
(Para integrar la ecuacin del movimiento no hace falta conocer la forma exacta de la
fuerza central)

r 2 = r02 cos r0 v0t

Solucin: = 0 tan ln 1 cos

T=

v0t
r0

r0
v0 cos

2.12 Una partcula de masa m sometida a la fuerza central F (r ) =

k
,k >0
r2

sigue una trayectoria circular de radio r0 . Determinar su energa, momento


angular y velocidad.
Solucin: E =

1k
k
, L = mkr0 , v =
2 r0
mr0

2.13 Una partcula se mueve con velocidad constante v a lo largo de una lnea
recta que dista b del origen O. Sea dA el rea barrida en el tiempo dt por el vector
de posicin que va desde O a la partcula. Demostrar que dA/dt es constante en el
tiempo e igual a L/2m, donde L es el momento angular de la partcula respecto al
origen O.

Tema 2. Sistemas conservativos


Tercera parte: Fuerza gravitatoria
A) Campo gravitatorio
r

Una masa M crea en su vecindad un campo de fuerzas, el campo gravitatorio E ,


dado por

r
GM r
E = 2 ur
r
r
siendo ur el vector unitario radial que sale de M.

B) Fuerza gravitatoria
r

La masa m, mucho menor que M, sufre una fuerza gravitatoria F debido a M


dada por

r
r
GMm r
F = mE = 2 ur
r

C) Energa potencial gravitatoria


La masa m, en presencia de M, adquiere una energa gravitatoria dada por

r r
GMm
E p = F dr =
r

y adems se satisface

r
r
F = E p

D) Energa total
r

Como F es una fuerza conservativa, que deriva de un potencial, se conserva la


energa, como suma de la energa cintica y la energa potencial gravitatoria, dada por

1
GMm
E = mv 2
2
r

E) Cuerpos continuos
Si la masa M que genera el campo gravitatorio no es puntual, basta sustituir en
las ecuaciones anteriores M por

dM , e integrar sobre todo el volumen del cuerpo.

F) Autoenerga gravitatoria
Por definicin, la autoenerga gravitatoria de un sistema es la energa adquirida
por el sistema debida a los campos gravitatorios generados por las masas que lo
componen. Matemticamente, se define por la ecuacin

U=

U = dU
i

siendo Ui la energa potencial gravitatoria de la masa mi

G) Ley de Gauss
r

Sea S una superficie cerrada, V su volumen interior y E un campo gravitatorio


arbitrario definido en V. Se satisface la ley de Gauss para el campo gravitatorio

r r
E n dS = 4GM

r
siendo n la normal a la superficie S, dirigida hacia fuera del volumen V, y M la masa
contenida en V, interior a S.
r

Si podemos encontrar una superficie S tal que el campo gravitatorio E sea


normal a S, y constante sobre S, la ley de Gauss nos da el valor de ese campo normal
En segn la frmula

En S = 4GM (S )
siendo M(S) la masa encerrada por S.
Si la distribucin de masa tiene simetra esfrica, el campo sobre una superficie
esfrica de radio r tiene la expresin

4GM (r )
GM ( r )
=
2
4 r
r2
y coincide con el campo creado por una masa puntual M (r ) localizada en el origen.
Er =

Por tanto, como consecuencia de la ley de Gauss, el campo creado por una distribucin
esfrica de masa en su exterior es igual al campo creado por una masa puntual, del
mismo valor que la masa total de la distribucin esfrica, localizada en el origen.

Problemas Resueltos
2.14 Calcular la autoenerga gravitatoria de una esfera homognea de radio R y
masa M.
Sea la densidad de masa de la esfera. Tomamos un casquete esfrico de radio
r y espesor dr. La masa de este casquete es

dM (r ) = 4r 2 dr

La masa contenida dentro del casquete esfrico es

4
M (r ) = r 3
3

El campo producido por M (r ) en la regin donde se encuentra dM (r ) es tal que


M (r ) acta como si estuviera concentrada en el origen. Esto hecho es consecuencia de
la ley de Gauss. Podemos pensar entonces que la autoenerga gravitatoria del elemento
de masa dM (r ) se debe a la presencia de una masa M (r ) en el origen. Esto es,

GM ( r )dM (r )
r
Esta es la autoenerga del casquete esfrico de radio r. Si integramos en r desde r = 0
hasta r = R , estamos sumando la contribucin de todos los casquetes que forman la
esfera de radio R. Por tanto, la autoenerga de la esfera es
R GM (r )dM (r )
R
4
= G 4 r 4dr
U =
3
r
0
0
dU =

3 GM 2
4
3G
= R3
=
5 R
3
5R
2.15 Calcular el campo gravitatorio y el potencial gravitatorio en el interior y en
el exterior de una esfera homognea de radio R y masa M.
Utilizamos la ley de Gauss para hallar el campo. Por simetra, el campo siempre
ser radial. Para el campo interior utilizamos S como la superficie esfrica de radio
r < R . La masa contenida en S es

M (S ) = M

r3
R3

Por tanto,

Eint

4GM ( S )
r3
GMr
=
= GM 2 3 = 3
S
r R
R

Vemos que en el interior de la esfera, el campo es proporcional a r. De la ecuacin

Vint = Eint dr
obtenemos el potencial interior

GMr
GMr 2
Vint =
dr =
+C
R3
2R 3

La constante C se determinar por la continuidad del potencial en la superficie r = R .


En el exterior de la esfera, utilizamos S como superficie esfrica de radio r > R .
La masa contenida en S es igual a la masa de la esfera, M ( S ) = M . Por tanto,

Eext =

4GM
GM
= 2
S
r

Integrando, obtenemos el potencial exterior

Vext =

GM
GM
+D
2 dr =
r
r

Las constantes C y D se hallan con la condicin frontera en el infinito y con la


condicin de continuidad del potencial sobre la superficie de la esfera.

Vext ( ) = 0

Vext (R ) = Vint (R )
Con esto,

D=0

C =

GM GM
3GM

=
2R
R
2R

Los potenciales gravitatorios tienen la expresin final

GM
r
3GM GMr 2 GM 2
=
+
=
(
r 3R 2 )
3
3
2R
2R
2R

Vext =
Vint

2.16 Calcular la fuerza gravitatoria sobre una masa m situada a una distancia
x < R , del centro de una esfera homognea de radio R, a la que se le ha
practicado un orificio de radio R/2, a una distancia R/2 del origen.

m
x
R
Por el principio de superposicin, vlido para los campos gravitatorios, el sistema
es equivalente a una esfera de radio R y densidad centrada en el origen, ms una
esfera de radio R/2 y densidad , centrada en R/2. La fuerza sobre m ser la suma de
las fuerzas debidas a las dos esferas.
Para la esfera de radio R, el campo a una distancia x < R , viene dado por

E1 =

GM 1 x
R3

y la fuerza sobre m es atractiva dirigida hacia el origen

F1 = mE1 =

GmM1x
R3

De forma anloga, para la esfera de radio R/2, el campo a una distancia x del
origen es

E2 =

GM 2 (R / 2 x )
(R / 2)3

y la fuerza sobre m es

F2 = mE2 =

GmM 2 (R / 2 x )
(R / 2 )3

La relacin entre las masas de las dos esferas es

4
M 1 = R 3
3
3

4 R
M 2 = ( )
3 2
con lo cual

R
M 1 = M 2 R 3
2
La fuerza total sobre m es la suma

F = F1 + F2

y utilizando la relacin entre las masas, obtenemos

GmM1x GmM2 (R / 2 x )
GM1m
R
GM1m

+
=
x + 2 x =
3
3
3
2 R2
R
R
(R / 2)

Por tanto, la fuerza sobre la masa m no depende de la coordenada x. Tenemos un campo


gravitatorio constante en el hueco practicado en el interior de la esfera de radio R.
F =

2.17 Una distribucin esfrica uniforme de estrellas en una galaxia tiene masa
total M y radio R. Calcular la fuerza ejercida sobre una masa m que se mueve a
una distancia r < R , del centro de la galaxia. Calcular la velocidad y la energa de
m si se mueve en una rbita circular de radio r.
La fuerza de atraccin de una esfera homognea de masa M y radio R, sobre una
masa m, situada a una distancia r < R , es

F = mEint =

GMm
r
R3

siendo Eint el campo gravitatorio en el interior de la galaxia. El potencial gravitatorio


creado por la galaxia en su interior es

Vint =

GM 2
(
r 3R 2 )
3
2R

La energa potencial gravitatoria de la masa m es

E p = mVint =

GMm 2
(
r 3R 2 )
3
2R

su energa total es constante y vale

1
GMm 2
E = mv 2 +
(
r 3R 2 )
3
2
2R
Para que la rbita sea circular la fuerza de atraccin gravitatoria debe
compensarse con la fuerza centrfuga, generada por la rotacin respecto al centro de la
galaxia. Esto es,

GMm
mv 2
r=
r
R3
de donde podemos obtener la velocidad de m en dicha rbita circular

v=

GM
r
R3

Slo nos queda expresar la energa en funcin del radio r de la rbita circular, en la
forma

GMm 2
1 GM
E = m 3 r2 +
(
r 3R 2 )
3
2 R
2R
GMm 2
=
(
2r 3R 2 )
3
2R

Problemas Propuestos
2.18 Obtener el campo gravitatorio y el potencial creado por un anillo de masa
M y radio R en un punto del eje perpendicular al anillo que pasa por su centro. Si
se coloca una partcula de ma sa m sobre el eje a una distancia h del centro,
calcular su velocidad cuando pasa por el centro, la distancia que recorrer hacia el
otro lado, y bajo qu condiciones el movimiento es armnico simple.
Solucin: Ez =

V =

(R

(R

GMz

+ z2 )
GM

3/ 2

+ z2 )
1

1
v 2 = 2GM
R 2 + h 2
R
d=h
2

1/ 2

h << R , con lo cual el movimiento es armnico de frecuencia =

GM
R3

2.19 Calcular el campo gravitatorio y el potencial gravitatorio en el interior y en


el exterior de un casquete esfrico de radio R y masa M.
Solucin: Eint = 0, Vint =

Eext =

GM
R

GM
GM
,
V
=

ext
r
r2

Problemas Resueltos
2.20 Un satlite describe una rbita circular en torno a la Tierra. Si se cambia de
repente la direccin de su velocidad, pero no su mdulo, estudiar el cambio en su
rbita y en su perodo.
Al cambiar slo la direccin de la velocidad, el momento angular vara pero la
energa total no. La rbita nueva corresponder a una trayectoria asociada a una energa
negativa, que como hemos visto, puede ser una rbita circular o elptica. El nuevo valor
del momento angular es

L = mVr0 sin = L sin

siendo el ngulo entre la velocidad y el radio vector en el instante del cambio, y r0 el


radio de la rbita circular, y la energa tiene el valor

E = E =

GMm
2r0

La fuerza central que acta entre al satlite y la Tierra es inversamente proporcional al


cuadrado de la distancia entre ellos. Para dicha fuerza se satisfacen las leyes de Kepler.
La tercera ley nos dice que el perodo de una rbita slo est relacionado con la energa
total, de la forma siguiente

E=

GMm
2a

a3 GM
=
T 2 4 2
donde a es el semieje mayor de la nueva rbita, que en este caso coincide con r0 . Es
decir,

4 2 GMm
T =

GM 2E

Al no variar la energa durante el cambio de la velocidad, no vara el perodo orbital.

La segunda ley de Kepler es la ley de las reas y establece que

L S
=
2m T

siendo S el rea de la rbita y T el perodo orbital. Como el perodo no vara, T = T ,

2mS 2 mS
=
L
L

con lo cual, el rea de la nueva rbita es

S =

L
S = S sin
L

Ya que la energa no vara, la nueva rbita es una elipse centrada en O, con semieje
a = r0 , y nos falta calcular su excentricidad. Tenemos

S = r02 = a 2
y

S = ab = a2 1 e 2
con lo cual

sin =

S
= 1 e2
S

Es decir, la excentricidad de la nueva rbita viene dada por

e = cos

2.21 Un satlite de masa m describe una rbita circular a una distancia H de la


superficie terrestre. Otra partcula de masa m/2 se mueve sobre la misma rbita
pero en sentido contrario, de modo que choca con el satlite quedando unida a l.
Calcular el apogeo y el perigeo de la rbita del cuerpo compuesto.
La velocidad de una masa m en una rbita circular de radio r puede obtenerse de
la condicin de equilibrio de las fuerzas en direccin radial

V2
GMm
m
=
r
r2

V =

GM
r

La velocidad no depende de la masa en movimiento. Concluimos que la velocidad del


satlite y de la partcula antes del choque tiene el mismo mdulo y sentido contrario con
el valor

Vi =

GM
R+H

siendo R el radio de la Tierra y M su masa. Durante el choque slo actan fuerzas


interiores, y el momento angular respecto a O se conserva constante. Antes del choque

L = mVi ( R + H )

m
m
Vi (R + H ) =
GM ( R + H )
2
2

Este es el valor del momento angular para el movimiento del cuerpo compuesto,
despus del choque.
Para determinar los puntos apsidales de la nueva rbita nos falta conocer la
energa del cuerpo compuesto. Como la colisin no es elstica, la energa no se
conserva en la colisin. Sin embargo, si se conserva la velocidad del centro de masas,
que corresponde a la velocidad V del cuerpo compuesto despus del choque. Tenemos

m
m

m + 2 V = mVi 2 Vi

con lo cual

1
V = Vi
3
Justo despus de la colisin, el cuerpo compuesto lleva una velocidad V y se encuentra
a una distancia R+H del centro de la Tierra. Por tanto, su energa total tiene el valor

1
m
GM
m
E = m + V 2
m+

2
2
R+H
2
Introduciendo el valor de la velocidad, llegamos a

E=

3 1 GM
3
GM
17 GMm
m
m
=
4 9 R+ H 2 R+ H
12 R + H

Una vez conocidos los valores L y E, hallamos los parmetros de la rbita a y e,


del cuerpo compuesto. Para el semieje mayor obtenemos el valor

17 GMm
3 GMm
=
12 R + H
2 2a
9
a = (R + H )
17

La excentricidad de la rbita satisface la frmula

e = 1+

2 EL2
3
G M m
2
2

Desarrollando el radicando

16EL2
16 17 1
17
=
2
2 3 =
27 12 4
81
27G M m
encontramos

e = 1

17 8
=
81 9

La distancia de mximo acercamiento, perigeo, est dada por

rP = a (1 e ) =

9 8
1
1 ( R + H ) = ( R + H )

17 9
17

y la distancia de mximo alejamiento, apogeo, es

rA = a (1 + e ) =

9 8
1 + (R + H ) = R + H
17 9

2.22 Un satlite artificial de masa m recorre una rbita elptica, con perodo T.
Las velocidades mxima y mnima en su rbita son Vmax , Vmin respectivamente.
Determinar los parmetros de la rbita.
La ecuacin de la elipse es

a(1 e 2 )
r=
1 + e cos
Las distancias al apogeo y perigeo son

rP = a(1 e )
rA = a(1 + e)
Por conservacin del momento angular L en los puntos apsidales
mVmin rA = mVmax rP
con lo cual, de las ecuaciones anteriores, eliminando el semieje mayor a obtenemos la
excentricidad de la rbita

e=

Vmax Vmin
Vmax + Vmin

Para hallar el semieje mayor, utilizamos la ley de las reas. Si T es el perodo,


ab el rea de la elipse, la ley de las reas se escribe

ab L 1
=
= V a (1 + e)
T
2m 2 min

Como b = a 1 e , despejamos el valor del semieje mayor


2

a=

Vmin T 1 + e
2 1 e2

Tenemos

1+ e
1+ e
V
=
= max
2
1 e
Vmin
1 e
Por tanto,

a=

T
V V
2 min max

Finalmente la energa total E se encuentra a partir de la expresin

E=

GMm
2a

obteniendo

E =

GMm
Vmin Vmax

2.23 Un planeta de masa M tiene un satlite de masa m, describiendo en torno a


l una trayectoria circular de radio R, con perodo T. Sbitamente el satlite se
para. Determinar el tiempo de cada del satlite sobre el planeta.
La energa del satlite en su rbita circular es

E=

1
GMm
mV 2
2
R

Adems, hay equilibrio de las fuerzas en direccin radial. La fuerza gravitatoria se


compensa con la fuerza centrfuga

V 2 GMm
=
R
R2

con lo cual, la energa es

E=

GMm
2R

Cuando el satlite se frena, su energa cintica se hace cero, y su energa se


reduce a la energa potencial, con lo cual

E =

GMm
= 2E
R

Segn la tercera ley de Kepler, existe la relacin entre el perodo de una rbita y la
energa del sistema

2 2 2 3
T ( E ) = G M m
2
3

El perodo de la nueva rbita satisface


3/ 2

T
E
T = T =
2 2
E'
Cuando el satlite se frena, su velocidad se hace cero, y as el momento angular
de la nueva rbita es cero. Esto quiere decir que la rbita del satlite pasar por el
centro del planeta. El punto de mximo alejamiento se produce en el momento inicial,
r = R , y el punto de mximo acercamiento es r = 0 . Por tanto, el tiempo que tarda en
caer es igual al tiempo que tarda en ir de r = R a r = 0 , es decir, el tiempo que tarda
en ir del mximo alejamiento al mximo acercamiento, esto es, un semiperodo. El
tiempo de cada es, entonces

t=

T
T
=
2 4 2

Problemas Propuestos
2.24 Un satlite artificial se lanza desde la superficie terrestre verticalmente
GM
hacia arriba con una velocidad inicial U =
. En el momento en que se
R
GM
para, se le da una velocidad transversal V =
. Hallar los parmetros de la
R
rbita en funcin de y . Aqu, R es el radio terrestre.

a=

R
2 2 2

Solucin:

e = 1

4(2 2 2 ) 2

(2 )

2 2

2.25 Una nave espacial de masa m llega con una velocidad V0 a las proximidades
de la Luna siguiendo una trayectoria hiperblica cuya asntota est a una distancia
b del centro de la Luna. Sea a la distancia de aproximacin mxima de la nave al
centro de la Luna. Calcular la velocidad necesaria V0 para que b =

10
R , a = 2R ,
3

siendo R el radio de la Luna, y la velocidad en el punto de aproximacin mxima


en dicho caso. En el punto de mxima aproximacin, la nave frena para describir
una rbita circular de observacin de radio a. Calcular la energa perdida por la
nave.

V0 =

3 GM
4 R

Solucin: VP =

5 GM
4 R

GMm b 2 + a2
E =
2a b 2 a2

Tema 2. Sistemas conservativos


Cuarta parte: Movimiento planetario. Satlites
A) Ecuaciones del movimiento
Suponemos que uno de los cuerpos, de masa M mucho mayor que m, se
encuentra en reposo en el origen de coordenadas O. El otro cuerpo, de masa m, se
mueve debido a la accin del campo gravitatorio creado por M. La ecuacin del
movimiento para la masa m es

r
d 2r r r
m 2 = F (r )
dt

donde

r
GMm r
F = 2 ur
r
r
siendo ur el vector unitario radial que sale de M.
Siendo la fuerza gravitatoria una fuerza central, el movimiento de la masa m se
realiza en un plano perpendicular a la direccin del momento angular respecto a O,
vector que se conserva constante. Adems se satisface la ley de las reas, y se conserva
la energa total E de la masa m.

B) rbitas planetarias: Descripcin analtica


Segn la 2 frmula de Binet, la ecuacin de la trayectoria, en coordenadas
polares (r, ) centradas en O, es

L2
1
r=
2
GMm 1 + e cos
Aqu, e es la excentricidad de la rbita, determinada por los dos parmetros
fundamentales del movimiento orbital, la energa total E y el mdulo L del momento
angular. Para obtener su valor, evaluamos la energa total

1 2
L2
GMm
E = mr& +
2
2
r
2mr
para la rbita anterior. Al ser la velocidad radial

r& =

GMm
e sin
L

encontramos que

G 2 M 2 m3 2
E=
(e 1)
2 L2
por lo que la excentricidad de la rbita vale

2 L2 E
e = 1+ 2 2 3
G M m
El tipo de trayectoria depende exclusivamente del valor de la energa total.
Cuando E < 0 , e < 1, la trayectoria es una elipse. La masa m se encuentra atrapada en
el campo gravitatorio de M. Un caso particular corresponde al movimiento circular con
excentricidad nula. Si E = 0 , e = 1 , tenemos una parbola. La masa m llega al infinito
con velocidad nula. Si E > 0 , e > 1, tenemos una hiprbola. La masa m llega al
infinito con velocidad distinta de cero.

C) rbitas planetarias: Descripcin grfica


Utilizamos el concepto de potencial efectivo. En este caso

U ef =

L2
GMm

2
r
2mr

Segn la ecuacin de la trayectoria en coordenadas polares, r tiene las dimensiones del

L2
cociente
. Es conveniente definir la variable adimensional x en la forma
GMm 2
L2
r=
x
GMm2
con lo cual el potencial efectivo queda expresado como

U ef

G 2 M 2m3 1
1
=
2
2
x
L
2x

La grfica correspondiente

U ef

G 2 M 2m3
2 L2 x 2
x =1

G 2M 2m 3
2L2

G 2M 2 m3
L2 x

muestra que para valores pequeos de x predomina el potencial centrfugo, que no


permite que la masa m se acerque al origen, donde se sita la masa M, mientras que
para valores grandes de x el potencial gravitatorio es ms importante, dificultando que
la masa m se escape al infinito. El mnimo del potencial se produce en el punto x = 1 .
Por tanto, en la regin x < 1 , el potencial efectivo decrece, y la fuerza resultante es
repulsiva, mientras que en la regin x > 1 , el potencial efectivo crece, y la fuerza
resultante es atractiva.
Pasamos ahora a determinar las trayectorias posibles dependiendo del valor de la
energa total. Para ello dibujamos de nuevo el potencial efectivo y una trayectoria de
energa definida E.

U ef
E4
E3
E2

G 2 M 2m3
2L2

x =1

E1

Si la energa es menor que el mnimo del potencial, no existe movimiento posible.


Cuando la energa es igual a E1 , igual al mnimo del potencial, la nica regin donde
puede encontrarse la masa m es la circunferencia x = 1 . La trayectoria es una

GMm
L2
circunferencia de radio
, que se recorre a velocidad constante
. Si la
2
L
GMm
energa es igual a E 2 , mayor que el mnimo de potencial, pero menor que cero, existen
dos puntos de retroceso, y la trayectoria es una elipse, contenida entre las dos
circunferencias correspondientes a dichos puntos de retroceso. Si la energa es igual a
E3 = 0 , slo existe un punto de retroceso, y la masa m puede llegar al infinito con
velocidad nula, describiendo una parbola. Si la energa es igual a E4 mayor que cero,
slo existe un punto de retroceso y la masa m puede llegar al infinito con velocidad no
nula, describiendo una rama de hiprbola.
Las trayectorias correspondientes a estos valores definidos de la energa se
dibujan en la siguiente grfica

E4
E3
E2

E1
O

y se estudian individualmente.

a) rbita elptica

b
a

r
c = ae

A partir del conocimiento de L y E, obtenemos los parmetros que caracterizan


una elipse: la excentricidad e y el semieje mayor a. Los puntos apsidales son los puntos
de la trayectoria donde la velocidad radial se hace cero. Corresponden a las posiciones
de mximo y mnimo alejamiento del centro de fuerzas O. El perigeo o distancia de
mnimo alejamiento corresponde a la posicin angular = 0 , y segn la figura vale

rP = a c = a ae = a(1 e )

El apogeo o distancia de mximo alejamiento corresponde a la posicin angular = ,


y segn la figura vale

rA = a + c = a + ae = a(1 + e)

El semieje menor b est dado por

b = a 2 c2 = a 1 e2
El rea de la elipse es

S = ab = a 2 1 e 2

La ecuacin de la trayectoria, en virtud de la definicin de los puntos apsidales,


puede escribirse como

1 e2
r=a
1 + e cos
donde, como en el caso general, la excentricidad vale

e = 1+

2L2 E
G 2 M 2 m3

Queda por determinar el semieje mayor de la rbita a. Utilizando el hecho de


que en un punto apsidal la energa cintica radial es nula, y la energa total es igual al
potencial efectivo evaluado en dicho punto, junto con la igualdad

L2
a(1 e ) =
GMm 2
2

obtenemos en el perigeo el valor de la energa total

E=

GMm
2a

Es una de las propiedades fundamentales del potencial gravitatorio, la energa es


independiente del valor del momento angular.
Por ultimo, la relacin entre la velocidad y la coordenada radial sobre la
trayectoria puede obtenerse a partir de la conservacin de la energa

GMm 1
GMm
= mV 2
2a
2
r
GM a
V2 =
2 1
a r

Todas estas frmulas son vlidas en un movimiento circular, para el que la


excentricidad es nula.

b) rbita parablica

r
O

En este caso, la excentricidad es igual a 1. La ecuacin de la trayectoria es

r= p

1
1 + cos

siendo

p=

L2
GMm 2

el llamado semilatus rectum. No existe distancia de mximo alejamiento, ya que

r si =

pero s existe una distancia de mnimo acercamiento, o perigeo, cuyo valor es

rP =

p
2

La energa total es cero, y la velocidad se relaciona con la coordenada radial


sobre la rbita segn la frmula

1
GMm
0 = mV 2
2
r
GM
V2 = 2
r
Se observa que la partcula llega al infinito con velocidad nula, ya que

V 0 si r

c) rbita hiperblica

La ecuacin de la trayectoria es

e2 1
r=a
1 + e cos

con excentricidad e > 1 , siendo vlida como en el caso general la frmula

2 L2 E
e = 1+ 2 2 3
G M m
Para esta trayectoria la energa total es mayor que cero, en particular

E=

GMm
2a

De aqu, la relacin entre la velocidad y la coordenada radial se expresa

GMm 1
GMm
= mV 2
2a
2
r
GM a
V2 =
2 + 1
a r
La masa m llega al infinito con velocidad no nula

V=

GM
si r
a

No existe distancia de mximo alejamiento, pero s existe una distancia de


mnimo alejamiento, o perigeo, con el valor

rP = a(e 1)

D) Leyes de Kepler
La primera ley establece que los planetas se mueven en rbitas elpticas con el
Sol en uno de sus focos.
La segunda ley es la ley de las reas, ya estudiada.
La tercera ley establece que los cuadrados de los perodos de revolucin sobre las
rbitas son proporcionales a los cubos de los semiejes mayores. Demostracin
2
2
Si T es el perodo orbital, y el rea de la elipse de semiejes a y b, es S = a 1 e ,
por la ley de las reas obtenemos la relacin

L a 2
=
1 e2
2m
T
Por otro lado, sabemos que en una rbita elptica se satisface

a(1 e 2 ) =

L2
GMm 2

De estas dos ecuaciones, eliminando L y e, obtenemos

a3 GM
=
T 2 4 2

Tema 3. Dinmica del slido rgido


Primera parte: Movimiento plano
1. Grados de libertad
Para caracterizar el movimiento de un slido rgido plano nos bastan tres
coordenadas o grados de libertad. Dos de ellas nos dan la posicin del centro de masas.
La tercera, un ngulo, nos da la orientacin del slido respecto de su centro de masas.

( xcm , ycm )

2. Campo de velocidades
r

Sean O y P dos puntos del slido rgido. La relacin entre sus velocidades VO y

r
VP es

siendo

r
r
r uuur
VP = VO + OP
r d r
=
uz
dt

el vector velocidad angular, que define la velocidad de giro respecto a un eje que pasa
por el centro de masa y es perpendicular al plano del slido.
Tomando el punto O como origen de un sistema de coordenadas mvil ligado al
slido, y P como cualquier punto arbitrario del slido, obtenemos la primera
interpretacin del movimiento de un slido plano:
'' El slido gira con velocidad
angular respecto al punto O, que a su vez se
r
desplaza con velocidad VO ''

3. Centro instantneo de rotacin


En el movimiento de un slido plano, siempre existe un punto CIR exterior o
interior al slido que se encuentra, en el instante dado, en reposo. Su posicin no est
fijada, sino que va cambiando con el tiempo. Es el centro instantneo de rotacin.
Respecto a l, un punto arbitrario P del slido tiene una velocidad

r
r
r uuuuuuur r uuuuuuur
VP = VCIR + CIR, P = CIR , P

As, tenemos la segunda interpretacin del movimiento:


'' El slido plano gira con velocidad angular respecto a su centro instantneo de
rotacin CIR ''
En cada caso prctico, para determinar la posicin del centro instantneo de
rotacin, basta considerar que segn la frmula anterior, la velocidad de cada punto del
slido es perpendicular a su vector de posicin respecto al CIR. Es decir, si dibujamos
las lneas perpendiculares a las velocidades conocidas de dos puntos del slido, el CIR
se encuentra situado en su punto de corte.

4. Momento lineal
Ya que el slido es un conjunto de partculas individuales, el momento lineal
total es igual a

r
r
p = MVcm

siendo M la masa del slido, y Vcm la velocidad de su centro de masa.

5. Momento angular y momento de inercia


Determinamos el momento angular de giro del slido respecto a su centro de
r
masa O, siendo el plano xy el plano del slido, y el eje z la direccin del vector . Sea
la densidad superficial de masa. Entonces

r
r
r
LO = r ( dS ) V

donde la velocidad de cada punto del slido se relaciona con la velocidad del centro de
masa

r r
r r
V = VO + r

Por definicin del centro de masa, y ya que nuestro origen se sita en el propio
centro de masa, se satisface

r dS =MrO = 0

con lo cual, el momento angular de giro resulta ser

r
r r r
r r r r
LO = r ( r ) dS = r 2 r ( r ) dS
r r

Por ser un slido plano r = 0 ya que es perpendicular a r y adems

r 2 = x 2 + y 2 , con lo cual
r
r
r
r
LO = r 2 dS = ( x 2 + y 2 ) dS = IO

2
2
siendo IO = ( x + y ) dS el momento de inercia del slido plano respecto al eje

que pasa por su centro de masa.

6. Teorema de Steiner
Si calculamos el momento angular respecto al centro instantneo de rotacin
CIR, llegamos a la frmula anloga

r
r
LCIR = I CIR

siendo

( x, y )

ICIR = ( x2 + y2 ) dS
las nuevas coordenadas de cada punto del slido respecto a un sistema

centrado en CIR. Si d es la distancia entre el centro de masa O y el centro instantneo


de rotacin CIR, ya que

r r r
r = r d

r r
r 2 = r 2 + d 2 2 r d

y r dS =0 , se satisface

r r
ICIR = r2 dS = ( r 2 + d 2 2r d ) dS = r 2 dS +d 2 dS

relacin que constituye el teorema de Steiner

ICIR = IO + Md 2
El momento de inercia respecto al eje que pasa por el centro instantneo de rotacin es
igual al momento de inercia respecto al eje que pasa por el centro de masa, ms el
producto de la masa total por el cuadrado de la distancia entre el centro de masa y el
centro instantneo de rotacin.

7. Energa cintica
Si tomamos como origen el centro de masa O,

T=

1 2
1 r
r r
1
1
V

dS
=
(VO + r )2 dS = MVO2 + IO 2

2
2
2
2

suma de la energa cintica de traslacin del centro de masa, y la energa cintica de


rotacin respecto al centro de masa.
Si tomamos como origen el centro instantneo de rotacin CIR,

T=

1 2
1 r r
1
V dS = ( r )2 dS = ICIR 2

2
2
2

Toda la energa cintica proviene del movimiento de rotacin respecto al CIR.


En cada caso debe decidirse que expresin es ms adecuada, el valor de T es el mismo.

8. Ecuaciones del movimiento


Tomando un sistema de coordenadas centrado en O, el movimiento de traslacin
se rige por el sistema de ecuaciones

M
M

d 2 xcm
dt 2

= Fx

d 2 ycm
= Fy
dt 2

siendo ( xcm , ycm ) la posicin del centro de masa me dida en unos ejes fijos en el
espacio, y el movimiento de rotacin respecto al centro de masa por la ecuacin

dLO
d
= IO
= IO
dt
dt
siendo M O el momento de las fuerzas aplicadas y la aceleracin angular producida.
MO =

Para un sistema de coordenadas centrado en el CIR slo necesitamos resolver la


ecuacin ligada a la rotacin del slido

M CIR = I CIR

9. Condicin de deslizamiento
Al estudiar el movimiento de slidos planos en contacto con superficies estticas
debemos suponer, para simplificar lo ms posible el clculo, que en cada instante slo
existe un punto de contacto entre el slido y la superficie. Dicho contacto produce
friccin y as, debemos incluir en nuestro modelo una fuerza de rozamiento que d
cuenta de este hecho. Como hemos visto en un tema anterior, la magnitud de la fuerza
de rozamiento depende principalmente de la velocidad del punto del slido que est en
contacto con la superficie. En general, podemos escribir la velocidad del punto de
contacto en la forma

VP = Vcm R

siendo R su distancia al centro de masa. Analizamos dos tipos de comportamiento:

a) Rodamiento sin deslizamiento


En este caso, el slido rueda sin deslizar cuando la velocidad del punto de contacto sea
nula. Al ser esta velocidad nula, la fuerza de rozamiento no tiene una magnitud
definida, sino que su valor ser el suficiente como para mantener este rgimen de
movimiento. El punto de contacto tiene la propiedad adicional de ser el centro
instantneo de rotacin del slido. Podemos escribir entonces que un slido rueda sin
deslizar cuando

Vcm = R
Fr = ?
b) Rodamiento con deslizamiento
En este caso, el punto de contacto tiene una velocidad no nula, lo que produce que el
slido deslice sobre la superficie. Es decir, Vcm R . Al ser esta velocidad diferente

de cero, la fuerza de rozamiento tiene la magnitud definida Fr = N , siendo N la


fuerza normal que ejerce la superficie sobre el slido.

10. Colisin entre un slido rgido y una masa puntual


En general, la colisin se describe de la forma siguiente. Una masa puntual es
lanzada hacia un slido rgido en reposo, impactando con l, de forma que se le
comunica al slido tanto momento lineal como mo mento angular. El momento lineal
absorbido produce un movimiento de traslacin del centro de masa, y el momento
angular absorbido produce un movimiento de rotacin respecto del centro de masa. As
pues, en el estudio de este tipo de colisiones, nuestro objetivo es doble: determinar la
velocidad de traslacin (velocidad del centro de masa) y la velocidad de rotacin
(velocidad angular). Para ello, debemos servirnos de las dos leyes de conservacin
imperantes en toda colisin: la ley de conservacin del momento lineal

r
r
pi = MVcm
r
r
Li = I cm

y la ley de conservacin del momento angular

La dificultad aadida en este tipo de problemas es el clculo de la posicin del centro de


masa, y del momento de inercia respecto al centro de masa.

Problemas Resueltos
3.1

A un cilindro de radio R y masa M, se le aplica una fuerza horizontal F en


un punto situado a una distancia d de su centro de masa, de forma que rueda sin
deslizar. Calcular la aceleracin del centro de masa y la fuerza de rozamiento
necesaria para que el cilindro efectivamente ruede sin deslizar.
F

( xcm , R )

Fr

Las ecuaciones del movimiento son

F Fr = Macm

para el movimiento de traslacin del centro de masa, y

Fd + Fr R = Icm

para el movimiento de rotacin respecto al centro de masa. De la condicin de


rodamiento sin deslizamiento Vcm = R , obtenemos, derivando respecto al tiempo, la
relacin entre la aceleracin del centro de masa y la aceleracin angular

acm = R
Adems, el momento de inercia del cilindro respecto de su eje tiene el valor

1
Icm = MR 2
2
Resolviendo simultneamente estas ecuaciones, obtenemos la aceleracin del
centro de masa

acm =
y la fuerza de rozamiento necesaria

Fr =

2d+R F
3 R M
R 2d
F
3R

Comprobamos que cuando 2d = R no se produce fuerza de rozamiento, y la


aceleracin del centro de masa corresponde a un movimiento de traslacin sin rotacin,
bajo la accin nicamente de la fuerza aplicada F. Esta solucin no es satisfactoria
puesto que dicho movimiento de traslacin debera ser frenado por la fuerza de
rozamiento Fr = N , que aqu no ha sido incluida.

3.2

Una bola de billar, de masa M y radio R, se encuentra en reposo sobre una


mesa horizontal, con la que presenta un coeficiente de rozamiento . Se le golpea
con un taco en direccin horizontal, a una altura R, comunicndole una velocidad
inicial V0 . Calcular la distancia recorrida por la bola en rgimen de deslizamiento,
antes de empezar a rodar sin deslizar.
En el instante inicial, el golpe producido por el taco de billar est dirigido hacia el
centro de la bola, por lo que slo le comunica un movimiento de traslacin, pero no de
rotacin. Entonces, la velocidad del punto de contacto tiene el valor

VP = Vcm R = V0 0 R = V0 0

con lo que inicialmente la bola desliza sobre la mesa horizontal.


La nica fuerza que acta sobre la bola de billar es la producida por el
rozamiento, y ya que el rgimen es de deslizamiento, su valor es Fr = N = Mg .
Se genera as una deceleracin constante del centro de masa

acm =

Fr
= g
M

Integrando en el tiempo, teniendo en cuenta su valor inicial, la velocidad del centro de


masa en el instante t es

Vcm = V0 gt

y as queda resuelto totalmente el movimiento de traslacin.


En cuanto al movimiento de rotacin, la fuerza de rozamiento genera un
momento de fuerzas positivo M = Fr R = MgR , que da lugar a un movimiento de
giro con la aceleracin angular constante

M
MgR 5 g
=
=
I cm 2 MR 2 2 R
5

Integrando en el tiempo, teniendo en cuenta su valor inicial, obtenemos la velocidad


angular de giro en el instante t

5 g
t
2 R

Vemos que, tras el golpe inicial, el movimiento de traslacin se va frenando y el


movimiento de rotacin se va acelerando. Todo ello debido a la presencia de una fuerza
de rozamiento constante. La bola de billar cambia de rgimen, y comienza a rodar sin
deslizar en el instante en el que Vcm = R . A partir de entonces, la fuerza de
rozamiento tendr el valor necesario para que se mantenga el rodamiento sin
deslizamiento. Calculamos el tiempo en el que se produce el cambio. Se debe satisfacer

V0 gt =

5 g
Rt
2 R

con lo cual

t=

2 V0
7 g

Calculamos ahora la distancia recorrida por la bola de billar en ese tiempo. Integrando
en el tiempo la velocidad del centro de masa obtenemos su posicin

1
xcm = x0 +V0 t gt 2
2
La distancia recorrida desde que se produce el golpe hasta que la bola cambia de
rgimen es igual a

2 V0
2 V02 12 V02
d = xcm x0 = V0

=
7 g 49 g 49 g
3.3

A una bola de billar, de masa M y radio R, se le comunica una velocidad


angular inicial 0 , sin velocidad lineal inicial. Calcular la distancia que recorre
hasta empezar a rodar sin deslizar.
Inicialmente la bola de billar desliza sobre el suelo horizontal, ya que la
velocidad del punto de contacto es distinta de cero

VP = Vcm R = 0 0 R = 0 R 0
La nica fuerza que acta es la fuerza de rozamiento Fr = N = Mg , con sentido
positivo, de signo contrario a la velocidad negativa del punto de contacto. La
aceleracin del centro de masa es positiva

acm =

Fr
= g
M

mientras que el momento de fuerzas generado por la fuerza de rozamiento


M = Fr R = MgR es negativo, y produce una desaceleracin constante del
movimiento de rotacin

M
MgR
5 g
=
=
2
I cm
2 R
MR 2
5

Integrando en el tiempo las aceleraciones correspondientes, teniendo en cuenta


sus valores iniciales, obtenemos la velocidad lineal y angular de la bola de billar en el
instante t

Vcm = gt
= 0

5 g
t
2 R

El movimiento cambia de rgimen cuando Vcm = R , en el instante dado por

5
gt = 0 R gt
2
2 0 R
t=
7 g
La distancia total recorrida en ese tiempo es

xcm x0 =

1
2 02 R 2
gt 2 =
2
49 g

3.4

Una escalera doble, formada por dos partes iguales de longitud L y masa M
cada una, se mantiene en reposo formado un ngulo = 30 con la vertical,
gracias a una cuerda EF situada entre sus puntos medios. Si en el instante inicial se
corta la cuerda, calcular la velocidad con que el extremo A llega al suelo. Resolver
el problema sin hacer uso del centro instantneo de rotacin, y haciendo uso de l.

L
E

En el momento que se corta la cuerda EF, el punto A se mueve verticalmente


hacia abajo, mientras que B y C lo hacen horizontalmente hacia afuera. El sistema
queda determinado por una sola coordenada . La velocidad del punto A viene dada por

VA = 2L& cos(90 ) = 2L& sin


Para hallar su valor al llegar al suelo, donde = 90 , utilizamos la conservacin de la
energa.
Para el tramo AC de escalera, la velocidad del centro de masa es, en mdulo,

VAC =

L &

Su momento de inercia respecto a un eje que pasa por el centro de masa es

Icm =

1
ML2
12

La energa cintica de AC es suma de la energa cintica de traslacin del centro de


masa y la energa cintica de rotacin respecto al centro de masa. Con esto

( Ec ) AC =

1
1
1
1 1
1
2
MVAC
+ I cm& 2 = ML2& 2 +
ML2&2 = ML2& 2
2
2
8
212
6

De manera anloga, dada la simetra de la escalera, la energa cintica del otro tramo de
escalera es

( Ec ) AB =

1
ML2&2
6

Evaluando la energa potencial de cada tramo de escalera en los respectivos


centros de masa, vemos que

( E p ) AC = Mg ( ycm ) AC = Mg L2 cos
( E p ) AB = Mg ( ycm ) AB = Mg L2 cos
E p = MgL cos
Con esto, la energa total de la escalera durante su movimiento de cada satisface

1
E = Ec + E p = ML2& 2 + MgL cos
3
Inicialmente cada tramo de la escalera parte del reposo formando un ngulo de 30 con
la vertical. Por tanto, el valor de la energa es

1
3
3
E = ML2 02 + MgL
= MgL
3
2
2
En el instante final, al llegar al suelo = 90 , de la conservacin de la energa
obtenemos el valor de la velocidad angular final

MgL

3 1 2 &2
= ML + MgL cos90
2 3

3 3g
& =
2 L
y de aqu la velocidad del punto A al llegar al suelo
VA = 2 L& sin90 = 6 3 gL
Ahora referimos el movimiento al centro instantneo de rotacin. Para determinar
la posicin del CIR de cada tramo de escalera, escogemos dos puntos del slido cuya
direccin de velocidad sea conocida. En este caso es fcil ver que la velocidad del punto
A siempre est dirigida verticalmente hacia abajo, y la velocidad del punto C siempre
est dirigida horizontalmente hacia fuera. Dibujando dos lneas perpendiculares
encontramos el CIR en su confluencia.

CIR

VA

cm

VC

La energa cintica es la energa cintica de rotacin respecto al CIR

( Ec ) AC = ( Ec ) AB =

1
ICIR& 2
2

siendo

ICIR = Icm + Md 2
y d la distancia entre el centro instantneo de rotacin y el centro de masa, que, segn la
figura, tiene el valor

d=

L
2

Con esto, calculamos el valor de la energa cintica total de la escalera en su cada

1 1
1
1

Ec = 2 ML2 + ML2 &2 = ML2& 2


2 12
4
3

valor que coincide con la energa cintica total del movimiento referido al sistema del
centro de masa de cada tramo de escalera. A partir de aqu el clculo es el mismo que
antes, con el mismo resultado final.

3.5

Una varilla homognea de masa m y longitud L, cuelga horizontalmente


suspendida de dos hilos verticales sujetos a ambos lados del centro de la varilla y a
la distancia x de l. Cortamos uno de los hilos. Calcular, en funcin de x, la tensin
que soporta el otro hilo en el instante inmediato al corte.

x
m
mg

Como el punto de suspensin no coincide con el centro de masa, y queremos


calcular el valor de la tensin, fuerza que se aplica sobre el punto de suspensin, es
aconsejable estudiar el movimiento como la combinacin del movimiento de traslacin
del centro de masa, que satisface

mg T = macm
y del movimiento de rotacin respecto al centro de masa, que satisface

Tx = I cm

Como el punto de suspensin se mantiene siempre en reposo, se satisface adems la


condicin de no deslizamiento

acm = x

siendo, en este caso, x el radio de giro del centro de masa respecto del punto de
suspensin.
Teniendo en cuenta que el momento de inercia de la barra de longitud respecto
de un eje que pasa por su centro de masa es

Icm =

1
ML2
12

podemos despejar de estas ecuaciones el valor de la tensin del hilo justo despus del
corte, obteniendo

mgI cm
mgL2
T=
=
mx 2 + I cm L2 + 12 x2
Cuando el hilo que no se ha roto est unido directamente al centro de masa, en cuyo
caso x = 0 , no existe rotacin de la varilla, y se obtiene el valor conocido T = mg .

3.6 Las dos poleas de la figura son cilndricas y tienen la misma masa m y radio
R. Cuando el sistema se abandona a s mismo, calcular las aceleraciones angulares
de las poleas y la tensin de la cuerda.

T
m

La polea superior mantiene un movimiento de rotacin gobernado por la


ecuacin del movimiento

1
TR = mR 21
2
de forma que le comunica a la cuerda que se desenrolla una aceleracin lineal igual a

a1 = 1 R

(Se supone que la cuerda no desliza en ningn momento sobre la polea superior)

La polea inferior tiene dos movimientos, uno de rotacin por el efecto del
momento de fuerzas generado por la tensin de la cuerda

TR =

1
mR 2 2
2

y un movimiento de traslacin en cada vertical por influencia del peso y la tensin de


la cuerda

mg T = ma2

Para poder resolver estas ecuaciones debemos imponer una condicin adicional
de deslizamiento de la cuerda sobre la polea inferior. Suponiendo que tal deslizamiento
no se produce, el movimiento vertical de la cuerda debe coincidir con el movimiento de
traslacin del punto de contacto con la polea inferior. Es decir,

a1 = a 2 2 R

la aceleracin lineal de la cuerda es igual a la aceleracin lineal de la polea menos la


aceleracin lineal producida por el giro sobre s misma.
Con las ecuaciones anteriores calculamos las aceleraciones angulares y la tensin
de la polea

1 = 2 =

2g
5R

1
T = mg
5
Una masa puntual m se mueve con velocidad V0 , dirigida segn el eje x, y
choca con un sistema de dos masas m, en reposo, y unidas por una barra rgida de
longitud, colocada en la direccin y. Despus del choque, la masa incidente queda
unida a una de las masas del sistema. Calcular la velocidad y posicin del centro de
masa y la velocidad angular del sistema alrededor del centro de masa despus del
choque. Calcular la energa perdida en el choque.

3.7

V0

y
m
x

Se conserva el movimiento del centro de masa y el momento angular respecto al


centro de masa. La posicin del centro de masa justo antes del choque viene dada por

2my = m(d y )
1
y= d
3
Se encuentra a una distancia d/3 del extremo donde quedan adheridas las dos masas m.
La velocidad del centro de masa despus del choque se obtiene de la ecuacin de
conservacin del momento lineal

mV0 = 3mVcm
1
Vcm = V0
3
y est dirigida en la misma direccin que la velocidad V0 de la masa incidente.
En el movimiento de rotacin respecto al centro de masa, cuya posicin hemos
calculado en el primer apartado, el momento angular respecto al centro de masa justo
antes del choque es

Li = mV0

d
3

Despus del choque, el sistema de masas se comporta como un slido rgido, con un
momento angular

L f = I cm
siendo Icm el momento de inercia de las masas respecto a su centro de masa, y la
velocidad angular de rotacin del sistema respecto a su centro de masa. De la
conservacin del momento angular respecto al centro de masa, obtenemos la velocidad
angular de giro despus del choque

mV0d
3I cm

Conociendo las posiciones relativas de las masas, podemos deducir su momento


de inercia
2

Icm
y de aqu, el valor de

2
d
2d
2
= 2m + m
= md
3
3
3

1 V0
2d

La energa cintica antes del choque es

1
Ec ,i = mV02
2

Despus del choque, la energa cintica es suma de la energa cintica de traslacin del
centro de masa con velocidad Vcm , y la energa cintica de rotacin respecto al centro
de masa con velocidad angular , resultando

1
1
1
1 1 2 1
2
2
2
Ec , f = 3mVcm
+ I cm 2 = +
mV0 = mV0
2
2
4
6 2 3 4
La energa perdida en el choque es

1
Ec , i Ec , f = mV02
4
equivale a la mitad de la energa inicial.

3.8

Un jugador de bisbol dispone de un bate, que es una barra homognea de


masa M y longitud D. Golpea la pelota a una distancia x del extremo libre. En el
impacto ejerce una fuerza F durante un intervalo de tiempo t muy corto.
Calcular la velocidad angular del bate despus del impacto, la velocidad lineal del
punto donde el jugador sujeta el bate, y el valor de x para que el jugador no sienta
en su mano el impacto de la pelota.
Durante el impacto, la pelota comunica al bate un impulso de mdulo

p = F t

Por conservacin del momento lineal, la velocidad del centro de masa del bate despus
del impacto es

Vcm =

F t
M

Adems se conserva el momento angular respecto al centro de masa del bate.


Inicialmente su valor es

D
F t ( D 2 x )
Li = p x =
2
2

y despus del impacto, el bate gira como un slido rgido con momento angular

L f = I cm =

1
MD2
12

Por tanto, de la conservacin del momento angular, obtenemos el valor de la velocidad


angular de giro despus del impacto

6F t ( 2D x )
MD 2

Una vez resuelto el problema de la dinmica del bate despus del impacto,
calculamos la velocidad lineal del punto P de la empuadura, en forma vectorial

r
r
r r
VP = Vcm + rP

con mdulo igual a

VP = Vcm

D F t 6 x

=
2

M D
2

El jugador no siente el impacto si su mano no se mueve despus del impacto, esto es, si
la velocidad del punto P es cero. Para ello x debe ser igual a

x=

D
3

Es decir, el jugador debe golpear la pelota a una distancia D/3 del extremo del bate.

Problemas Propuestos
3.9

Una bola de billar de masa M y radio R se golpea como indica la figura,


comunicndole un impulso p,

( xcm , R )

Fr

iniciando su movimiento sobre una mesa horizontal que presenta un coeficiente de


rozamiento . Calcular:
a) Velocidad del centro de masa y velocidad angular de giro en el instante inicial
b) la condicin que debe verificar el ngulo de impacto para que la bola deslice.
c) distancia que recorre la bola antes de iniciar el movimiento de retroceso
d) la condicin que se debe cumplir para que se produzca el movimiento de
retroceso
e) una vez iniciado el retroceso, distancia que recorre la bola antes de cambiar de
rgimen y rodar sin deslizar
f) velocidad del centro de masa y velocidad angular de giro en ese instante
Solucin:

a) Vcm =

p
5 p
sin , =
cos
M
2 MR

b) La bola desliza para cualquier ngulo de impacto


p2
c) xcm x0 =
sin 2
2
2 m g
d) > 45

25 p 2
2
e) xcm
x0 =
cos sin )
2 (
98 m g
5 p
5 p
f) Vcm =
( cos sin ) , =
( cos sin )
7M
7 MR
3.10 Una biela OS de longitud b est articulada con la manivela QS de longitud
a. El extremo O se mueve a lo largo del eje QR sin sufrir rozamiento, y el punto Q
est fijo.

a
Q

Determnese:
a) la relacin entre las velocidades angulares de OS y QS
b) qu tipo de movimiento realiza el sistema respecto al punto O ?
c) el centro instantneo de rotacin
d) en qu punto de la trayectoria conjunta el sistema realiza una traslacin sin
rotacin?
Solucin:

a
cos
&
b b2 a2 sin2
b) traslacin de O ms rotacin alrededor de O
c) En el punto de corte de la recta radial dibujada desde Q y
la recta vertical dibujada desde O
d) = 90
a) & =

3.11 Una pelota maciza de radio R rueda sin deslizar por un plano inclinado y
sube por el interior de un anillo de radio d. Calcular la altura inicial h desde la que
debe dejarse caer la pelota para que pueda describir el anillo.
Solucin: h =

27d 17 R
10

3.12 Dos proyectiles de masa M y velocidad V, se incrustan simultneamente en


una barra de masa M y longitud D. Una de las masas golpea a la barra en uno de
sus extremos, y la otra golpea a la barra a una distancia D/4 del centro, incidiendo
sobre ella del lado opuesto a la primera masa. Antes del impacto, la barra
descansa en una mesa horizontal sin rozamiento. Calcular la velocidad angular de
la barra, y las velocidades de sus extremos despus del impacto.
M

D
4

V
M cm

D
2

2V
9D
5
Solucin: VA = V
54
2
VB = V
27
=

V
M

Tema 3. Dinmica del slido rgido


Segunda parte: Movimiento general
1. Grados de libertad
Ahora tenemos seis grados de libertad, tres para la posicin del centro de masa, y
tres para la rotacin alrededor del centro de masa.

2. Eje instantneo de rotacin


Consideramos slo el caso de un slido con un punto fijo E, cuya posicin dentro
del slido puede cambiar en el tiempo. En tal caso, la velocidad de un punto arbitrario P
del slido es

r
r uuur
VP = EP
Se comprueba as que aquellos puntos P que se encuentran en la recta paralela al vector
r
velocidad angular que pasa por E estn en reposo tambin en ese instante de tiempo.

Es decir, esa recta es el conjunto de puntos del slido que se encuentran en reposo
simultneamente en dicho instante. Esa recta forma el eje instantneo de rotacin,
efectuando el slido un movimiento de rotacin, sin traslacin, en torno a ese eje.
Cuando no existe tal punto E, el movimiento general del slido es combinacin de
un movimiento de traslacin en direccin paralela a un determinado eje, y un
movimiento de rotacin en torno a dicho eje.

3. ngulos de Euler
La posicin del slido con un punto fijo E queda determinada definiendo un
conjunto de tres ngulos de rotacin independientes. El conjunto ms utilizado lo
forman los llamados ngulos de Euler, que pasamos a definir. Sea OXYZ un sistema de
referencia fijo, y Oxyz un sistema de referencia ligado al slido, que comparte el mismo
origen con el sistema anterior. Dicho origen se sita en el punto fijo E. Giramos un
ngulo en torno al eje OZ, y dibujamos la llamada lnea de los nodos en el plano XY.
Los ngulos de Euler quedan definidos como sigue:

es el ngulo entre el eje OX y la lnea de los nodos (arbitraria)


es el ngulo entre el eje fijo OZ y el eje mvil Oz
es el ngulo entre la lnea de los nodos y el eje Ox, en el plano xy

O
Y

x
Lnea de los nodos

Los rangos de variacin de los ngulos de Euler son

0 < 2
0 <

0 < 2

4. Velocidad angular en funcin de los ngulos de Euler


Las velocidades angulares asociadas a los ngulos de Euler reciben los nombres de
precesin ( & ), nutacin (& ) y espn (& ).

Sea un el vector unitario en la direccin de la lnea de los nodos. La velocidad


angular del slido tiene la expresin directa

r
r
r
r
= &uZ &u n + & u z

Es ms til escribir el vector velocidad angular tomando como referencia los ejes fijos
del slido, que estn definidos en virtud de su simetra espacial. Escribiendo los
vectores unitarios anteriores en funcin de los vectores unitarios segn los ejes fijos en
el slido,

r
uZ = cos
r
un = cos

r
r
r
uz sin sin u x sin cos u y
r
r
ux sin u y

obtenemos las componentes del vector velocidad angular en la forma

x = & sin sin & cos


= & sin cos + & sin
y

z = & cos + &

5. Matriz de inercia. Ejes principales. Momento angular


Se define la matriz de inercia I como la matriz que tiene las componentes

Iij = ( r 2 ij xi x j ) dV

donde es la densidad de masa del slido, r la distancia del elemento de masa


arbitrario del slido dm = dV al eje de giro,
r
vector r para ese elemento de masa, y

{ xi } las

coordenadas cartesianas del

1 i = j
ij =

0 i j

El momento angular L de un slido rgido arbitrario tiene las componentes

Li =

I ij j

j = x, y , z

A partir de ahora, ser mas conveniente renombrar los ejes del slido con los nmeros
1,2,3 en lugar de las letras x,y,z. As, las componentes del momento angular pueden
escribirse en la forma

L1 = I111 + I122 + I13 3


L2 = I 211 + I22 2 + I233
L3 = I 311 + I322 + I333
Una de las propiedades fundamentales que nos permite describir el movimiento de
cualquier slido rgido de forma sencilla es consecuencia del carcter matricial del
momento de inercia. En virtud de su propiedad de simetra, Iij = I ji , siempre nos ser
posible encontrar un sistema de ejes dentro del slido, llamados ejes principales, tales
que la matriz del momento de inercia sea diagonal, en la forma Iij = diag ( I1 , I2 , I3 ) ,
siendo Ii los llamados momentos principales de inercia. Tomando como referencia
estos ejes destacados, el clculo se simplifica puesto que las componentes del momento
angular slo dependen de la componente en la misma direccin de la velocidad angular,

L1 = I11
L2 = I 2 2
L3 = I3 3
En general, los ejes principales coinciden con los ejes de simetra del slido, y se
identifican con los ejes xyz fijos al slido, mencionados anteriormente, respecto a los
cuales deben definirse los ngulos de Euler.

6. Energa cintica
Tomando como referencia los ejes principales centrados en el punto fijo del slido,
la energa cintica tiene la expresin

Ec =

1
1
1
I112 + I2 22 + I 332
2
2
2

7. Ecuaciones del movimiento


Tomando como referencia unos ejes fijos en el espacio, la ecuacin para el
movimiento de rotacin del slido rgido respecto a un eje que pasa por su punto fijo, es

r
r dL
M =
dt

siendo M el momento de las fuerzas exteriores, y L el momento angular de giro


medido en el sistema de ejes fijo en el espacio.
Si nos referimos a los ejes principales del slido la ecuacin del movimiento es

r
r dL r r
M =
+ L
dt r
que se denomina ecuacin de Euler, siendo L el momento angular de giro medido en

r
r
r
r
L = I11 u1 + I2 2u2 + I 3 3u3

el sistema de ejes fijo en el slido

y el vector velocidad angular, medido en el sistema de ejes fijo en el slido

r
r
r
r
= 1u1 + 2 u2 + 3u3

Por tanto, por componentes, la ecuacin de Euler se escribe

d1
+ ( I 3 I2 ) 23
dt
d 2
M 2 = I2
+ ( I1 I3 )31
dt
d
M 3 = I3 3 + ( I 2 I1 )1 2
dt
M 1 = I1

y su solucin conjunta nos da la evolucin temporal de las componentes de la velocidad


angular, respecto a los ejes principales del slido.

8. Utilidad de la ecuacin de Euler


a) Para el estudio de sistemas no equilibrados, sistemas que no se mantienen en
equilibrio al realizar un movimiento de rotacin debido a que el vector velocidad
angular de giro no est dirigido segn un eje principal. Por efecto de esta falta de
simetra, aparecen momentos de fuerza que tienden
a desequilibrar el sistema, y por
r
tanto, ser necesario aplicar un par de fuerzas M que elimine la descompensacin del
sistema. En otros casos, se puede realizar una labor de equilibrado variando los
momentos de inercia del cuerpo en rotacin hasta obtener el estado de equilibrio
deseado.
En general, el problema se simplifica considerando que la velocidad angular de rotacin
es constante. Con ayuda de las ecuaciones rde Euler, calculamos partiendo del valor
r
conocido de la velocidad angular , el par M creado. Elegimos los ejes 123 como los
ejes principales del sistema.

b) Para
r el estudio de sistemas giroscpicos. En este caso, se conoce de antemano el
par M que se aplica al slido, y se desea obtener las velocidades de rotacin del
sistema en funcin de los ngulos de Euler, para casos especiales. Con esta
informacin, es posible describir de forma completa el movimiento del sistema
giroscpico. Para resolver el problema, debemos utilizar la relacin entre las
componentes 1,2,3 de la velocidad angular y los ngulos de Euler.

Problemas Resueltos
3.13 Un rbol de longitud L est apoyado en dos cojinetes P y Q, y lleva
colocadas asimtricamente dos masas, iguales a m/2 a una distancia a del eje del
rbol, formando con l un ngulo . El conjunto gira con velocidad angular
r
constante . Calcular la fuerza necesaria que hay que aplicar sobre los cojinetes
para mantener el movimiento.

m/2
a

D/2

m/2

D/2

Los ejes principales del slido coinciden con sus ejes de simetra, que se definen
como se muestra en la figura.

El eje 1 es perpendicular al plano formado el brazo y el eje central del rbol.


Contenidos en ese mismo plano, el eje 3 es perpendicular al brazo donde se encuentran
las masas, y el eje 2 est dirigido a lo largo de los brazos del rbol.
Los momentos principales de inercia corresponden a los momentos de inercia de
masas puntuales, y son

I1 = 2

m 2
a = ma 2
2

I2 = 0
I3 = I 1 = ma 2
y el vector velocidad angular viene dado, en el sistema de ejes principales, por

r
r
r
r
= 1u1 + 2 u2 + 3u3
1 = 0
2 = cos
3 = sin

Las ecuaciones de Euler, suponiendo que la velocidad angular de giro del rbol es
constante, se escriben

M 1 = ( I3 I 2 ) 2 3
M 2 = ( I1 I 3 )31
M 3 = ( I 2 I1 )1 2

En nuestro caso, con los datos anteriores, el momento de fuerzas que sufre el rbol
debido a su movimiento de rotacin no equilibrado (ya que el vector velocidad angular
no est dirigido segn un eje principal) tiene las componentes

M 1 = ( I 3 I 2 ) 23 = ma2 2 sin cos


M 2 = ( I1 I3 ) 31 = 0

M 3 = ( I 2 I1 )1 2 = 0

Las fuerzas aplicadas F en los puntos P y Q deben generar un par de fuerzas que
anule este momento no nulo M 1 < 0 .
2

F
D/2

D/2

M1
F
1

Deben estar dirigidas en sentido contrario a las agujas del reloj y ser tales que

D
= M1
2
a2 2
F = m sin cos
D

2F

3.14 Calcular la velocidad de precesin en torno al eje vertical de un trompo


pesado que gira sobre s mismo con velocidad , mucho mayor que & .
z

&

mg
Y

Ya que ? & , el momento angular del sistema es prcticamente igual al momento


angular respecto al eje z de simetra, debido a la rotacin del trompo sobre s mismo.
Entonces,

r
r
L = I 3 u z

Sin embargo, la variacin temporal del vector L se debe, ya que es constante, a la

precesin del eje z del cuerpo en torno al eje vertical Z con velocidad angular & . Se
debe cumplir

r
r dL
r r
r r
r
M =
= & u Z L = &I 3 uZ u z = &I3 sin u x
dt

siendo M el momento de las fuerzas exteriores (el peso del trompo) respecto al punto
fijo del trompo

r r
r
r r
r
M = r mg (uZ ) = mga uz uZ = mga sin u x

siendo a la distancia medida sobre el eje del trompo desde el punto fijo hasta el centro
de masa.
Igualando las dos ecuaciones, obtenemos la precesin del eje del trompo respecto a
la vertical

mga
& =
I 3
3.15 Establecer las ecuaciones de movimiento de un cuerpo rgido simtrico con
respecto a un eje, tiene un punto fijo sobre ese eje. Discutir el movimiento
rotacional del cuerpo, suponiendo que no existen fuerzas externas.
Escogemos el eje de simetra como eje principal. Las ecuaciones de Euler para un
cuerpo simtrico, tal que I1 = I 2 , en ausencia de fuerzas externas, son

d1
+ ( I3 I2 ) 2 3 = 0
dt
d 2
I2
+ ( I 2 I 3 ) 31 = 0
dt
d
I3 3 = 0
dt
I2

La tercera ecuacin nos dice que 3 es una constante del movimiento

3 = C

Introduciendo este resultado en las ecuaciones restantes, obtenemos

d1
+ ( I 3 I 2 ) 2 C = 0
dt
d 2
I2
+ ( I 2 I 3 )1C = 0
dt
I2

De aqu, podemos extraer las ecuaciones del movimiento para cada una de las
componentes de la velocidad angular en la forma

d 21
2
I
+ ( I 3 I 2 ) C 21 = 0
2
dt
d 2 2
2
+

I 22
I
I
C 2 2 = 0
(
)
3
2
2
dt
2
2

con la solucin conjunta general

1 = A cos t
2 = A sin t
siendo

I 3 I2
C
I2
r

Concluimos entonces que el vector de velocidad angular tiene una magnitud


constante

= 12 + 22 + 32 = A2 + C 2
y que precesa en torno al eje 3 con una velocidad angular . El ngulo formado por
el vector de velocidad angular y el eje 3 el cuerpo satisface

C
cos = 3 =

A2 + C 2
3.16 Calcular el momento de fuerza necesario para que el disco de radio a y
masa m, gire con velocidad angular constante , en una direccin que forma un
ngulo constante con el eje vertical. Calcular la precesin respecto al eje
vertical.
z

y
x

Elegimos los ejes (xyz) como los ejes principales del disco, de forma que el vector
r
est contenido siempre en el plano yz. La velocidad angular del sistema es,
incluyendo la precesin en torno al eje vertical,

r
r
r
= sin u2 + cos + & u3

El momento de inercia del disco respecto del eje vertical es

1
I3 = ma 2
2
y utilizando el teorema de ejes perpendiculares para un cuerpo slido plano

I3 = I 1 + I 2

obtenemos por simetra, los otros dos momentos de inercia

I1 = I 2 =

1
1
I 3 = ma 2
2
4

Con ayuda de las ecuaciones de Euler, obtenemos el momento de fuerza necesario


para mantener el movimiento del disco

1
M 1 = ( I 3 I 2 )23 = ma 2 sin cos + &
4
M 2 = ( I1 I3 )31 = 0

M 3 = ( I 2 I1 )1 2 = 0

El momento angular del sistema viene dado por

L1 = 0
1
L2 = ma2 sin
4
1
L3 = ma2 cos + &
2

y adems es constante sobre los ejes mviles del disco (recordar que la velocidad
angular se encuentra siempre en el plano yz, pero dicho plano precesiona respecto a la

vertical con velocidad angular & ) . La variacin temporal de L se debe, pues, a la


precesin respecto al eje vertical. Se cumple
r entonces
r

r
M = & u3 L

ecuacin referida a los ejes mviles. La nica componente no nula del momento de
fuerzas M 1 = & L2 nos da la condicin que debe satisfacer la precesin

1
1
ma 2 sin cos + & = ma 2 sin&
4
4
con la solucin

1
& = cos
2
y de aqu, el momento de fuerzas necesario para mantener el movimiento se reduce a

1
1
M 1 = ma 2 sin cos + & = ma 2 2 sin cos
4
8

Problemas Propuestos
3.17 Un cilindro homogneo de masa m, radio R y altura H gira en torno a un
eje que forma un ngulo de 30 con el eje del cilindro y que pasa por su centro de
masa. Calcular el momento de fuerzas que se debe aplicar en el soporte del
cilindro para mantener el movimiento.
z

30
y
x

Solucin: M 1 =

3
m 2 (3 R 2 H 2 ) , M 2 = 0, M 3 = 0
48

3.18 Una barra de longitud L y masa despreciable, se une a un disco de masa m y


radio a. El conjunto gira respecto al punto fijo de contacto E. El disco gira
respecto a s mismo con velocidad angular . Calcular la precesin & del sistema
respecto al eje vertical, suponiendo que no hay nutacin ( permanece constante),
que ? & y que L ? a .

&

mg

E
g
Solucin: & =

Tema 4. Relatividad especial


Primera parte: Relatividad de Galileo
1. Principio de relatividad
Las leyes de la mecnica son las mismas en dos sistemas de referencia, si se mueven
de modo que su velocidad relativa sea constante.+
Se define un sistema inercial como el sistema de referencia en el que una partcula,
en ausencia de fuerzas, se mueve con velocidad constante.

2. Transformacin de Galileo
r
Consideramos dos sistemas inerciales S y S, cuya velocidad relativa V est
dirigida segn el eje Ox. Se supone que el plano yz es paralelo al plano y'z', y los ejes
x, x' son coincidentes. Las ecuaciones que relacionan las coordenadas de los dos
sistemas
x = x + Vt
y = y
z = z
reciben el nombre de transformacin de Galileo para S y S. Por tanto, por el
principio de relatividad, las leyes de la mecnica son las mismas si realizamos sobre el
sistema una transformacin de Galileo.

La ley de transformacin para la velocidad y la aceleracin es


r r r
v = v + V
r r
a = a
Vemos as que la aceleracin no depende del sistema de referencia elegido.

3. Efecto Doppler no relativista. Corrimiento de


Este efecto relaciona la frecuencia medida de una onda con las velocidades relativas
del transmisor T, el medio y el receptor R. Suponiendo que el transmisor emite ondas
cada Te segundos, queremos determinar el perodo de la onda observado por el
receptor. La onda viaja en el medio con una velocidad V. El transmisor se mueve con
velocidad VT hacia el receptor, y el receptor se mueve con velocidad V R hacia el
transmisor.

Por tanto, en virtud de la adicin no relativista de velocidades, la distancia entre


transmisor y receptor disminuye VT + VR metros en cada segundo, y la onda llega al
receptor con la velocidad relativa V + VR . El transmisor emite una onda de perodo Te ,
siendo t = 0 el instante en el que emite el primer frente de onda y t = Te el instante en
el que emite el ltimo frente de onda, y finaliza la emisin. Si ambos observadores, el
transmisor y el receptor, estuvieran en reposo relativo VR = VT = 0 , el intervalo de
tiempo transcurrido entre la recepcin del primer frente de onda y la recepcin del
ltimo frente de onda sera Te .
Durante dicho intervalo de tiempo, la distancia entre el transmisor y el receptor
disminuye, con lo cual el ltimo frente de onda debe recorrer una distancia menor para
llegar al receptor, que la distancia que debe recorrer el primer frente de onda. Las
distancias recorridas por el primer y ltimo frente de onda difieren en la cantidad
d = (VR + VT ) Te
Ya que la velocidad con que se acerca la onda al receptor es V + VR , la diferencia entre
el tiempo de vuelo del primer frente de onda y el tiempo de vuelo del ltimo frente de
onda es
d
V +V
t =
= R T Te
V + VR V + VR
Por tanto, el perodo de la onda observado por el receptor ser igual al perodo emitido
menos la diferencia de tiempo ganada debida al movimiento de acercamiento del
transmisor y receptor. Es decir,
V +V
V VT
TR = Te t = 1 R T Te =
Te
V + VR
V + VR

Introduciendo la frecuencia de la seal


1
T=

obtenemos la relacin buscada


V + VR
R =
e
V VT
Es el efecto Doppler no relativista. La frecuencia medida de una onda depende de la
velocidad relativa del receptor y transmisor respecto a la velocidad de las ondas en el
medio.
El caso que ms no interesa se refiere a un receptor en reposo (radar en la superficie
terrestre), y un transmisor (galaxia en emisin espectral) movindose con una velocidad
pequea frente a la velocidad de las ondas en el medio estelar. Con esto, el cambio de
frecuencia por efecto Doppler puede aproximarse al valor
V
V
R =
e ; 1 + T e
V VT
V

De aqu, podemos calcular la variacin relativa en la frecuencia medida por el


observatorio terrestre
e VT
R = R
=
e
V
En el caso ms general, las ondas luminosas que llegan a la Tierra procedentes de
galaxias lejanas se estudian a travs de su espectro de longitudes de onda. La longitud
de onda se define en la forma
V
= = VT

La variacin de la longitud de onda debido al efecto Doppler, en esta aproximacin, es


e
V
R = R
= T
e
V
Cuando una galaxia se aleja de la Tierra, VT < 0 , obtenemos R > e . La luz que
recibimos procedente de esa galaxia sufre un corrimiento hacia longitudes de onda
mayores, es el llamado corrimiento al rojo. Si la galaxia se acerca a la Tierra, VT > 0 ,
obtenemos R < e , y se produce un corrimiento hacia longitudes de onda menores. Es
el llamado corrimiento al azul.

4. Velocidad relativa de las galaxias. Edad del Universo


Segn hemos visto en el punto anterior, midiendo el desplazamiento en longitud de
onda del espectro que llega a la superficie terrestre emitido por cualquier galaxia,
podemos estimar cul es la velocidad relativa de esa galaxia respecto a la Tierra. Para
estimar la distancia que nos separa de ella nos basamos en el modelo cosmolgico ms
aceptado actualmente.
Dicho modelo acepta que el universo se encuentra en continua expansin, y que
dicha expansin es uniforme en cualquier direccin y respecto a cualquier punto del
universo. Por tanto, respecto a la Tierra, todas las galaxias se alejan de forma uniforme,
por lo que una galaxia situada a una distancia r de la Tierra, se debe alejar de nosotros a
una velocidad
Vg = r
siendo un valor de referencia que vale aproximadamente
= 1018 seg -1
Podemos estimar adems con este modelo cosmolgico la edad del Universo, y su
tamao, caracterizado por un cierto radio R. La frontera del Universo est formada por
los rayos de luz que se emitieron en su nacimiento, y se alejan de cualquier punto con
una velocidad igual a la velocidad de la luz. La distancia que han recorrido es igual al
radio del universo, que satisface

c
= 1018 seg-luz = 3 1010 aos-luz

y la edad del universo ser igual al tiempo necesario para que los rayos de luz recorran
esta distancia
1
= cR = = 1018 seg = 3 1010 aos

R=

4.1

La velocidad de la corriente de un ro es Va , y el ro tiene anchura L. Una


lancha se desplaza con velocidad Vr respecto al agua en reposo. Qu direccin
debe tomar la lancha para llegar a la otra orilla en el menor tiempo posible?
Calcular el punto de atraque.
Escogemos el eje x en la direccin de la corriente del ro, paralela a la orilla, y el eje
perpendicular y a la orilla del ro. Por la ley de composicin de velocidades, la
velocidad de la lancha respecto a la orilla es igual a
r r
r
V = Va + Vr

Vx = Va + Vr sen
V y = Vr cos
siendo el ngulo que forma la trayectoria de la lancha con la perpendicular a la orilla
del ro. Si las velocidades son constantes, la integral de la ley del movimiento de la
lancha es
x = (Va + Vr sen ) t
y = Vr cos t

Cuando y = L la lancha se encontrar en la otra orilla del ro. El tiempo que tarda
en cruzar el ro viene dado por
L
t=
Vr cos
y la posicin en la que llega en la otra orilla est dada por
V + Vr sen
x= a
L
Vr cos
El tiempo de cruce es mnimo cuando la trayectoria sigue la lnea recta definida por el
V
valor = 0 . En este caso, la posicin del punto de llegada es x = a L , y = L .
Vr

4.2

Un nadador que parte de A, se desplaza con velocidad constante Vn respecto


al agua sobre un ro de anchura d cuyas aguas estn animadas de una corriente de
velocidad constante Vr .
a) El nadador efecta trayectos, de longitud d, de ida y vuelta: cruzando la orilla,
AA1 A , en un tiempo t1 y de forma paralela a la orilla, AA2 A , en un tiempo t2 .
Calcular el valor del cociente t1 / t2
b) El nadador deja el borde A, cuando se encuentra a una distancia d de la proa
de una motora, de anchura L, que se desplaza de forma paralela a la orilla con
una velocidad constante u respecto del agua. Determinar la direccin y
magnitud de la velocidad absoluta mnima del nadador para que no choque con
la embarcacin.

A1

a)

r
Vr

b)

r
Vr

r
Vn

r
u

A2

La ley de composicin de velocidades es


r r r
V = Vn + Vr
r
siendo V la velocidad absoluta del nadador respecto a la orilla. Para ir de A a A1 el
r
nadador debe formar un ngulo con la normal a la orilla, de forma que V sea
perpendicular a la orilla, perpendicular entonces a la velocidad del ro. As, se debe
satisfacer
Vn2 = V 2 + Vr2
con lo cual la velocidad absoluta del nadador es
V = Vn2 Vr2

El tiempo t1 que tarda en cruzar el ro de anchura d y volver est dado por


d
d
d
t1 = 2
=2 =2
Vn cos
V
V 2 V 2
n

En el viaje AA2 , de longitud d, la velocidad del nadador respecto a la orilla es

Vn Vr , y en el viaje de vuelta, A2 A , de longitud d, la velocidad es Vn + Vr . El tiempo


que tarda en recorrer la distancia AA2 A es

t2 =

2dV
d
d
+
= 2 n2
Vn Vr Vn + Vr Vn Vr

El cociente de tiempos nos queda


t1
V2
= 1 r2
t2
Vn

Tomamos el instante inicial t = 0 cuando el nadador sale de A. En ese instante, la


motora se encuentra en x = d y viaja con velocidad u respecto del agua. El nadador
evita la embarcacin si llega al punto B, a una distancia L perpendicular a la orilla, en
primer lugar.
El tiempo que tarda el nadador en llegar a B es
L
tn =
Vn cos
siendo el ngulo que forma el nadador con la normal a la orilla. El tiempo que tarda la
motora en llegar al mismo punto es
d + L tan
tm =
u Vr
siendo u Vr la velocidad absoluta de la motora respecto de la orilla. El nadador evita
la colisin con la motora, si tn < tm . La velocidad del nadador para conseguir esto
satisface, por tanto
L ( u Vr )
L ( u Vr )
Vn >
=
( d + L tan ) cos d cos + L sen
El mnimo de esta velocidad se alcanza cuando
dVn
=0
d

d sen L cos = 0
L
tan =
d
Esto es, cuando el nadador se dirige en lnea recta hacia B. La velocidad mnima es,
entonces,
L (u Vr )
Vn =
d 2 + L2

4.3

Una lnea espectral que aparece con una longitud de onda de 5000 en el
laboratorio se observa con 5000.2 en el espectro de la luz emitida por una
galaxia lejana. Calcular su velocidad y posicin desde la Tierra.
Si e es la longitud de onda emitida por la galaxia y R , la longitud de onda
recibida en la Tierra, el corrimiento por efecto Doppler viene dado por
Vg
R e
=
=

e
c
Introduciendo los datos, obtenemos
R
5000.2 5000
Vg = c e
=c
= 4 10 5 c
e
5000.2
La galaxia se aleja de la Tierra.
La distancia r desde la Tierra satisface
Vg = r
siendo un valor de referencia que vale aproximadamente
= 10 18 seg -1 = 3 10 10 aos 1
Obtenemos
r : 105 aos-luz

4.4

Las velocidades de las galaxias relativas a la Tierra no son istropas en el


firmamento, ya que dependen de la direccin de observacin. La anisotropa es
consecuencia del movimiento del Sol respecto al centro de nuestra galaxia, con una
velocidad de 300 km/s. Examinamos todas las galaxias situadas a una distancia de
3.26 x 107 aos-luz. En el laboratorio la lnea de emisin del hidrgeno tiene una
longitud de onda de 6563 . Calcular los valores mximo y mnimo de la longitud
de onda recibida en la Tierra, y la direccin de observacin en que se producen.
En este problema debemos suponer que la Tierra no est en reposo, sino que se
mueve con una velocidad de arrastre V R , debido al movimiento del Sol, que vale 300
km/s=10-3 c, y est dirigida segn una cierta direccin que elegimos como el eje z. En
este caso el desplazamiento Doppler viene dado por
1
1 c + VR
=
R e c Vg
siendo V g la velocidad de alejamiento de las galaxias
Vg = r

Con esto,
R = e

Con los datos del problema,

c +r
c + VR

Vg : 10 10 3,26 107 c = 0,00326c

La velocidad de la Tierra es de 300 Km/s si la direccin de observacin coincide


con la direccin de su movimiento (eje z), y es de -300 km/s si la direccin de
observacin es opuesta a la direccin de su movimiento. Cuando la direccin de
observacin forma un ngulo con la direccin del movimiento de la Tierra, la
velocidad de traslacin de la Tierra vale
VR = 300cos km/s
Por tanto, el mximo en la longitud de onda recibida se produce cuando VR es
mnima, es decir, en la direccin -z, y el mnimo en la longitud de onda se produce
cuando VR es mxima, es decir, en la direccin +z. Con los datos del problema,
c + 0,00326c
max = e
: 6591,0 A
c 0,001c
y
c + 0,00326c
min = e
: 6577,8 A
c + 0,001c

4.5

Un astronauta desea conocer su velocidad de aproximacin a la Luna. Para


ello enva una seal de frecuencia 5000 MHz y compara esta frecuencia con su eco,
observando una diferencia de 86 Hz. Calcular la velocidad del astronauta respecto
a la Luna.
Suponemos un receptor en reposo (la Luna), y el transmisor aproximndose con una
velocidad VT (el astronauta). La frecuencia recibida por la Luna sufre un
desplazamiento por efecto Doppler en la forma
1
V
R = e
: e 1 + T
V
c

1 T
c
expresin vlida si VT = c , siendo c la velocidad de la luz, velocidad a la que se
propaga la seal enviada por el astronauta.
La frecuencia reflejada por la Luna es

R = R

Esta seal llega de nuevo al astronauta con una frecuencia


1
V
e = R
: R 1 + T
V
c

1 T
c

En esta recepcin, el astronauta est en reposo y la Luna se acerca a l con velocidad


VT .
La relacin entre las frecuencias de la seal emitida y del eco recibido es, entonces
2

V
e = e 1 + T
c

Vemos que la frecuencia del eco siempre ser mayor que la frecuencia emitida, ya que
el astronauta se acerca a la Luna. Con los datos del problema, encontramos la velocidad
de aproximacin del astronauta a la Luna

5000,086

VT = c e 1 = c
1 : 2,58km/s
5000
e

Tema 4. Relatividad especial


Segunda parte: Cinemtica relativista
1. Principio de relatividad especial
Como nos demuestra nuestra experiencia fsica, todos los observadores inerciales
deben ser equivalentes, no slo respecto a la dinmica, como ya descubri Newton, sino
tambin respecto a la propagacin de la luz. Esto implica que la velocidad de la luz
debe ser la misma en todas direcciones, independientemente del estado de movimiento
del observador, probado que sea un observador inercial. Y esto nos lleva
ineludiblemente al principio de relatividad de Einstein: todos los observadores
inerciales son fsicamente equivalentes, as que no puede realizarse ningn experimento
fsico que discrimine de alguna forma los observadores inerciales entre s.
Una de las consecuencias del principio de relatividad es que el tiempo deja de ser un
concepto absoluto, ya que cada observador mide su tiempo propio. La experiencia
diaria nos muestra que dos relojes sincronizados siempre marcarn el mismo tiempo,
sea cual sea su movimiento relativo. Pero esta experiencia est adquirida en un contexto
muy reducido, cuando la velocidad relativa es pequea frente a la velocidad de la luz.
Mantener que esta sincronizacin se produce incluso a velocidades relativas cercanas a
la velocidad de la luz va en contra del principio de la constancia de la velocidad de la
luz. Por tanto, el tiempo es una cantidad relativa, que depende del camino recorrido por
el observador.

2. Proporcionalidad entre intervalos de tiempo


Consideramos los dos observadores inerciales mostrados en la figura. Uno de ellos,
O, se encuentra en reposo en la posicin x = 0 en todo instante de tiempo. El otro, O,
se mueve con velocidad constante V a lo largo del eje x, y su posicin medida por el
observador O es x = x0 + Vt .
tiempo

nosa
lumi
l
a

Se

t
O

x0

Debido a la uniformidad del movimiento entre los dos observadores inerciales y de


la constancia de la velocidad de la luz, si uno de ellos emite seales luminosas a
intervalos de tiempo t, el otro observador recibir las seales a intervalos de tiempo
medidos en su propio sistema de referencia, como muestra la figura. De esta manera se
cumple la llamada proporcionalidad entre intervalos
= Kt
donde K es una constante que depende slo de la velocidad relativa entre los dos
observadores, y no de su posicin relativa. Si la proporcin es igual a la unidad, ambos
observadores estn en reposo relativo. Si es mayor que la unidad, los observadores se
alejan entre s, y si es menor que la unidad, los observadores se acercan entre s.
Adems, si el observador en movimiento cambia el sentido de su movimiento, la nueva
constante de proporcionalidad ser
1
K =
K
As, se comprueba que la propagacin de la luz es muy diferente de la propagacin del
sonido, que tena en cuenta la velocidad del transmisor y del receptor (efecto Doppler
no relativista). En este tema, la teora de la relatividad se va a desarrollar partiendo del
principio de relatividad y de la proporcionalidad de intervalos temporales, ya que nos
aporta una visin ms fsica que el desarrollo formal de la relatividad especial a partir
de las frmulas de transformacin de Lorentz.
La utilidad de la proporcionalidad de intervalos radica en lo siguiente. En los
ejemplos prcticos tratados en relatividad, slo es posible sincronizar los relojes de dos
observadores en movimiento relativo en el instante que ambos observadores se
encuentren en el mismo punto del espacio, que tomaremos como origen de
coordenadas. As, segn muestra la figura, la proporcionalidad de los intervalos de
tiempo t y
tiempo

O
a
inos
lum
l
a

Se

t
O

es equivalente a la proporcionalidad entre el instante de emisin t de una seal medido


por un observador inercial O, y el instante de recepcin de esa seal por otro
observador inercial O, medido por ese observador O. A partir de aqu, utilizamos el
concepto de la proporcionalidad de intervalos en este sentido.

3. Transformacin de Lorentz
Un suceso relativista se define como un acontecimiento que en el espacio-tiempo
tiene una coordenada espacial y una coordenada temporal (se supone que el movimiento
se restringe a una dimensin).
La transformacin de Lorentz es el conjunto de frmulas que define la relacin
entre las coordenadas ( x, t ) de un suceso relativista medidas por un observador O en
reposo, y las coordenadas

( x, t )

medidas por un observador O en movimiento

uniforme con velocidad V . Se escriben en la forma


x = ( x Vt )
Vx
t = t 2
c
donde hemos introducido el factor definido por
1
>1
=
V2
1 2
c

Para obtener la relacin inversa basta sustituir las coordenadas primadas por las
coordenadas sin prima y cambiar de signo a la velocidad relativa, ya que el observador
en reposo se mueve con velocidad V respecto al observador en movimiento.

4. Contraccin de la longitud
Una de las consecuencias del carcter relativo del tiempo es la contraccin de la
longitud. La distancia medida por un observador en reposo depende del estado de
movimiento del objeto medido. En particular, la distancia medida decrece al aumentar
la velocidad del objeto. Este fenmeno puede estudiarse directamente a partir de la
transformacin de Lorentz.
Imaginemos el movimiento uniforme con velocidad V de una barra homognea.
Cualquier observador unido a la barra mide una longitud propia L0 . Calculamos la
longitud L de la barra medida por el observador O en reposo. Tomando incrementos en
la transformacin de Lorentz, obtenemos
x = ( x V t )
Ahora tenemos que identificar el significado de x,x , t recordando la nocin de
suceso relativista. As, x ser la separacin espacial de dos sucesos relativistas
cualesquiera que sirvan para medir la longitud de la barra desde el punto de vista del
observador ligado a la barra. Por ejemplo, se pueden enviar sendas seales a los

extremos de la barra, y medir el tiempo de retorno. Obviamente, x = L0 . De forma


anloga, x = L .
Adems t ser la diferencia de tiempo entre los sucesos relativistas anteriores
desde el punto de vista del observador en reposo. Es fcil ver que t = 0 puesto que el
observador en reposo conoce exactamente la longitud de la barra cuando recibe
simultneamente las seales desde sus puntos extremos. Por tanto, hemos establecido
que la longitud vista por el observador en reposo satisface
L0 = L
En general, se establece este fenmeno en su significado inverso
1
L = L0 < L0

Es decir, un observador en reposo siempre mide una longitud menor que la longitud
medida por el observador en movimiento (ligado a la barra).

5. Dilatacin del tiempo


De forma anloga, el movimiento del observador produce una variacin en el
tiempo medido. De nuevo, este fenmeno puede explicarse partiendo de la
transformacin de Lorentz
Vx
t = t 2
c

Para ello, necesitamos precisar un poco ms la nocin de tiempo, subrayando el carcter


intrnseco que tiene para cada observador individual. Hablamos de tiempo propio de
un observador como el tiempo medido para sucesos que ocurren en su sistema de
referencia, respecto a los cuales el observador est en reposo. Relacionamos as el
tiempo propio medido por el observador en movimiento con el tiempo propio t
medido por el observador en reposo.
En la frmula anterior, t corresponde al tiempo propio del observador en
movimiento si los sucesos ocurren en su sistema de referencia. Respecto al observador
en reposo, la posicin de ese sistema de referencia del observador en movimiento
satisface x = Vt . Por tanto, t = si x = Vt . Entonces, la relacin entre los tiempos
propios queda establecida en la forma
V2
V2
= t 2 t = 1 2 t
c
c

Introduciendo el valor del factor , obtenemos


1
= t <t

Es la dilatacin del tiempo: para un observador en movimiento, el tiempo transcurre


ms lentamente que para un observador en reposo.

6. Adicin relativista de velocidades


Este resultado es el anlogo relativista al teorema de Galileo de adicin de
r r r
velocidades, en la forma v = v + V . Para ello, imaginamos el movimiento uniforme de
un mvil cuyas coordenadas espacio-temporales sean ( x, t ) medidas por un observador

O en reposo, y ( x, t ) medidas por un observador O en movimiento uniforme con


velocidad V. Por tanto, la velocidad (uniforme) del mvil respecto a ambos
observadores ser
x
t
x
v=
t
v =

Ya que ambos observadores tienen un movimiento relativo uniforme, las


coordenadas espacio-temporales medidas respecto a ellos estn relacionadas por la
transformacin de Lorentz. Haciendo uso de ella, podemos obtener el resultado
x ( x Vt ) x Vt
v = =
=
t
Vx t Vx
t 2
c2
c
Dividendo numerador y denominador por t, e introduciendo la velocidad del mvil
respecto al observador en reposo, obtenemos el teorema de adicin relativista de
velocidades
v V
v =
Vv
1 2
c
que coincide con el resultado no relativista si las velocidades son mucho menores que la
velocidad de la luz.

7. Sucesos propios e impropios


Un suceso propio es el suceso relativista cuyas coordenadas espacio-temporales
para dos observadores en movimiento relativo uniforme estn relacionadas mediante la
transformacin de Lorentz. Son los sucesos para los cuales no han variado las
condiciones externas desde su comienzo hasta su finalizacin. En particular, son
sucesos propios todos los sucesos instantneos, que duran un solo instante de tiempo
(salida de una nave, emisin de una seal).
Un suceso impropio es el suceso relativista para el cual han variado las
condiciones externas desde su comienzo hasta su finalizacin. Por ejemplo, la emisin
desde la Tierra de una seal a una nave en movimiento y su recepcin por la nave. La
nave no se encuentra en la misma posicin en el instante de emisin que en el instante

de recepcin. Esto implica que las condiciones externas han cambiado para la seal,
desde su envo hasta su recepcin. Para relacionar las coordenadas espacio-temporales
respecto a observadores en movimiento relativo de un suceso impropio no podemos
hacer uso de la transformacin de Lorentz. Se debe resolver el problema aplicando la
proporcionalidad de intervalos de tiempo y la constancia de la velocidad de la luz.

Problemas resueltos
4.11 Una antena emite ondas de radio con intervalos de tiempo T1 . Los impulsos
son reflejados por un automvil, y regresan a la antena con intervalos de tiempo
T2 . Calcular la velocidad del automvil, suponiendo que es constante.
Nos fijamos en un determinada onda de radio que se emite entre los instantes t = 0
y t = T1 . Si el primer frente de onda se refleja en el automvil en el instante t cuando
ste se encuentra a una distancia x de la antena, y el ltimo frente de onda se refleja en
el instante t cuando el automvil se encuentra a una distancia x de la antena, entonces
la velocidad del automvil, supuesta constante, debe ser igual a la distancia recorrida
entre las dos reflexiones, dividida por el tiempo transcurrido entre las dos reflexiones
x x
V=
t t
Segn el principio de constancia de la velocidad de la luz, la onda se propaga con
velocidad constante c. Entonces, la distancia de la antena al automvil debe ser igual a
la distancia recorrida por la onda desde su emisin hasta su reflexin. Obtenemos as la
relacin para el primer frente de onda
x = x0 + ct
y anlogamente para el ltimo frente de onda
x = x0 + c ( t T1 )
siendo x0 la posicin del automvil en el instante inicial respecto al origen de
coordenadas localizado en la antena.
Adems, ya que el movimiento de la onda es uniforme, debe tardar el mismo tiempo
en llegar al automvil que en volver a la antena despus de la reflexin. Deducimos as
el recorrido del primer frente de onda: se emite en t = 0 , se refleja en el tiempo t y
regresa a la antena en el tiempo 2t. Anlogamente, el ltimo frente de onda se emite en
t = T1 , se refleja en el tiempo t y regresa a la antena en el tiempo T1 + 2 ( t ' T1 ) ,
despus de la reflexin. Con esto, el perodo T2 con que recibe la antena la onda
reflejada satisface
T2 = T1 + 2 ( t T1 ) 2t

con lo cual la diferencia de los tiempos de reflexin vale


T +T
t t = 1 2
2
y de aqu, la distancia recorrida por el automvil entre las reflexiones del primer y
ltimo frente de onda, es
T T
x x = c (t T1 ) ct = c ( t t) cT1 = c 2 1
2
Por tanto, la velocidad del automvil es
x
T T
V=
=c 2 1
t
T2 + T1
Si el automvil se aleja de la antena, V > 0 , entonces T2 > T1 , y la seal se retrasa. Si el
automvil se acerca a la antena, V < 0 , entonces T2 < T1 , y la seal se adelanta.

4.7

Un reloj se mueve con velocidad V, respecto de un reloj en reposo. Se


sincronizan ambos relojes en el instante inicial t=0, cuando el reloj en movimiento
pasa por delante del reloj en reposo. En el instante t1 se emite una seal desde el

reloj en reposo, que llega al reloj en movimiento en un tiempo propio . Hallar la


relacin entre t1 y , y el instante t2 en el que la onda reflejada por el reloj en
movimiento llega al reloj en reposo.
La onda se enva en el instante t1 , se refleja en el instante t (instante en el reloj en
movimiento), y llega de nuevo al reloj en reposo en el instante t2 . Como el movimiento
entre los relojes es uniforme, existe una proporcionalidad entre los intervalos de tiempo
para los dos relojes. Y ya que los relojes estn sincronizados en el instante inicial, la
proporcionalidad se establece entre los distintos instantes de tiempo. Se satisface
= Kt1

t2 = K
con lo cual, la constante de proporcionalidad K resulta ser
t
K= 2
t1
Estudiamos ahora la relacin entre el tiempo de emisin t1 , el tiempo de reflexin t,
y el tiempo de recepcin t2 . En virtud del carcter uniforme del movimiento relativo, y
de la constancia de la velocidad de la luz, el tiempo de reflexin es el tiempo medio
entre la emisin y la recepcin
t +t
t= 1 2
2

Por otro lado, para que se produzca la reflexin en el instante determinado t, es


necesario que la distancia recorrida por la seal desde el reloj en reposo sea igual a la
distancia recorrida por el reloj en movimiento desde el instante inicial. Esto es
Vt = c (t t1 )
con lo cual
V
t1 = 1 t
c

De forma anloga, obtenemos la relacin entre el instante de reflexin y el instante de


recepcin
V
t2 = 1 + t
c

Una vez determinada la relacin entre los tiempos de emisin, reflexin y recepcin,
la constante de proporcionalidad K resulta ser
c +V
>1
c V
De aqu, obtenemos el valor desconocido del instante de recepcin en la forma
c +V
t2 = K 2 t1 =
t1
c V
K=

Si el reloj en movimiento se acercara al reloj en reposo, la constante de


proporcionalidad sera, con el cambio de V por V
K =

c V
1
=
c +V K

y la relacin de tiempos quedara


= K t1
t2 = K

4.8

Una estrella se aleja de la Tierra con velocidad V. Emite una radiacin con
longitud de onda 0 medida en su sistema de referencia. Calcular la longitud de
onda de la radiacin recibida en el sistema de referencia de la Tierra, y su
aproximacin en el caso no relativista V = c .

Sea T0 el perodo de la radiacin emitida, medido en el


estrella. El perodo de la radiacin que incide en la
proporcionalidad de intervalos de tiempo medidos en los
instante de la emisin del primer pulso de onda, medido en
la estrella, este pulso llega a la Tierra en el instante terrestre

sistema de referencia de la
Tierra se obtiene de la
dos sistemas. Si t1 es el
el sistema de referencia de

c +V
t1
c V
Si t2 es el instante de emisin del ltimo pulso de onda, medido en el sistema de
referencia de la estrella, este pulso llega a la Tierra en el instante terrestre
1 = Kt1 =

c +V
t2
c V
Por tanto, la relacin entre los perodos de la radiacin emitida por la estrella y recibida
en la Tierra es
c +V
T = 2 1 = K ( t2 t1 ) = KT0 =
T0
c V
2 = Kt2 =

Introduciendo la longitud de onda de la radiacin = cT , y en virtud que la


velocidad de la luz no depende del estado de movimiento del observador, obtenemos la
relacin entre las longitudes de onda
c +V
0
c V
Es el efecto Doppler relativista. Cuando la estrella se aleja de la Tierra, V > 0 , > 0 ,
la radiacin recibida en la Tierra sufre un corrimiento del espectro hacia el rojo
(longitudes de onda mayores). Si la estrella se acerca a la Tierra, V < 0 , < 0 , la
radiacin recibida en la Tierra sufre un corrimiento del espectro hacia el azul
(longitudes de onda menores).
=

Cuando la velocidad de la estrella es mucho menor que la velocidad de la luz,


podemos hacer la aproximacin
1/2

V
1 + c

V
1 c

: 1+

1 / 2

V
2c

:1 +

V
2c

con lo cual
2

c +V
V
V
=
0 : 1 + 0 : 1 + 0
c V
c
2c

y tenemos el efecto Doppler no relativista, estudiado en la primera parte de este


captulo.

4.9

Un reloj se mueve con velocidad V, respecto de un reloj en reposo. Se


sincronizan ambos relojes en el instante inicial t=0. Calcular el tiempo que indica
el reloj en movimiento, si el reloj en reposo indica un tiempo t.
Para relacionar los tiempos medidos en los dos sistemas de referencia podemos
utilizar cualquier suceso relativista, que nos permita utilizar el principio de
proporcionalidad de intervalos y la constancia de la velocidad de la luz. Supongamos
que enviamos una seal luminosa desde el reloj en reposo en el instante t1 hacia el reloj
en movimiento. La seal se ve reflejada en el instante t para el reloj en reposo, y en el
instante para el reloj en movimiento. La seal reflejada llega de nuevo al reloj en
reposo en el instante t2 . Por tanto, nuestro suceso relativista ser la llegada de la seal
al reloj en movimiento.
Como el reloj en movimiento se mueve con velocidad constante respecto del reloj
en reposo, debe existir una proporcionalidad entre los intervalos de tiempo medidos por
el reloj en movimiento y los intervalos de tiempo medidos por el reloj en reposo. Es
decir, si el reloj en reposo enva una seal de perodo T, el reloj en movimiento recibe
la seal con perodo KT, medido en su sistema de referencia. Aplicamos este concepto a
la emisin de la onda en el instante t1 . Por tanto, para el reloj en reposo la onda se
emiti en el intervalo de tiempo entre t = 0 y t = t1 . Por la proporcionalidad de
intervalos, la seal debe recibirse por el reloj en movimiento en el intervalo de tiempo
entre = 0 y = Kt1 . Como los relojes estn sincronizados en el tiempo inicial, la
proporcionalidad de intervalos equivale a la proporcionalidad entre los instantes de
emisin por parte del reloj en reposo y recepcin por parte del reloj en movimiento. Es
decir, la seal se emite por el reloj en reposo en el instante t1 , y se recibe por el reloj en
movimiento en su instante = Kt1 . De forma anloga, la seal se refleja en el reloj en
movimiento en el instante , y se recibe de vuelta en el reloj en reposo en su instante
t2 = K . Por tanto, la relacin entre los tiempos de emisin, reflexin y recepcin es
= Kt1

t2 = K
con lo cual
= t1t2
Este ltimo resultado puede compararse con el tiempo de la reflexin medido por el
reloj en reposo
t +t
t= 1 2
2

Como ya vimos en el problema 4.7, los tiempos t1 , t2 en funcin del tiempo t


medido por el reloj en reposo son

V
t1 = 1 t
c

V
t2 = 1 + t
c

Entonces, la relacin entre los tiempos medidos por ambos relojes es

V2
t
c2
e introduciendo el factor , obtenemos el resultado final
1
= t <t

Hemos comprobado que el tiempo medido por el reloj en movimiento es menor que el
tiempo medido por el reloj en reposo. Existe una dilatacin del tiempo para un
observador en movimiento, respecto a un observador en reposo.
= 1

4.10 Una barra de longitud L0 en su sistema de referencia, se mueve con


velocidad V en la direccin de su eje, respecto a un reloj en reposo. Cuando su
extremo trasero A pasa por el reloj en reposo, ste emite una seal luminosa hacia
un espejo fijado en el extremo delantero B. La seal se refleja, recorre de nuevo la
barra y vuelve al reloj en reposo. Calcular el instante T de recepcin de la onda
luminosa segn el reloj en reposo, y de aqu obtener la longitud de la barra en el
sistema en reposo.
En el sistema de referencia fijado a la barra, la seal luminosa tarda un tiempo en
regresar al punto de partida A, dado por
2L
= 0
c
De la proporcionalidad entre los intervalos de tiempo medidos en ambos sistemas, si
parte del punto A en el instante , el instante de recepcin en el sistema en reposo es
T = K =

c + V 2L0
cV c

Sea L la longitud de la barra medida en el sistema en reposo. La distancia d


recorrida por la seal luminosa en el sistema en reposo es
d = cT
y debe ser igual al doble de la longitud de la barra, 2L, ms la distancia desde el
extremo A al reloj en reposo VT, en el momento T. Por tanto,
d = 2 L + VT
De estas dos ecuaciones, despejamos la longi tud de la barra en el sistema en reposo
1
L = ( c V )T
2

Introduciendo el valor de T y el factor obtenemos


1
2L c + V 1
= L < L0
(c V ) 0
2
c c V 0
La longitud de un cuerpo en movimiento medida en un sistema en reposo es menor que
la longitud real, medida en el sistema de referencia ligado al cuerpo. Existe una
contraccin de la longitud debida al movimiento de un cuerpo.
L=

4.11 La luz de una seal procedente del centro de una barra en reposo de
longitud L0 , alcanza sus extremos simultneamente. Calcular el tiempo, medido en
un reloj en reposo, si la barra se mueve con velocidad V segn su eje.
Sea L la longitud de la barra en el sistema en reposo. La distancia entre la seal
luminosa emitida y el extremo izquierdo de la barra disminuye una distancia c + V en
cada segundo. El tiempo que tarda en llegar al extremo izquierdo es
L
tiz =
2 (c + V )
desde el momento del destello. La distancia entre la seal emitida y el extremo derecho
de la barra, disminuye c V en cada segundo. El tiempo que tarda en llegar al extremo
derecho es
L
td =
2( c V )
El tiempo de retraso es (la seal alcanza el extremo izquierdo antes)
L 1
1
LV
t = td tiz =

= 2

2 c V c +V c V 2
Debido a la contraccin de la longitud, para el sistema en movimiento, la longitud
de la barra es
1
L = L0

con lo cual, el retraso temporal en la recepcin de la onda luminosa es


1 L0V
t =
0
c2 V 2
La conclusin es que la simultaneidad de sucesos tiene carcter relativo, y depende del
sistema de referencia.
4.12 Dos astronaves viajan con velocidad relativa V. Pasado un tiempo t desde el
encuentro de las dos naves, una de ellas A descubre que un asteroide se encuentra
a una distancia x. Calcular las coordenadas t' y x' del asteroide, medidas por la
otra nave B.

Para detectar el asteroide, ambas naves deben emitir seales luminosas que se
reflejen en el asteroide y regresen a cada nave. Sea t1 el tiempo de la emisin por la
nave A, t el tiempo de reflexin por el asteroide y x la distancia entre la nave A y el
asteroide en el momento de la reflexin. Sea t1 el tiempo de recepcin por la nave B de
la seal enviada por la nave A en el instante t1 , t el tiempo de reflexin de la seal por
el asteroide segn la nave B y x la distancia entre la nave B y el asteroide en el
momento de la reflexin. Se cumple
x = c ( t1 t )
para la nave B, y

x = c ( t1 t )

para la nave A. Por la proporcionalidad de los tiempos medidos por A y por B,


obtenemos la relacin entre los instantes de emisin por la nave A y de recepcin por la
nave B
t1 = Kt1
con lo cual
t1 =

c +V
c +V
t1 =
c V
c V

x
t c

Despus de reflejarse en el asteroide, la seal vuelve a cada nave en los tiempos


x
t2 = t +
c
x
t2 = t +
c
Ahora se satisface la relacin inversa de tiempos
t2 = Kt2
con lo cual
t2 =

c V
c V
x
t2 =
t +
c +V
c +V
c

Ya que la velocidad de la luz es constante en todo el proceso, el tiempo y posicin


del asteroide segn la nave A satisfacen
t + t
t = 1 2
2
t t
x = c 2 1
2
De aqu obtenemos la ley de transformacin de Lorentz, para el paso de coordenadas
de espacio-tiempo entre dos sistemas inerciales que se mueven con velocidad relativa
V. Para la variable temporal escribimos

Vx
t = t 2
c

donde hemos introducido el factor definido por


1
=
V2
1 2
c
y para la variable espacial
x = ( x Vt )

4.13 Una astronave A parte de la Tierra con una velocidad V en el instante que
sus relojes y los de la Tierra sealan el valor cero. Al cabo de un tiempo T segn
los relojes de la Tierra, parte una segunda nave B a una velocidad U. Suponiendo
que U es mayor que V, determinar el instante respecto de los relojes de la Tierra
en el que la segunda astronave alcanza a la primera. Determinar el instante en el
que B alcanza a A medido por la Tierra, el instante en que parti la nave B de la
Tierra, segn el reloj en A, el instante en que B alcanza a A, segn los relojes de A,
y la distancia a la que se encontraba A de la Tierra cuando parti B, en el sistema
de referencia de A. Deducir entonces la velocidad de B medida en A.
Las dos naves se mueven con velocidad uniforme respecto a la Tierra. Entonces,
respecto a la Tierra, la nave B alcanza a la nave A cuando las distancias recorridas por
ambas naves respecto de la Tierra sean iguales. El tiempo de vuelo de A ser t a una
velocidad V y el tiempo de vuelo de B ser t T a una velocidad U. Por tanto,
U (t T ) = Vt
de donde obtenemos
UT
t=
U V
Tomamos un sistema de referencia S fijo en la Tierra, y un sistema S' mvil, fijo en
A, que se mueve respecto de la Tierra con velocidad V. La transformacin de Lorentz
que liga las coordenadas espacio-temporales de ambos sistemas es
Vx
t = t 2
c

y
x = ( x Vt )
En t=0, los relojes de la Tierra y de A estn sincronizados. Entonces, si el reloj de la
Tierra marca un tiempo T, y la posicin de la Tierra en su sistema de referencia es x=0,
el reloj en A marca el tiempo

V0

= T 2 = T
c

En el instante en que B alcanza a A, los relojes de la Tierra marcan el tiempo t, a una


distancia de la Tierra Vt. Este suceso se produce respecto a A, en el instante dado por
UT V 2
VVt
1 = t 2 =
1 2
U V
c
c

y los relojes de A marcan el tiempo (de nuevo x=0, en la Tierra),


V2
c2
1 = t1 = UT
U V
1

En el sistema de referencia de A, la distancia x' a la Tierra, cuando el reloj de la


Tierra marca el tiempo t y el espacio x=0, est dada por la transformacin de Lorentz
x = VT
Por tanto, cuando sale B, (el tiempo t es T), la nave A dista de la Tierra una distancia D
(en mdulo) medida en A,
D = VT
De los dos resultados anteriores, vemos que respecto al sistema de referencia de A,
la nave B tarda un tiempo 1 en alcanzarla, y debe recorrer una distancia D para
ello. Por tanto, deducimos que la velocidad (uniforme) de B, medida por A es
D
U V
W=
=
1 1 VU
c2
que coincide con la frmula relativista de adicin de velocidades.

4.14 Dos naves viajan con velocidades opuestas sobre una estacin espacial. Son
testigos de dos acontecimientos A y B, que conforme a las observaciones de la nave
1, se producen durante el tiempo t, y conforme a las observaciones de la nave 2,
durante el tiempo t pero en sentido inverso (primero B luego A). Calcular el
tiempo y el lugar de los sucesos, segn el sistema de referencia de la estacin
espacial y comprobar si A y B tienen relacin de causalidad.
Supongamos que en el sistema de referencia de la estacin espacial, el suceso B
tuvo lugar a una distancia x0 del suceso A, despus de un tiempo t0 . Para ello,
utilizando la transformacin de Lorentz entre la estacin espacial y las dos naves, se
debe cumplir

Vx

nave 1: t = t0 20
c

Vx

nave 2: t = t0 + 20
c

De las dos ecuaciones anteriores despejamos el tiempo medido en el sistema de


referencia de la estacin espacial
t0 = 0
y la posicin del suceso B en este mismo sistema de referencia
ct 2
x0 =
V
En el sistema de referencia de la estacin espacial, los dos sucesos ocurren
simultneamente en distintos puntos del espacio. No pueden por tanto tener una relacin
de causa-efecto. Son sucesos independientes.

4.15 Una astronave de longitud L0 en su sistema de referencia, parte de la


Tierra con velocidad V. Ms tarde, se emite tras ella una seal luminosa que llega
a la cola del cohete en el instante , segn los relojes de la astronave y de la Tierra.
Determinar cundo llega la seal a la cabeza del cohete, segn los relojes del
mismo y segn los relojes de la Tierra. La seal se refleja en la cabeza del cohete y
se dirige a la cola del cohete. Determinar cundo alcanza la cola del cohete segn
los relojes de la nave y de la Tierra.
Respecto al sistema ligado al cohete, la distancia que debe recorrer para llegar a la
cabeza es L0 , mantenindose el cohete en reposo respecto a su propio sistema de
L
coordenadas. El tiempo necesario es = 0 . Adems, despus de reflejarse, el tiempo
c
que tarda la seal en volver a la cola es el mismo .
Respecto al sistema de la Tierra, el cohete tiene una longitud contrada L = L0 / .
Cuando la seal se dirige hacia la cabeza del cohete su velocidad es c y la cabeza se
aleja de la seal con velocidad V. Por tanto, la velocidad relativa entre la seal y la
cabeza del cohete es c V . El tiempo que tarda en llegar a la cabeza, medido por la
Tierra, ser
L
L0
t1 =
=
c V (c V )
y de forma anloga para el viaje de la cabeza a la cola, la velocidad relativa es c + V , y
el tiempo empleado es
L
L0
t2 =
=
c + V ( c +V )

Tema 4. Relatividad especial


Tercera parte: Dinmica relativista
1. Masa relativista

La inercia de un cuerpo es consecuencia de su resistencia al cambio en su estado de


movimiento, y se identifica usualmente con la masa. Clsicamente su valor depende de
la cantidad de materia presente en el cuerpo.

Sin embargo, cuando un cuerpo acelera uniformemente en su propio sistema de


referencia, respecto a un observador exterior tendr una aceleracin no uniforme sino
decreciente que evite que el cuerpo pueda alcanzar la velocidad de la luz. El observador
exterior concluir que la resistencia al cambio de movimiento crece al aumentar la
velocidad. Por tanto, la relatividad lleva a la conclusin de que la inercia, o masa, crece
con la velocidad, y depende tanto de la cantidad de materia presente como de su estado
de movimiento. La masa de un cuerpo medida en su propio sistema de referencia en
reposo se denomina masa en reposo, y su masa total o masa relativista cuando se
mueve siempre es mayor que la masa en reposo. En particular, puede deducirse la
expresin general
m0
m (V ) =
= m0
V2
1 2
c
siendo m0 la masa en reposo.

En la fsica clsica, el momento lineal se define como el producto de la masa y la


velocidad. En relatividad, tal definicin se mantiene, pero la masa debe definirse como
masa relativista.

2. Leyes de conservacin
En la fsica clsica, las cantidades dinmicas ms importantes son las que se
conservan durante las interacciones. Por ejemplo, cuando dos cuerpos colisionan entre
s la masa total, energa y momento despus de la colisin son iguales a sus valores
antes de la colisin. El principio de relatividad demanda que las leyes de la fsica sean
las mismas en todos los sistemas de referencia inerciales.
La propiedad ms importante del momento lineal en la mecnica clsica es su
conservacin en sistemas cerrados. Este resultado se deriva de las leyes de Newton en
ausencia de fuerzas exteriores. En la mecnica relativista hemos visto que la masa total

de un cuerpo en movimiento siempre es mayor que su masa en reposo. Definimos por


tanto el momento relativista en la forma
p = m ( V )V = m0V
e imponemos como principio la conservacin del momento relativista de un sistema
cerrado en todos los sistemas inerciales de referencia.
El trabajo realizado sobre un cuerpo es igual al producto de la fuerza aplicada por la
distancia recorrida en la direccin de la fuerza. Si la inercia de un cuerpo aumenta con
la velocidad esto implica que la fuerza que aplicamos produce una aceleracin
decreciente. Ya que el trabajo realizado no vara significativamente la velocidad del
cuerpo, se transforma en una energa interna que almacena el cuerpo. La conclusin es
que la energa y la masa se deben reinterpretar de forma conjunta en relatividad. En
particular, se satisface la relacin de Einstein entre la energa total de un cuerpo y su
masa relativista
E = m (V ) c 2 = m0 c2
y esta es la expresin que tomaremos para definir la energa relativista de un cuerpo.
Adems imponemos como principio la conservacin de la energa relativista en un
sistema cerrado en todos los sistemas inerciales de referencia.
La relacin entre el momento lineal relativista de un cuerpo y su energa total que
generaliza la expresin no relativista es
E = c2 p 2 + m02 c4
que se queda reducida en el caso de una masa en reposo nula, en el caso de los fotones,
E = cp

En la fsica clsica la energa cintica es una cantidad significativa que tiene una
frmula simple. La frmula newtoniana es slo una aproximacin a la energa cintica
relativista. En relatividad, la energa cintica se define como la diferencia entre la
energa total de un cuerpo y su energa en reposo, en ausencia de movimiento. Por
tanto, la energa cintica es la energa adquirida por el cuerpo debido a su movimiento.
Tiene la expresin
K = E E 0 = ( 1) m0c 2
Por ltimo, las unidades de medida en relatividad son diferentes que en la fsica
clsica, principalmente debido al cambio de magnitud de las cantidades fsicas. La
energa se mide en MeV y el momento lineal en MeV / c

Problemas Resueltos
4.16 Una partcula de masa en reposo m, es alcanzada por un fotn de energa Q,
que queda absorbido. Calcular la velocidad final del cuerpo compuesto, si la
partcula inicialmente estaba en reposo.
Por conservacin de la energa relativista
E = mc 2 + Q
siendo E la energa del cuerpo compuesto, despus de la absorcin del fotn. El fotn
no tiene masa en reposo, por lo que la expresin de su energa relativista es
Q = cp
siendo p el momento lineal del fotn.
Por conservacin del momento lineal, p ser el momento del cuerpo compuesto. Por
tanto, el cuerpo compuesto tiene momento lineal p y energa E. La velocidad viene
dada por la expresin
c2 p
V=
E
con lo cual
Q
V=
c
Q + mc 2

4.17 Un fotn de energa Q choca con un electrn en reposo, el cual retrocede


segn la direccin dada por el ngulo . El fotn se dispersa segn la direccin
dada por el ngulo . Calcular la energa final del fotn y su longitud de onda.

fotn

fotn

Por conservacin del momento lineal, en la direccin del fotn incidente


Q Q
= cos + p cos
c
c
siendo Q la energa del fotn incidente, Q' la energa del fotn emergente, y p el
momento del electrn despus de la colisin. La conservacin del momento en la
direccin perpendicular a la direccin del fotn incidente se escribe

Q
sen p sen
c

0=

La conservacin de la energa relativista es


Q + mc 2 = Q + E
siendo m la masa en reposo del electrn y E su energa final, que se relaciona con su
momento final p a travs de

E = c 2 p 2 + mc 2

Primero eliminamos E. Escribimos la ecuacin de conservacin de la energa en la


forma

( Q + mc

= c 2 p 2 + m 2c 4

Despus eliminamos el ngulo . Para ello, escribimos las ecuaciones de conservacin


del momento en la forma
cp cos = Q Q cos
cp sen = Q sen
elevamos al cuadrado y sumamos resultando
c 2 p2 = Q 2 +Q 2 2QQ cos
Por ltimo, eliminamos p. De la ltima ecuacin y de la conservacin de la energa,
obtenemos

( Q + mc

Desarrollando el lado izquierdo

= Q 2 + Q2 2QQ cos + m 2c 4

( Q Q ) mc2 = QQ cos

Despejando Q', la energa final del fotn, obtenemos


1 1
1
= +
(1 cos )
Q Q mc 2
De la relacin entre la longitud de onda del fotn y su energa
c
Q = h = h

obtenemos la relacin entre las longitudes de onda del fotn incidente y del fotn
emergente
h
= + 2 (1 cos )
mc
La colisin elstica entre un electrn y un fotn, tal como se ha desarrollado aqu, se
conoce como efecto Compton, y la longitud de onda caracterstica

h
mc 2
se conoce como la longitud de onda de Compton.
c =

4.18 Calcular la energa umbral del proceso


P1 + P2 P + N + +
en el que un protn P1 incide sobre otro protn en reposo P2 y se produce en la
colisin un protn P , un neutrn N y un pin + . Las masas en reposo son
m ( ) c 2 = 140 MeV

m ( P ) c 2 = m ( N ) c 2 = 938 MeV

La energa umbral es la energa cintica del protn incidente tal que las partculas
finales tienen energa mnima. Tomemos inicialmente el sistema de referencia en el que
el sistema de los dos protones iniciales tiene momento nulo. En este sistema P1 y P2
poseen un momento lineal del mismo mdulo y de signo contrario y la misma energa.
En este sistema, el proceso umbral corresponde a una velocidad nula de las partculas
finales P, N , + . Si V es la velocidad de los protones iniciales en este sistema, por
conservacin de la energa, se debe satisfacer
2m ( P ) c 2
= m ( P ) c 2 + m ( N ) c2 + m( ) c2
2
V
1 2
c
de donde obtenemos la velocidad V de los protones en el sistema de momento total
nulo, que satisface
1
1 m ( )
= 1+
2 m ( P)
V2
1 2
c
Para calcular la velocidad de P1 en el sistema del laboratorio, donde P2 posee una
velocidad nula, vemos que segn el teorema de adicin relativista de velocidades
U V
0 = V ( P2 ) =
UV
1+ 2
c
la velocidad U relativa entre ambos sistemas es igual a V. Con esto, la velocidad de P1
respecto al sistema del laboratorio es
V +V
2V
V ( P1 ) =
=
2
V
V2
1+ 2 1+ 2
c
c

La energa cintica umbral ser

Ku = E ( P1 ) m ( P ) c 2

resultando
1

V
P
(
)
1

Ku = m ( P ) c
1
1

c2

Desarrollando el radicando, obtenemos


2

V ( P1 )
c2

= 1

(c

4V 2 c 2
2

+V2

(c
=
) (c
2

V 2

+V 2

)
)

2
2

con lo cual,
2

V
P
(
)
1
1

c2

2
2
2
1 = c + V 1 = 2V

c2 V 2
c2 V 2

y
2V 2
Ku = m ( P ) c 2
c V 2
2

Introduciendo el valor de V obtenido anteriormente,

m ( ) ( m ( ) + 4 m ( P ))
2m ( P )
V2
=
1

2
2 m ( P ) + m ( )
c2

( 2m ( P ) + m ( ) )
2

con lo cual,
m ( )
m ( )
2V 2
=
2 +

2
2
c V
m( P )
2 m ( P )
y la energa umbral queda definitivamente
m ( )
m ( )
m ( )
2
Ku = m ( P ) c 2
2 +
= 2 +
m ( ) c
m ( P)
2m ( P )
2 m ( P )

4.19 Una partcula de masa en reposo m, incide con energa E sobre otra
partcula idntica en reposo. Las dos partculas se separan con energas iguales.
Calcular el ngulo que forman sus direcciones con la direccin inicial.
Por conservacin del impulso y de la energa relativista, el ngulo formado con la
direccin inicial es el mismo para las dos partculas, si bien se desplazan por ambos
lados respecto de la direccin inicial. De aqu,
p = 2 p cos
E + mc 2 = 2 E

siendo E' y p' la energa y el impulso de una de las partculas finales. Con la definicin
de energa relativista
E 2 = m 2c 4 + c 2 p 2
E2 = m2c 4 + c 2 p 2
obtenemos, a partir de las ecuaciones de conservacin
2
E 2 = m 2c 4 + c 2 ( 2 p cos )
junto con

c 2 ( 2 p cos ) = 4 c 2 p 2 cos2 = cos2 4 E2 4m 2c 4


2

El ltimo factor lo desarrollamos de la siguiente forma


4 E 2 4m 2c 4 = 2 E + 2 mc 2 2 E 2mc 2 = E + 3mc 2

)(

con lo cual el ngulo de dispersin satisface

) (

E 2 = m 2c 4 + cos 2 E + 3mc 2

)( E mc )
2

)( E mc )
2

Simplificando obtenemos
cos2 =

E + mc 2
E + 3mc 2

Problemas Propuestos
4.20 Una partcula en reposo, con masa en reposo m, emite un fotn de energa
Q. Calcular la masa en reposo de la partcula despus de la emisin.
Q

Solucin: M = m m 2 2
c

4.21 Una partcula con masa en reposo m y energa cintica 2mc 2 , choca con una
partcula en reposo de masa 2m, y se adhiere a ella. Calcular la masa en reposo de
la partcula compuesta.
Solucin: M = 17 m

4.22 Calcular la energa cintica umbral del proceso


P1 + P2 P1 + P2 + P3 + P
en el que un protn P1 incide sobre otro protn en reposo P2 y se produce un
protn P3 y un antiprotn P . Las masas en reposo son

m ( P ) c2 = m ( P ) c 2 = 938 MeV

Solucin: Ku = 6m ( P ) c2

4.23 Una partcula de masa en reposo m, se desintegra en dos partculas con


masas en reposo m1 y m2. Calcular las energas de las partculas emitidas.
m2 + m12 m22
E1 =
2m
Solucin:
2
m + m22 m12
E2 =
2m

4.24 El mesn K0 se desintegra formando piones cargados con masas en reposo


idnticas, segn el proceso K 0 + + . Las masas en reposo son
m K 0 c 2 = 497.7 MeV

( )

m ( ) c 2 = 139.6 MeV
Determinar la velocidad de los piones resultantes en la desintegracin de un mesn
en reposo.

Solucin: V = 1 4

m ( )

( )

m K0

c ; 0.83 c

Problemas Propuestos
4.25 Un cohete de longitud L0 , que marcha con velocidad v = 0.6 c , lleva un
receptor en su punta. Se emite un pulso de radio desde una estacin espacial en
reposo en el momento en que pasa frente a ella la cola del cohete. Calcular la
distancia a la que se encuentra la punta del cohete de la estacin espacial en el
instante en el la punta del cohete recibe la seal de radio. En trminos de la
estacin espacial, determinar el intervalo de tiempo entre la llegada de la seal y
su emisin desde la estacin. Determinar ese mismo intervalo de tiempo de
acuerdo con las medidas en el sistema en reposo del cohete.
2L
4L
Solucin: d = 2 L0 , t = 0 , = 0
c
c
4.26 Dos naves espaciales de longitud L0 , se cruzan entre s. Los instrumentos de
medida de la nave A sealan que la parte delantera de la nave B invierte el tiempo
t0 en recorrer toda la longitud de A. Calcular la velocidad relativa de ambas
naves. En el extremo frontal de B se sita un reloj. Determinar el intervalo de
tiempo medido por este reloj al pasar del extremo frontal de A al extremo
posterior de A.
Solucin: v =

L0
L
v2
,t = 0 1 2
v
t0
c

4.27 A las doce del medioda un cohete espacial pasa frente a la Tierra con una
velocidad de 0,8 c. Los observadores de la nave y los de la Tierra estn de acuerdo
en que efectivamente es medioda.
a) A las 12:30 pm, segn un reloj situado en la nave, sta pasa por delante
de una estacin interplanetaria que se encuentra fija en re lacin a la
Tierra y cuyos relojes sealan el tiempo de la Tierra. Qu hora marca
el reloj de la estacin?
b) A qu distancia de la Tierra (en coordenadas terrestres) se encuentra
la estacin?
c) A las 12:30 pm, hora de la nave, se establece comunicacin con la
Tierra desde la nave. Cundo en tiempo de la Tierra recibe sta la
seal enviada?
d) La estacin terrestre contesta inmediatamente. Cundo se recibir la
respuesta (hora de la nave?
Solucin
a) 12:10 pm
b) 1,44 108 km
c) 12:18 pm
d) 12:54 pm

4.28
Un mesn K0 en reposo se desintegra dando lugar a un mesn + y a un
mesn -, cada uno de los cuales posee una velocidad de 0,85 c. Al desintegrarse un
mesn K0 que marcha a una velocidad de 0,9 c, determinar la velocidad mxima y
mnima que puede alcanzar uno de los me sones .
Solucin: vmax=0,99c, vmin=0,21c
4.29 Existe un servicio de naves espaciales desde la Tierra a Marte. Cada nave
est equipada con dos luces idnticas, una delante y otra detrs. Las naves
marchan normalmente a una velocidad v0 , relativa a la Tierra, de forma que el
faro delantero de la nave espacial que se dirige hacia la Tierra aparenta ser verde
( =5000 ?) y la luz de cola de una nave que despega aparenta ser roja ( =6000 ?).
Determinar el valor de la velocidad. Una nave espacial acelera para adelantar a
otra nave que marcha por delante de ella. Determinar la velocidad que deber
adquirir la primera nave (relativa a la Tierra) para que la luz de cola de la nave
que est por delante de ella parezca un faro delantero de luz verde.
2v c 2
Solucin: v0 =c/11, v tierra = 2 0 2 = 0,18 c
c + v0
4.30
Demostrar que los procesos que se sealan a continuacin son imposibles
desde un punto de vista dinmico:
a) Un fotn choca con un electrn en reposo y entrega toda su energa al
electrn.
b) Un fotn situado en el espacio libre se transforma en un electrn y un
positrn.
c) Un positrn rpido y un electrn en reposo se destruyen mutuamente
dando lugar a un solo fotn.
4.31 Un fotn de elevada energa choca y es dispersado por un protn que se
encuentra inicialmente en reposo y puede retroceder libremente. El protn
retrocede formando un ngulo de 30 con una energa cintica de 100 MeV.
Calcular la energa del fotn incidente y la direccin y energa del fotn
dispersado. La energa en reposo del protn es de 938 MeV.
Solucin: Q = 339 MeV , Q = 229 MeV , = 104
4.32 Supongamos que un cierto acelerador puede suministrar a los protones una
energa cintica de 200000 MeV. La masa en reposo de un protn es de 938 MeV.
Calcular la mxima masa en reposo de una partcula X que pueda producirse por
el choque de uno de estos protones de alta energa con un protn en reposo segn
el proceso siguiente
p+ p p+ p +X
2
Solucin: M X c = 17539 MeV

Tema 5. Oscilaciones de sistemas con un


grado de libertad
Introduccin
Estudiamos las propiedades que debe tener un sistema para que se produzcan
oscilaciones en l. La primera propiedad es la existencia de un punto de equilibrio
estable. Slo cuando el sistema es desplazado de esta posicin de equilibrio, exhibe
caractersticas oscilatorias. Este desplazamiento genera en el sistema una fuerza
restauradora, y aparecen vibraciones alrededor del punto de equilibrio.
La existencia de tal posicin de equilibrio nos da una referencia respecto a la
cual podemos medir el desplazamiento del sistema cuando se encuentra en
movimiento. De esta forma, las ecuaciones del movimiento y la energa del sistema
se expresan de la forma ms sencilla.
Tenemos una segunda propiedad esencial. Cuando un sistema vibratorio se
desplaza de la posicin de equilibrio, la fuerza restauradora le obliga a moverse de
nuevo hacia la posicin de equilibrio. La inercia del sistema (masa, momento de
inercia) hace que el sistema sobrepase el punto de equilibrio, y as realice la segunda
parte de la oscilacin.
Por otro lado, la friccin puede ser responsable de la ausencia de movimientos
oscilatorios. Por tanto, para la existencia de movimientos oscilatorios, la friccin no
debe ser muy grande. Dicho de otra forma, la fraccin de energa perdida por
rozamiento no debe ser muy grande.

Clasificacin de las oscilaciones


Respecto a la duracin en el tiempo,
Oscilacin Transitoria: Desde el momento inicial t = 0 , hasta que se alcanza la
forma A cos ( t + )

Oscilacin Permanente: Si se ha alcanzado la forma A cos ( t + ) y sta perdura


en el tiempo.
Respecto a la existencia de una fuerza exterior,
Oscilacin Forzada: Hay una fuerza exterior sobre el sistema.
Oscilacin Libre: No acta ninguna fuerza exterior sobre el sistema.
Respecto al efecto del rozamiento,
Oscilacin Amortiguada: El sistema sufre una fuerza de rozamiento, en direccin
contraria a su movimiento.
Oscilacin no Amortiguada: La fuerza de rozamiento es despreciable.

Respecto al perodo del movimiento,


Oscilacin Peridica: El sistema repite su estado de movimiento (misma posicin y
velocidad) cada intervalo de tiempo T .
Oscilacin no Peridica: No se produce esta repeticin temporal del estado del
sistema. ( T = 0 )
Respecto a la dependencia temporal de las variables del sistema,
Oscilacin Armnica: La dependencia temporal de las variables del sistema
(posicin y velocidad) es de la forma sin ( t + ) , cos ( t + ) .
Oscilacin no Armnica: La dependencia temporal no es armnica.

Primera parte: Oscilaciones libres


1. Movimiento armnico simple (MAS)
La ecuacin del movimiento es de la forma
d 2s
+ 2s = 0
2
dt
La variable s representa una coordenada del sistema en movimiento. Puede ser la
coordenada x (elongacin de un muelle), un ngulo (ngulo de giro de un
pndulo). Representa el desplazamiento del sistema respecto de su posicin de
equilibrio s = 0 , ( x = 0, = 0 ) .
La solucin ms general de la ecuacin del movimiento es
s = A cos ( t + )
donde definimos las cantidades:
q

A amplitud del movimiento. El sistema se mueve entre los lmites


A s A
frecuencia angular del movimiento, medida en rad/s. Tambin se
llama frecuencia natural del sistema.

t + fase instantnea del sistema

fase inicial en t = 0

Las variables A, dependen del estado inicial del sistema (posicin y velocidad en
t = 0 ). La variable depende de las caractersticas fsicas del sistema (masa,
momento de inercia, constante del muelle, gravedad,..)

Otras variables que podemos definir en un MAS son:


q

T perodo del movimiento. Tiempo que transcurre entre dos estados


idnticos de movimiento del sistema. El sistema tiene la misma posicin s
2
y la misma velocidad ds . Se cumple T =
dt

frecuencia del movimiento. El sistema realiza un ciclo de su


movimiento en un tiempo T . La frecuencia es el nmero de ciclos que
1
realiza el sistema en la unidad de tiempo. Por tanto, = =
T 2

De la solucin general, obtenemos la velocidad y aceleracin del sistema


ds
v=
= A sin ( t + )
dt
d 2s
a = 2 = A 2 cos ( t + ) = 2 s
dt
Vemos por tanto, que en un MAS la fuerza restauradora
F = ma = m 2 s
est dirigida hacia el punto de equilibrio s = 0 , ya que F es positiva cuando s < 0 , y
es negativa cuando s > 0 . En general, la fuerza restauradora tendr la expresin
F = ks , donde k ser la constante de recuperacin del sistema.

2. Energa en un MAS
La energa cintica del sistema es
2

1 ds 1
Ec = m = mA2 2 sin 2 ( t + )
2 dt
2
Es la energa que posee el sistema debido a su movimiento.

Como hemos visto, la fuerza restauradora del sistema es


F = ma = m 2 s
De aqu, la energa potencial del sistema, debida a su posicin, resulta ser

1
sds = m 2 s 2
2
0
0
donde hemos tomado como referencia el cero de la energa potencial en el punto de
equilibrio s = 0 .
Introduciendo la dependencia temporal de s, obtenemos
1
E p = mA2 2 cos2 ( t + )
2
Ep =

Fds =m

La energa total del sistema es


1
1
E = Ec + E p = mA2 2 sin2 ( t + ) + mA2 2 cos 2 ( t + )
2
2
1
= mA2 2
2
El sistema es conservativo ya que la energa se conserva constante durante el
movimiento.

3. Determinacin de para un MAS


La frecuencia angular depende de las caractersticas fsicas del sistema,
como la masa, la constante de recuperacin, etc. Un mtodo directo para hallar
es hallar la ecuacin del movimiento e identificar 2 como el coeficiente de la
variable s .
Otro mtodo alternativo, para sistemas de un grado de libertad en los que sea
difcil determinar la ecuacin del movimiento, es el mtodo de Rayleigh, o mtodo
de las energas. Consta de dos partes importantes. La primera es la suposicin de que
el movimiento es armnico simple, con la posicin
s = A cos ( t + )
y la velocidad
ds
v=
= A sin ( t + )
dt
En segundo lugar, la suma de las energas cintica y potencial es una constante,
como hemos visto. Ya que el movimiento es oscilatorio, esto supone que en la
posicin extrema s = A , el sistema llega al reposo, y entonces toda la energa es
potencial. Al pasar por la posicin de equilibrio, la energa potencial es cero, y toda
la energa es cintica. La conservacin de la energa requiere que el cambio en la
energa cintica sea igual, y de signo contrario, al cambio en la energa potencial.
Por tanto, la energa potencial mxima (cuando s = A ) debe ser igual a la energa
cintica mxima (cuando s = 0 ). Entonces, si k es la constante recuperadora del
sistema, tenemos la igualdad
1 2 1
kA = m 2 A2
2
2
y de aqu, despejamos la frecuencia angular de vibracin del sistema
k
m
El mtodo de Rayleigh no slo da el resultado exacto cuando el sistema sea
conservativo, sino que puede servir tambin para obtener las frecuencias naturales
en forma aproximada para sistemas en los que no pueda ignorarse el calor y la
friccin (sistemas no conservativos).
En un sistema con un solo grado de libertad, la energa slo involucra dicha
coordenada. Si el sistema tiene varios grados de libertad, los mtodos de energa
todava son vlidos, y utilizan las matrices para ordenar la multiplicidad de trminos.
Por ltimo, destacar que el clculo de energas hecho aqu sirve como introduccin
=

para el mtodo generalizado de Lagrange, donde el llamado Lagrangiano es funcin


de las energas cintica y potencial, y la ecuacin de movimiento equivale a exigir
que dicho Lagrangiano haga mnima la integral Accin, funcin de las coordenadas
del sistema y del tiempo.
Un tercer mtodo, el mtodo funcional, es muy til en sistemas con dos
grados de libertad. Determina 2 utilizando la forma funcional
fuerza restauradora (en mdulo)
2 =
masa distancia
donde la distancia recorrida mide el desplazamiento respecto del punto de equilibrio
necesario para que la fuerza restauradora tenga el valor dado en el numerador. Como
ejemplo general,
F = ks
y
ks k
2 = =
ms m
que coincide con el resultado anterior.

4. Potencia desarrollada en un MAS


Un concepto muy importante en los movimientos oscilatorios es el de
potencia. La potencia es la razn de transferencia de energa al sistema, por unidad
de tiempo. Si el movimiento es peridico, como lo es el MAS, slo es til estudiar la
potencia media, promediada sobre un perodo de oscilacin. Nos da la transferencia
de energa sobre un perodo. Si sobre el sistema acta la fuerza F , la potencia
instantnea en el tiempo t , suministrada al sistema, es
Pins = F v
y la potencia media es, por definicin,
1
P=
T

F vdt

Si P > 0 , el sistema gana energa en cada ciclo, y si P < 0 el sistema cede energa
en cada ciclo.
Para el MAS,

s = A cos ( t + )
F = -ks
ds
v=
= A sen ( t + )
dt

con lo cual

1
P = k A2 sen ( t + ) cos ( t + ) dt = 0
T
0
ya que el valor medio en un perodo del producto de la funcin seno por la funcin
coseno es cero. Por tanto, no hay transferencia de energa, y la energa se conserva.

5. Sistemas sencillos con oscilaciones libres


I. MUELLE SIMPLE
k

equilibrio

La nica fuerza que acta sobre la masa m es la debida al muelle. Un muelle


simple es un sistema elstico que obedece a la ley de Hooke: la fuerza restauradora
es proporcional al desplazamiento respecto del punto de equilibrio. La coordenada
que define al sistema es x , el desplazamiento respecto al equilibrio, con lo cual
F = kx
2
La aceleracin de la masa m es a = d x

dt 2

, y la ley de Newton, F = ma , se

lee
d 2x
= kx
dt 2
que es la ecuacin de movimiento de un MAS
d 2x k
+ x=0
dt 2 m
con la frecuencia angular
m

k
m

La solucin de la ecuacin del movimiento es


x = A cos ( t + )
y el sistema es conservativo, la energa es constante y vale
1
1
E = kA2 = m 2 A2
2
2
siendo A la amplitud del movimiento.

II. PNDULO SIMPLE


O

L
T
mg

Fc

La masa m realiza una rotacin respecto al punto de suspensin, fijo en O. La


coordenada del sistema es , el ngulo respecto de la vertical. Las fuerzas que
actan sobre m son la gravedad en la vertical hacia abajo, la tensin T del hilo y la
fuerza centrfuga Fc debida a la rotacin, en la direccin radial desde O hacia
afuera. El momento de fuerzas respecto al punto O es
M = mgL sin TL sin0 + Fc L sin0
= mgL sin
siendo L la longitud de la cuerda que une el punto O con la masa m . El signo
menos en la ecuacin anterior indica que el momento de fuerzas tiende a disminuir
el ngulo de giro .

Si consideramos que el ngulo es pequeo, esto es,


( rad ) << 1
podemos utilizar la aproximacin
sin
con lo cual, el momento de fuerzas es proporcional al ngulo de giro
M = mgL
La ecuacin de movimiento se obtiene de la segunda ley de Newton para la rotacin
respecto a un punto fijo,
d 2
M =I 2
dt
siendo I el momento de inercia de m respecto a O,
I = mL2
Llegamos a la ecuacin de movimiento
d 2
mgL = mL2 2
dt
ecuacin de un MAS,
d 2 g
+ =0
dt 2 L
con frecuencia angular,
=

g
L

La solucin general es

= A cos ( t + )

siendo A la amplitud angular del giro, esto es, el ngulo vara entre los valores
A y A.
Es til reconsiderar la conservacin de la energa en este caso, ya que la
ecuacin de movimiento corresponde a una rotacin, y no a una traslacin como en
el caso general. La velocidad de giro de la masa m es
dx d ( L )
d
v=
=
=L
dt
dt
dt
siendo x la longitud sobre el arco de radio L . Con esto, la energa cintica se
escribe
2

1
1
d 1 2 2 2 2
Ec = mv 2 = mL2
= mL A sin ( t + )
2
2
dt 2
Tomando como cero de energa potencial el punto ms bajo de la trayectoria, la
energa potencial de la masa m es
E p = mgh

siendo h la altura desde el punto ms bajo,


h = L L cos
Cuando ( rad ) << 1 , la aproximacin vlida para cos es
1
cos 1 2
2
con lo cual, en esta aproximacin la altura h es
1
h = L (1 cos ) L 2
2
y la energa potencial queda
1
E p = mgL 2
2
Introduciendo la dependencia temporal del ngulo de giro ( t ) , la energa potencial
es
1
E p = mgLA2 cos2 ( t + )
2
Teniendo en cuenta el valor de la frecuencia angular del movimiento, llegamos a la
conservacin de la energa total de la masa m
1
1
E = Ec + E p = mgLA2 sin 2 ( t + ) + mgLA2 cos2 ( t + )
2
2
1
1
= mgLA2 = mL2 2 A2
2
2

III. PNDULO FSICO

cm

mg

O es el punto fijo de suspensin, respecto al cual gira el cuerpo de masa m .


es el ngulo de giro respecto a la vertical. El momento de la fuerza de la gravedad
respecto al punto O es
M = mga sin
siendo a la distancia de O al centro de masa del cuerpo. La ecuacin de movimiento
para la rotacin respecto al punto fijo O es
d 2
M = IO 2
dt
donde IO es el momento de inercia del cuerpo respecto del punto O. Puede
utilizarse, si es conveniente, el teorema de Steiner para calcular IO
IO = Icm + ma 2

donde I cm es el momento de inercia del cuerpo respecto de un eje que pasa por su
centro de masa y es perpendicular al plano que define el movimiento.
La ecuacin de movimiento es
d 2
mga sin = I O 2
dt
y con la aproximacin de ngulos pequeos, sin , obtenemos la ecuacin de un
MAS,
d 2 mga
+
=0
dt 2
IO
con frecuencia angular

mga
IO

La solucin general es, de nuevo


= A cos ( t + )
y la energa total de la masa m se conserva, con el valor
1
1
E = I O 2 A2 = mgaA2
2
2

IV. MAS BAJO POTENCIAL V (x ) GENERAL


Para que un potencial V ( x ) admita un MAS alrededor del punto x = x0 , debe
ocurrir necesariamente que
a) V ( x ) tiene un punto de equilibrio local en x = x0 . Esto es, x = x0 es
mnimo local de V ( x )

V ' ( x0 ) = 0

V '' ( x0 ) > 0

b) Los desplazamientos respecto al punto de equilibrio x = x0 deben ser


pequeos. Esto es, la distancia al punto de equilibrio x x0 es pequea
frente a ambos valores x , x0 .

V (x )
V' (x0) = 0
V (x0 )

V' '( x0) > 0

x0

La ecuacin de movimiento para un sistema en un potencial general V ( x ) se


sigue de la segunda ley de Newton, F = ma , donde F =
por el potencial, y a =

dV
es la fuerza generada
dx

d 2x
es la aceleracin del sistema. Por tanto, la ecuacin del
dt 2

movimiento es
dV
d 2x

=m 2
dx
dt

Si se cumplen las hiptesis a) y b), podemos desarrollar la funcin potencial


en serie de Taylor alrededor del punto de equilibrio, manteniendo slo el trmino de
orden dos en la diferencia x x0 , puesto que es pequea. Obtenemos, cerca del
punto de equilibrio,
1
2
V ( x ) V ( x0 ) + V ' ( x0 )( x x0 ) + V '' ( x0 )( x x0 )
2
1
2
= V ( x0 ) + V '' ( x0 )( x x0 )
2

V (x )

1
V ( x0 ) + V ' ' ( x0 )( x x0 )2
2

V ( x0 )

x0
y de aqu,

F = V '' ( x0 )( x x0 )

proporcional a la distancia al punto de equilibrio. Definiendo esa distancia como la


variable s , la fuerza restauradora es
F = ks
donde la constante recuperadora vale, en este caso,
k = V '' ( x0 )
Obtenemos as la ecuacin de movimiento de un MAS, para la variable s
d 2s
m 2 = ks
dt
con frecuencia angular
=

y solucin general
o mejor,

V '' ( x0 )
k
=
m
m

s = A cos ( t + )
x = x0 + A cos ( t + )

oscilacin de amplitud A, respecto al punto de equilibrio x = x0 .


Nota: El movimiento ser un MAS cuando la separacin del punto de equilibrio sea
pequea. La condicin matemtica que debe satisfacerse es que los trminos que no
hemos tenido en cuenta en el desarrollo de Taylor deben ser despreciables frente al
trmino cuadrtico en el desplazamiento ( x x0 ) . En particular, para el primer
trmino despreciado, se debe satisfacer

1
1
2
3
V ' ' ( x0 )( x x0 ) >> V ' ' ' ( x0 )( x x0 )
2
6
y de aqu, el desplazamiento respecto al punto de equilibrio debe estar acotado por

(x x0 ) << 3 VV'''''((xx0 ))
0

V. OBJETO FLOTANTE

equilibrio
mg

Cuando un objeto de masa m se introduce en un lquido y flota, se establece


un equilibrio entre el peso del cuerpo y el empuje Eeq del fluido sobre el cuerpo,
dado por
Eeq = ( masa lquido desalojado ) g
La ecuacin de equilibrio es
Eeq = mg
Si ahora se desplaza el cuerpo de esta posicin de equilibrio, surge una fuerza
restauradora igual al aumento o disminucin del peso del lquido desalojado,
respecto a su valor en el equilibrio:

F = mg E = mg Eeq + E ' = E '

donde E ' es el exceso o defecto del empuje debido al lquido desplazado respecto al
equilibrio.
E' = E + mliq g
equilibrio

mg

Supongamos que el cuerpo tiene una seccin A y se introduce una distancia


y respecto de la posicin de equilibrio. La fuerza que sufre el cuerpo es
F = mliq g
donde
mliq = gAy

es el exceso de lquido desalojado, y es la densidad del lquido. La ecuacin del


2
movimiento se obtiene de la segunda ley de Newton, F = ma , donde a = d y

dt 2

es

la aceleracin vertical que sufre el cuerpo. Obtenemos


d 2y
m 2 = gAy
dt
que corresponde a un MAS,
d 2 y gA
+
y=0
dt 2
m
con frecuencia angular
=

gA
m

Aqu m es la masa del cuerpo en movimiento, y puede expresarse como


m = c AL
siendo c la densidad del cuerpo sumergido, y L su dimensin vertical. Con esto, la
frecuencia angular tiene la expresin
g
=
c L

VI. TUBO EN U
Tenemos una columna de un determinado lquido, de seccin A y longitud
total L . Desplazamos la columna una distancia y respecto de su posicin de
equilibrio. Estudiamos el movimiento subsiguiente, en particular, la aparicin de una
fuerza restauradora con la expresin general F = ks .

y
2

equilibrio

El incremento de energa potencial respecto de su posicin de equilibrio es


igual al aumento de energa en el brazo izquierdo (donde aumenta la cantidad de
lquido una distancia y ), menos la disminucin de energa en el brazo derecho
(donde disminuye la cantidad de lquido una distancia y ). Por tanto, respecto al
equilibrio

E p = E p ( izda ) E p ( dcha )

En el brazo izquierdo, la cantidad de lquido adicional respecto del equilibrio


es, y su centro de masa est situado una distancia y por encima del nivel de
2
equilibrio. Por tanto, con la frmula usual de la energa potencial debida a la
gravedad, mgh , obtenemos
y 1
E p ( izda ) = Ayg = Agy2
2 2
De forma anloga,
1
y
E p ( dcha ) = Ayg = Agy 2
2
2
con lo cual, la energa potencial del sistema desplazado del equilibrio, es
E p = Agy 2
originndose la fuerza restauradora
dE p
Fy =
= 2 gAy
dy

La masa en movimiento en cada instante es la masa total de la columna


m = AL
2
y la aceleracin de la columna en direccin vertical es a = d y

, con lo cual la
dt 2
segunda ley de Newton nos da la ecuacin del movimiento en la forma
d 2y
AL 2 = 2 gAy
dt
que corresponde a un MAS,
d 2 y 2g
+
y=0
dt 2
L
de frecuencia angular
=

2g
L

VII. CUERPO ELSTICO


El alargamiento L producido en un cuerpo elstico por una fuerza, es
proporcional a su longitud inicial L0 . La deformacin, definida por el cociente,
L L L0
=
L0
L0
es la misma para todo el cuerpo. La fuerza aplicada dividida por la seccin A del
cuerpo se llama tensin.

Si la deformacin es muy pequea,


L
<< 1
L0
se satisface la ley de Hooke de la elasticidad,
Tensin = Y deformacin
siendo Y el mdulo de elasticidad de Young, con unidades de fuerza por unidad de
rea (presin).
Definiendo el alargamiento del cuerpo segn la variable x = L L0 , la fuerza
restauradora, debida a la deformacin elstica del cuerpo, es
x
F = Tensin A = YA
L0
y la ecuacin de movimiento para el cuerpo es
d 2x
x
m 2 = YA
dt
L0
en la direccin en que se produce el alargamiento. Esta ecuacin corresponde a un
MAS,
d 2 x YA
+
x =0
dt 2 mL0
de frecuencia angular
YA
mL0

VIII. TORSIN DE HILOS


En una torsin, si el ngulo girado es pequeo, el momento recuperador del
hilo es proporcional al ngulo girado y tiende a disminuirlo (tiene un signo negativo)
M = C
donde C es una constante recuperadora que depende del material con el que est
hecho el hilo. Unimos en el extremo del hilo un cuerpo de momento de inercia I , y
estudiamos el tipo de movimiento creado cuando giramos el sistema un ngulo .

M = C

Como el sistema es conservativo, la energa se conserva y la ecuacin de


movimiento puede obtenerse de la condicin de energa constante
dE
=0
dt
Para ello, debemos saber que la energa cintica corresponde a la energa cintica de
rotacin de un cuerpo arbitrario con un punto fijo (el punto que une el cuerpo con el
hilo), por lo que se escribe
1 d
Ec = I
2 dt

Para estudiar la energa potencial debida al giro, imaginemos que el cuerpo ha


girado un ngulo , y que cada elemento del cuerpo situado a una distancia r del
punto fijo ha recorrido una distancia lineal x dada por x = r . Entonces, si F es la
fuerza recuperadora que genera el momento de fuerzas M , la energa potencial
debida al giro es

1
C d = C 2
2
0
0
0
0
donde hemos identificado M con el momento de la fuerza recuperadora, M = Fr ,
ecuacin vlida para todos los puntos del cuerpo, independientemente del valor de
su distancia r al eje de giro.
Ep =

Fdx =

Frd =

Md =

Por tanto, de la conservacin de la energa obtenemos la ecuacin del


movimiento
d 1 &2 1

I + C 2 = 0

dt 2
2

d 2
+ C = 0
dt 2

que corresponde a un MAS,


d 2 C
+ =0
dt 2
I

con frecuencia angular


=

C
I

IX. MUELLE DE AIRE


En el equilibrio, el peso del pistn mg compensa la fuerza ejercida por el gas
debido a su presin, pA , siendo A el rea del pistn.

m
pA

equilibrio
mg

La condicin de equilibrio es

mg = pA
Si ahora subimos el pistn una distancia vertical y , al aumentar el volumen
ocupado por el gas, disminuye su presin.

m
(p + p )A

mg

As, la fuerza ejercida por el gas ya no compensa el peso del pistn, y se genera una
fuerza neta restauradora hacia abajo, de valor
F = ( p + p ) A mg = p A
siendo p la variacin de presin del gas al aumentar su volumen V .
Como el gas es ideal,

pV = cte
de donde deducimos que la variacin de presin, al variar el volumen, es
V
p = p
V
El incremento de volumen al mover el pistn es V = Ay , siendo V = AL el
volumen total del gas encerrado por el pistn en el estado de equilibrio. La presin
p corresponde al valor del equilibrio p = mg . Por tanto, el incremento de presin
A
resulta

mg y
A L
y de aqu, obtenemos la fuerza restauradora
y
F = mg
L
p =

2
La aceleracin vertical del pistn de masa m es a = d y

dt 2

y la ecuacin de

movimiento para el pistn es


d 2y
y
m 2 = mg
dt
L

que corresponde a un MAS,


d2y g
+ y=0
dt 2 L

de frecuencia angular
=

g
L

6. Asociacin de muelles
I. EN SERIE

k1
equilibrio

x1

F1 = F2

k2
equilibrio

x2

Unimos dos muelles, y calculamos la constante recuperadora efectiva del


muelle compuesto. Si el muelle 1 se alarga una distancia x1 y el muelle 2 una
distancia x2 , la distancia que se alarga el muelle compuesto es x = x1 + x2 . Como
los muelles estn unidos entre s, la fuerza que ejercen sobre el cuerpo y entre ellos
es la misma,
F = F1 = F2
Con esto, la constante recuperadora efectiva es, por definicin,
F
F
k= =
x x1 + x2
de donde obtenemos la ley de asociacin de muelles en serie,
1 x1 x2 1 1
= + = +
k F1 F2 k1 k2

II. EN PARALELO

k1

equilibrio

k2
x

x
m

Ahora, el desplazamiento los dos muelles es el mismo x1 = x2 = x , y la fuerza


sobre el cuerpo es suma de las fuerzas producidas por cada muelle, F = F1 + F2 . La
ley de asociacin de muelles en paralelo es
F F F
k = = 1 + 2 = k1 + k 2
x
x
x

7. Asociacin de hilos
I. EN SERIE
C1

M1 =M2

1
2
C2

= 1 + 2

El ngulo girado por el cuerpo es suma de los ngulos girados por los dos
hilos, = 1 + 2 y como los hilos estn unidos entre s, el momento de fuerza
generado por el giro es el mismo para los dos, M = M 1 = M 2 . Con esto, la constante
recuperadora para el sistema es
M
C=

y de aqu, obtenemos la ley de asociacin de hilos en serie


1

1
1
=
= 1+ 2=
+
C M M M C1 C2

II. EN PARALELO
C1

C2

Ahora, el ngulo girado es el mismo, = 1 = 2 y el momento recuperador es


suma de los momentos generados por cada hilo, M = M1 + M2 . De aqu, obtenemos
la ley de asociacin de hilos en paralelo
M M1 M 2
C=
=
+
= C1 + C2

Problemas Resueltos
6.1

Determinar la posicin de equilibrio y la frecuencia angular del sistema


de resorte, masa y polea mostrados. El resorte tiene una constante k, y la polea
puede considerarse como desprovista de friccin y de masa despreciable.

En el equilibrio, sobre la masa m acta su peso hacia abajo, y la tensin de la


cuerda hacia arriba. Por tanto, la condicin de equilibrio para la masa m es
mg = T
Sobre la polea acta la tensin de la cuerda hacia abajo, dos veces, una por cada lado
de la polea, y la fuerza elstica del muelle, hacia arriba. Si L es la longitud del
muelle en el estado de equilibrio, y L0 su longitud natural, la condicin de equilibrio
para la polea es
2T = k ( L L0 )
Con estas dos ecuaciones, se deduce que
mg
L = L0 + 2
k
Una vez establecido el equilibrio, desplazamos la masa m una distancia
vertical y hacia abajo, dejndola en libertad. La ecuacin de movimiento para la
masa m es
d 2y
m 2 = T
dt
donde T es la tensin de la cuerda menos su valor en el equilibrio T. Fijndonos
ahora en la polea vemos que si la masa se desplaza una distancia y hacia abajo, el
muelle debe alargarse una distancia y/2 respecto de su posicin de equilibrio. Como
la polea no tiene masa, su ecuacin de movimiento es
y
2T = k
2
de donde obtenemos
y
T = k
4
con lo cual, la masa m realiza el movimiento

d2y
y
= k
2
dt
4

que es un MAS,
d2y
k
+
y=0
2
dt
4m

de frecuencia
=

k
4m

6.2

Determinar la posicin de equilibrio y la frecuencia angular del sistema


de masa nica mostrado. La masa de la polea es despreciable.

m
En el equilibrio, sobre la masa m acta su peso y la tensin del hilo. De aqu
que la tensin de equilibrio sea
mg = T
Sobre la polea acta en la parte izquierda y hacia arriba una tensin T , en la parte
derecha, tambin hacia arriba, la fuerza elstica del muelle, y hacia abajo la tensin
del hilo. Por tanto, la condicin de equilibrio para la polea es
T + k ( L L0 ) = T
Como adems, la cuerda unida al muelle en la parte superior es inextensible, la
tensin T debe ser igual a la fuerza elstica
T = k ( L L0 )
De estas ecuaciones obtenemos la posicin de equilibrio del muelle,
T mg
k ( L L0 ) = =
2
2
mg
L = L0 +
2k
Desplazamos ahora la masa m una distancia y hacia abajo. La ecuacin de su
movimiento es
d 2y
m 2 = T
dt

donde T es la tensin del hilo menos su valor en el equilibrio T. Fijndonos ahora


en la polea vemos que si la masa se desplaza una distancia y hacia abajo, el muelle
debe alargarse una distancia 2y respecto de su posicin de equilibrio. Adems, la
cuerda y el muelle ejercen la misma fuerza, y como la polea no tiene masa, su
ecuacin de movimiento es
T = 2k ( 2 y ) = 4 ky
de donde deducimos que la masa m realiza el movimiento
d2y
m 2 = 4 ky
dt
que es un MAS,
d 2 y 4k
y=0
+
dt 2
m
de frecuencia
k
=2
m

6.3

Determinar la posicin de equilibrio, y la frecuencia angular del sistema


mostrado. Las masas y fricciones en las poleas son despreciables.

m
De forma anloga a los dos problemas anteriores, la tensin del hilo en el
equilibrio es
mg = T
La tensin de equilibrio de la primera polea satisface
2T1 = T = mg
y la tensin de la cuerda en la segunda polea debe ser igual a la fuerza elstica del
muelle
T2 = k ( L L0 )
Con esto, la ecuacin de equilibrio para la segunda polea resulta ser
T2 + k ( L L0 ) = 2 k ( L L0 ) = T1

Con estas ecuaciones se deduce que la posicin del muelle en el equilibrio


satisface
mg
2k ( L L0 ) = T1 =
2
mg
L = L0 +
4k
Ahora desplazamos hacia abajo la masa m una distancia y. La ecuacin de su
movimiento es
d 2y
m 2 = T
dt

donde T es la tensin del hilo menos su valor en el equilibrio T. Como la primera


polea no tiene masa, el valor T1 , tensin de su cuerda menos el valor en el equilibrio
T1 , satisface
2T1 = T
Al desplazarse la masa m una distancia y hacia abajo, la primera polea se desplaza
una distancia y hacia abajo, y la segunda polea una distancia 2y hacia abajo. Esto
quiere decir que el muelle debe alargarse una distancia 4y respecto de su posicin de
equilibrio. Como la cuerda es inextensible, el valor de la tensin T2 respecto del
equilibrio, es
T2 = 4ky
y como la segunda polea tampoco tiene masa, su ecuacin del movimiento es
T2 + 4ky = 8ky = T1
De estas ecuaciones obtenemos T
T = 2T1 = 16ky
con lo cual, la masa m realiza el MAS,
d 2 y 16 k
+
y =0
dt 2
m
de frecuencia
k
=4
m

6.4 Dos masas m se encuentran en el interior de un tubo liso, sin friccin,


que gira en un plano horizontal alrededor del punto fijo O, con una velocidad
angular constante . Cada una de las masas est unida a O por un resorte de
constante k. Determinar la posicin de equilibrio, y la frecuencia angular de
oscilacin del sistema.

El movimiento de cada masa es independiente, por lo que slo debemos


fijarnos en una de ellas. Sea r la distancia medida en la direccin del tubo desde la
posicin de la masa m hasta el punto O. Las fuerzas que actan sobre m en la
direccin del tubo son la fuerza centrfuga, hacia afuera, debida a la rotacin
respecto a O, y la fuerza elstica del muelle. Si r0 es la longitud natural del muelle,
la posicin de equilibrio req de la masa m satisface

m2 req = k req r0
req =

kr0
k m 2

Ahora desplazamos la masa m una distancia s hacia afuera. La coordenada


radial ser entonces
r = req + s
y de la ecuacin de movimiento para la masa m,
d 2r
m 2 = m 2r k ( r r0 )
dt
obtenemos
d 2s
m 2 = m 2 (req + s ) k ( req + s r0 )
dt
y utilizando la condicin de equilibrio, esta ecuacin se reduce a
d 2s
m 2 = m 2s ks
dt
que es la ecuacin de un MAS,
d 2s k

+ 2 s = 0
2
dt
m

de frecuencia angular
k
=
2
m

6.5

Un pequeo cuerpo orbital se desplaza de su rbita circular una


pequea distancia . Determinar el perodo y la ecuacin de movimiento de la
perturbacin.
Como ya se estudi en el primer parcial (Movimiento planetario), el
movimiento de una masa m en el campo gravitatorio terrestre satisface la ley de las
reas,
r 2& = cte
y la ecuacin del movimiento para la coordenada radial se escribe
dVef
mr&& = F ( r ) =
dr
donde
GMm
L2
Vef =
+
r
2 mr 2

es el potencial efectivo que sufre la partcula. Escrito as, el problema se reduce a


una sola dimensin, la dimensin radial y podemos emplear los resultados del MAS
en presencia de un potencial unidimensional arbitrario. Respecto a la coordenada
radial, la rbita circular es un punto de equilibrio, y respecto a dicho punto, la masa
m oscilar en un movimiento armnico, cuya frecuencia est dada por
=

Vef ( r0 )
m

donde r0 es el radio de la rbita circular.


Por tanto, al calcular , debe cumplirse que r0 es el punto de equilibrio radial,
esto es
Vef ( r0 ) = 0

GMm
L2
=
r02
mr03
La posicin de equilibrio est dada por
L2
r0 =
GMm2
Entonces, la derivada segunda del potencial en el punto de equilibrio es
GMm
L2

Vef ( r0 ) = 2 3 + 3 4
r0
mr0
y eliminando L2 utilizando el valor r0 , obtenemos
GMm
GMm GMm
Vef ( r0 ) = 2 3 + 3 3 = 3
r0
r0
r0
Con esto, la frecuencia angular de la oscilacin respecto a la rbita circular es
GM
=
r03
y la ecuacin de movimiento corresponde al MAS,
GM
&& + 3 = 0
r0
donde es el desplazamiento radial respecto de la rbita circular
= r r0

6.6

Determinar el momento de inercia de un cuerpo plano de masa m a


partir de las observaciones de la frecuencia angular de la oscilacin libre del
cuerpo cuando ste se suspende de dos hilos ligeros por dos extremos opuestos
del cuerpo. La longitud de los hilos es H, y la distancia entre los puntos de
suspensin en el cuerpo es D. Generalizar al caso de que tuvisemos N hilos de
suspensin.

Al moverse el cuerpo en el plano horizontal, asciende y desciende una


cantidad pequea. Inicialmente, cuando no existe giro, las tensiones de los hilos no
producen un torque sobre el eje vertical. Pero cuando el cuerpo asciende, los hilos ya
no estn verticales, y existe un torque descompensado no nulo. Este torque est
dirigido en sentido contrario a la rotacin, y se produce un MAS. Sea el ngulo que
forman los hilos con la vertical cuando el cuerpo ha girado un ngulo .

Por la geometra del sistema, los dos ngulos no son independientes sino que existe
la relacin

d
D/2

d = H sin =

D
sin
2

que expresa el hecho de que la proyeccin sobre el plano horizontal del giro del hilo
coincide con el giro del cuerpo.

El momento de giro respecto al punto O, generado por la tensin en los hilos


es
D
T sin
2
El signo menos indica que el momento tiende a disminuir el ngulo de giro. Con
ayuda de la anterior relacin geomtrica, la ecuacin de rotacin para el cuerpo
respecto de la vertical es
D
D2
I&& = M = TD sin = TD
sin = T
sin
2H
2H
Para ngulos pequeos, sin , obtenemos la ecuacin de un MAS,
D2
&& + T
=0
2 HI
M = 2

Para determinar la tensin de los hilos, suponemos que para ngulos


pequeos, la componente vertical de las tensiones sean aproximadamente igual a T,
y estas tensiones deben equilibrar el peso del cuerpo. As,
2T = mg
con lo cual
mg
T=
2
y la frecuencia angular de oscilacin es
= T

D2

mgD 2

2 HI
4HI
As, pues, conocida la frecuencia de oscilacin del cuerpo, midiendo su perodo de
oscilacin, el momento de inercia del cuerpo est dado por
TD 2
mgD 2
I=
=
2 H 2 4 H 2

Cuando tenemos el cuerpo colgado de N hilos, la tensin sobre cada hilo es


mg
T=
N
y el torque
D
M = N T sin
2
y se comprueba que la expresin para la frecuencia angular, y de aqu la expresin
para el momento de inercia no varan, como funciones de la tensin sobre cada hilo.

6.7

Determinar la frecuencia de oscilacin de un resorte de constante k


unido por un extremo a una pared, y por el otro a una masa m, si consideramos
que el resorte tiene una masa no nula M.

El sistema sigue siendo conservativo, el movimiento se produce al


transformarse la energa cintica en energa potencial elstica, y viceversa. Al
considerar que el muelle tiene una masa no nula, la energa potencial del sistema se
deduce de la misma expresin, pero la energa cintica s vara. Es decir, la ecuacin
de conservacin de la energa se escribe
1
1
E = mV 2 + Ec , muelle + kx 2
2
2
siendo
dx
V=
dt
la velocidad de la masa m.
La energa cintica del muelle es, por definicin,
1
2
Ec , muelle =
dm Vdm
2
siendo dm un elemento arbitrario de masa, con velocidad V dm . Escogemos un
sistema de coordenadas fijo en la pared, siendo s la distancia del elemento de masa
dm a la pared cuando el resorte est en reposo. Cuando el resorte se alarga, vamos a
suponer que el alargamiento del muelle hasta el elemento dm es proporcional a su
longitud en reposo s, esto es,
l = k (x ) s
donde la constante k depende de x. Para determinar esta constante, sabemos que el
alargamiento total es x,

x=

L0

dl =

L0

k ( x ) ds =k (x ) L 0

siendo L0 la longitud del muelle en reposo. Entonces,


x
k (x) =
L0
con lo cual, el alargamiento l del muelle hasta el elemento dm es
x
l=
s
L0
La velocidad del elemento de masa dm respecto a nuestro sistema de
coordenadas ser igual a la variacin temporal de la longitud l. Esto es,
dl s dx s
Vdm = =
= V
dt L0 dt L0
Adems, si el muelle es homogneo, el elemento de masa dm es
M
dm =
ds
L0
y con esto, la energa cintica del muelle se escribe

1M 2 0 2
1
=
V
s
ds
MV 2
3
2 L0
6
0
y la energa cintica total del sistema es
1
1
1
M
Ec = mV 2 + MV 2 = m + V 2
2
6
2
3
De la conservacin de la energa, obtenemos la ecuacin de un MAS, de forma
anloga al caso de un muelle sin masa. La frecuencia de oscilacin es
k
=
M
m+
3
Ec , muelle =

Problemas Propuestos
6.7

Determinar por el mtodo de energa de Rayleigh, y por el mtodo de la


forma funcional, la frecuencia de oscilacin del sistema dado.

m
c

k
Solucin: 2 = 2

6.8

kb
g

2
mc
c

Una partcula desliza hacia atrs y hacia delante, sin friccin, entre dos
planos inclinados que forman un ngulo con la horizontal, unidos suavemente
por su punto ms bajo. Supongamos que la partcula parte del reposo en un
punto inicial a una altura h desde la horizontal.
a) Calcular su perodo, y determinar si se trata de un movimiento
oscilatorio, armnico simple, peridico.
b) Determinar la componente horizontal de la velocidad de la partcula
cuando pasa por el punto de unin de los dos planos
c) Considerando la forma de la trayectoria, si quisiramos aproximar el
movimiento a un MAS, podramos hacerlo en el punto ms alto de la
trayectoria, en el punto ms bajo de la trayectoria, o en ningn punto de
la trayectoria.

Apndice: Oscilaciones armnicas en dos dimensiones


Estudiamos el movimiento de una partcula situada en un plano horizontal,
que sufre la accin de una fuerza restauradora proporcional a la distancia de la
partcula al origen de coordenadas, en la forma vectorial
F = kr
El sistema tiene dos grados de libertad, ( x , y ) , y su ley de movimiento
mx&& = kx
my&& = ky
admite las soluciones
x = A cos ( 0t )
y = B cos ( 0t )

donde 0 = k

. El movimiento se compone de oscilaciones armnicas en cada


m
una de las direcciones, con la misma frecuencia pero distintas amplitudes y fases
iniciales.
Si eliminamos la variable de tiempo t , obtenemos la ecuacin de la
trayectoria de la partcula, en la forma
x 2 y2
x y
+ 2 2cos
= sen 2
2
A
B
AB
siendo = el desfase inicial de las dos oscilaciones armnicas. Del valor de
ese desfase depende el tipo de movimiento realizado por la partcula, ya que dicho
desfase est relacionado directamente con las condiciones iniciales de movimiento
x& ( 0)
tan =
0 x ( 0 )
tan =

y& ( 0)
0 y ( 0 )

En el caso general, la trayectoria es una elipse centrada en el origen.


Estudiamos por simplicidad el caso particular A = B . Entonces, la trayectoria es una

elipse con semiejes de longitudes 2 A cos , 2 A sen


respectivamente y uno
2
2
de ellos forma un ngulo de 45 con el eje x .
a) Cuando = , 3 , los dos semiejes tienen el mismo valor A , y la
2
2
trayectoria elptica se transforma en una trayectoria circular de radio A .
b) Cuando = 0, , uno de los dos semiejes se hace igual a cero, y la
trayectoria elptica se transforma en la trayectoria rectilnea y = x .

En todas las trayectorias, el sentido de giro en el plano xy se determina


conociendo la variacin temporal del ngulo polar definido en dicho plano. De la
transformacin de coordenadas polares a coordenadas cartesianas
y
= arctan
x
obtenemos
d
1
d y
1
& xy
& )
=
= 2
yx

2
2(
dt
y dt x x + y
1+
x
El signo de la velocidad angular de giro es igual al signo del ltimo factor.
Evaluando este factor
& xy
& = A2 sen ( t ) cos ( t ) cos ( t ) sen ( t )
yx
= A2 sen ( ) = A2 sen
vemos que para 0 < < , esta velocidad es negativa y el giro se realiza en sentido
de las agujas del reloj. Para < < 2 , la velocidad angular es positiva y el giro se
realiza en el sentido contrario a las agujas del reloj.
Todas estas conclusiones se resumen en las siguientes grficas de las trayectorias
bidimensionales, calculadas en el caso particular A = B = 1 .

= 0

= 30

= 60

= 90

= 120

= 150

= 180

= 210

= 240

= 270

= 300

= 330

En general, las oscilaciones bidimensionales no tienen por qu poseer las


mismas frecuencias angulares en los movimientos segn los ejes x,y, indicando la
existencia de cierta anisotropa espacial. La solucin general sera en este caso
x = A cos ( xt )

y = B cos yt

y la trayectoria no ser una elipse, sino una de las llamadas figuras de Lissajous.
Estas curvas sern cerradas si el movimiento se repite a intervalos regulares de
tiempo, lo cul slo es posible si las frecuencias son conmensurables entre s, es

decir, cuando el cociente x


sea una fraccin racional. En caso de que no sea una
y
fraccin racional, la trayectoria ser abierta, que pasar por cualquier punto del
rectngulo de lados A,B. Es decir, la curva llena todo el espacio interior a este
rectngulo. Por simplicidad tomamos de nuevo el caso A = B = 1 . Algunas figuras
de Lissajous se representan en las siguientes grficas, para el caso 3 x = 2 y

= 0

= 15

= 30

= 60

Segunda Parte: Oscilaciones Amortiguadas


El amortiguamiento es la causa de la reduccin en amplitud de las
oscilaciones, y est asociado con la disipacin de energa producida por
I. Amortiguamiento viscoso, cuando el cuerpo se mueve en el seno de un
fluido. La fuerza de resistencia v es proporcional a la velocidad
relativa entre el mvil y el fluido.
II. Amortiguamiento de Coulomb, causado por la friccin entre dos
superficies en contacto. La fuerza de rozamiento, N , es proporcional
a la componente normal de la fuerza de contacto entre las dos
superficies.
III. Amortiguamiento histertico, producido por la friccin interna cuando
un slido es deformado.
IV. Amortiguamiento elctrico, producido por la disipacin de energa por
efecto Joule.

6.9

Determinar la disipacin de energa por amortiguamiento viscoso en un


perodo de movimiento si el movimiento del cuerpo est dado por x = A cos t .
La energa perdida por rozamiento es igual al trabajo de la fuerza viscosa
dE = vdx
y la razn de disipacin de energa es la prdida de energa en la unidad de tiempo.
Es decir,
dE
dx
= v = v 2
dt
dt
En un ciclo de movimiento, la energa total perdida ser
E =

dE
dt =
dt

v2 dt

La velocidad instantnea del mvil es


dx
v=
= A sen t
dt
con lo cual

A2 2T
2
0
Esta prdida de energa conlleva una disminucin progresiva en la amplitud de las
oscilaciones del mvil.
E = A
2

sin2 t dt =

1. Oscilaciones amortiguadas
Estudiamos las oscilaciones de un sistema de un grado de libertad, que posee
una cierta masa, est sometido a una fuerza elstica y a una fuerza viscosa,
proporcional a su velocidad instantnea. La ecuacin del movimiento se sigue de la
ley de Newton
d 2x
m 2 = kx v
dt
o en virtud de v = dx
dt
d 2 x dx k
+
+ x =0
dt 2 m dt m
Definimos los parmetros
k

=
m
2m
con lo que la ecuacin de movimiento queda
d 2x
dx
+
2

+ 02 x = 0
2
dt
dt
0 =

La solucin general es de la forma x = Ae t . Introduciendo esta expresin en


la ecuacin de movimiento, vemos que es la solucin de la ecuacin caracterstica
2 + 2 + 02 = 0
con las dos races
1,2 = 2 02

La expresin para cambia de real a imaginario cuando el coeficiente de


amortiguamiento alcanza el valor crtico
k
m
La cantidad de amortiguamiento en cualquier sistema puede especificarse
convenientemente por la razn entre el coeficiente de amortiguamiento y el
coeficiente de amortiguamiento crtico

=
c
Utilizando este valor como referencia las dos races pueden expresarse como
c = 0 =

1,2 = 0 2 1

de lo que se sigue que el comportamiento del sistema amortiguado depende del valor
de .

2. Movimiento subamortiguado o subcrtico (0 < ? < 1)


Esta es la condicin ms comn en la prctica. La solucin general puede
escribirse en la forma

x = Ae1t + Be 2t = Ae 0 t cos 0 1 2 t +

El trmino en coseno muestra que el movimiento es oscilatorio con un perodo


2
T=
0 1 2
y el trmino exponencial muestra que la amplitud del movimiento oscilatorio
A(t ) = Ae 0 t
decrece en el tiempo debido al rozamiento viscoso.
En la grfica siguiente se muestran las distintas curvas de decaimiento para
distintos valores del coeficiente adimensional de rozamiento, . Como datos
iniciales, se considera que el sistema parte del reposo, a una distancia A del punto
de equilibrio x = 0 . Entonces, la fase instantnea debe satisfacer
dx
2
dt = A 0 cos ( ) A 0 1 sin ( ) = 0
t = 0
tan =

1 2
Para el sistema no amortiguado se elige 0 = .
1

=0
(osc. libre)
= 0 .1

x
A

= 0 .2

-1
0

tiempo t

6.10 Determinacin experimental de la viscosidad de un fluido


Una placa delgada de superficie total A y masa m se une a un muelle de
constante k , y se le permite oscilar verticalmente. La posicin de equilibrio es
k ( L L0 ) = mg
Si desplazamos la placa una distancia x respecto de esta posicin de equilibrio, la
ecuacin del movimiento
mx&& = mg k ( L + x L0 ) = kx
corresponde a un movimiento armnico de frecuencia 0 = k m y perodo de
oscilacin
T0 = 2

m
k

Introducimos ahora el sistema en un fluido, que se opone al movimiento de la


placa con una fuerza viscosa, dx . La nueva posicin de equilibrio debe tener
dt
en cuenta el empuje del fluido sobre la placa, resultando
k ( L ' L0 ) = ( m m ') g
siendo m ' la masa del fluido desalojado. Una vez desplazada la placa respecto de su
posicin de equilibrio una distancia x , la ecuacin de movimiento
dx
mx&& = (m m ') g k ( L '+ x L0 )
dt
dx
= kx
dt
corresponde a un movimiento amortiguado. Supongamos que la placa puede oscilar
en el seno del fluido. En dicho caso, el coeficiente de amortiguamiento debe ser
inferior a su valor crtico, la frecuencia de oscilacin es = 0 1 2 y el perodo
de oscilacin vale
T = 2

m
1
k 1 2

Relacionando los perodos del movimiento oscilatorio libre y del movimiento


amortiguado en el seno del fluido, vemos que el coeficiente de amortiguamiento
debe estar dado por
T
= 1 0
T

La fuerza viscosa que ejerce un fluido sobre un cuerpo que se mueve en su


seno es proporcional al rea del cuerpo en la direccin de su movimiento, a su
velocidad y a la viscosidad del fluido, . Entonces, establecemos la igualdad
= A
y de aqu, la viscosidad del fluido vale

2m 0 4 m
T
1 0
=
=
A
A
AT0
T

6.11 Un disco de radio R y momento de inercia I est ligado a un hilo de


constante elstica C bajo torsin. Si la frecuencia natural de oscilacin libre es
f1 y vale f2 cuando el sistema se sumerge en un cierto fluido, determinar el
coeficiente de amortiguamiento del fluido.
I
Solucin: = 4 2 f12 f 22
R

3. Movimiento amortiguado crtico (? = 1)


El caso = 1 se llama amortiguamiento crtico porque representa la transicin
entre los movimientos oscilatorios y no oscilatorios, en presencia de una fuerza de
amortiguamiento. Segn la teora de las ecuaciones diferenciales, la solucin general
de la ecuacin del movimiento cuando las dos races son iguales
1 = 2 = c = 0
es de la forma
x = ( A + Bt ) e 0t
Como caso particular, si el cuerpo parte del reposo en la posicin x = A, la
solucin es x = A(1 + 0t ) e 0t . Tomando de nuevo el valor 0 = , la grfica del
movimiento
1

=1

0 =

x
A

tiempo t

muestra que el cuerpo nunca ejecuta una oscilacin, sino que el rozamiento es lo
suficientemente grande como para frenar el movimiento justo cuando el cuerpo pasa
por su posicin de equilibrio. Aunque no tiene significado fsico, el cuerpo tarda un
tiempo infinito en llegar a esta posicin. Hay que recordar que una fuerza de
rozamiento proporcional a la velocidad instantnea del mvil deja de tener sentido
fsico si la velocidad es muy pequea.

6.12 Un dispositivo de disparo se disea de forma que tras el disparo un


muelle detenga el retroceso del sistema. Al final del retroceso acta un
mecanismo amortiguador de forma que el sistema vuelva a su posicin inicial
sin ejecutar ninguna oscilacin. Determinar la constante elstica k del muelle y
el coeficiente de amortiguamiento si la masa del sistema es m, la velocidad
inicial de retroceso es V0, y la distancia total de retroceso es d.
Durante el retroceso el mecanismo amortiguador no es operativo. El retroceso
termina al recorrer una distancia d si toda la energa cintica inicial se transforma
en energa potencial del muelle. Para ello es necesario que
1
1
mV02 = kd 2
2
2
2
mV
k = 20
d
Para que el sistema no ejecute oscilaciones al volver a su posicin de equilibrio es
necesario que el amortiguamiento sea crtico. Es decir, el coeficiente de
amortiguamiento vale
k
V
= 2m 0 = 2m
= 2m 0
m
d

4. Movimiento sobreamortiguado (? > 1)


En este caso, las dos races de la ecuacin caracterstica son distintas, y ambas
son reales negativas. La solucin general es de la forma
x = Ae1t + Be 2t
suma de dos exponenciales que decaen en el tiempo. El movimiento no es peridico,
el sistema no oscila, y se ve frenado de manera ms severa que en el caso crtico. La
posicin de equilibrio se alcanza de forma ms lenta que en el movimiento crtico.
Este tipo de movimiento es de poco inters prctico.

6.13 Un sistema que ejecuta oscilaciones libres de frecuencia 0 , se ve

sobreamortiguado con un coeficiente ? = 2 . Si parte del reposo en la posicin


x = A , determinar su ley de movimiento y la amplitud de movimiento que ha
perdido cuando ha transcurrido un tiempo igual a un periodo de la oscilacin
libre del sistema. Calcular este ltimo resultado si el amortiguamiento fuese
crtico.
De la solucin general,

( + 1) t + De( 1) t
2

x = Ce
las condiciones iniciales se satisfacen si
A= C+ D

) (

0 = C + 2 1 + D 2 1

con la solucin

C=

1 +

D=

Al cabo de un perodo de oscilacin libre, 0t = 0T0 = 2 , la amplitud del


movimiento es
( + 2 1 )2 1
( 2 1)2
x 1

e
e
= 1 +
+ 1
2
2

A 2
2
1
1

Tomando el valor = 2, obtenemos x = 0.20, es decir, se ha perdido el 80% de la


A
amplitud inicial de movimiento.
En el caso crtico, la solucin que satisface la condicin inicial se calcul
anteriormente
x
= (1 + 0t )e 0 t
A
resultando x = (1 + 2 )e 2 0.014 , habiendo perdido en este caso prcticamente
A
la totalidad del movimiento inicial.

5. Parmetros del movimiento subamortiguado


Como hemos visto este tipo de movimiento es el que tiene ms utilidad
prctica. Existen tres parmetros fundamentales que definen el comportamiento del
movimiento subamortiguado.
Tiempo de relajacin t . Es el tiempo necesario para que la energa del sistema
disminuya en un factor 1 . Siendo un tipo de movimiento oscilatorio, la energa
e
total se define como
1
E = kA2 ( t )
2
0 t
la amplitud del movimiento. Entonces, si E0 es la energa
siendo A(t ) = Ae
inicial del movimiento, al cabo de un tiempo t el sistema tendr una energa
E (t ) = E0 e 2 0 t . Por definicin del tiempo de relajacin
E (t )
E (t + ) =
e
con lo cual
2 0 = 1
1
2 0
As, cuanto mayor sea el amortiguamiento menor ser el tiempo de relajacin.
=

Decremento logartmico . Debido al rozamiento, el sistema pierde energa y la


amplitud del movimiento decrece. Definimos e como el factor por el que
disminuye la amplitud despus de un ciclo. Es decir,
x (t + T ) = e x (t )
Como la parte que depende de la funcin coseno no vara al cabo de un ciclo, esta
igualdad se debe satisfacer para la funcin amplitud
Ae 0 ( ) = e Ae 0 t
= 0 T
siendo T el perodo del movimiento subamortiguado
2
T0
T=
=

1 2
t +T

Factor de calidad Q. Aunque en este tema apenas se hable de Q , su importancia


radica en su aplicacin prctica. Cuanto mayor sea el factor Q para un sistema,
menor ser el rozamiento sufrido y mayor el nmero de oscilaciones del sistema
antes de que su amplitud disminuya de forma considerable. En los sistemas reales, el
factor Q depende de numerosos parmetros fsicos del sistema, que deben ajustarse
de manera que sea lo mayor posible. En nuestro caso, para sistemas simples, se
define por

1
Q= 0 =

6. Movimiento con amortiguamiento seco


La fuerza de rozamiento seco entre dos superficies slidas en contacto puede
tomarse independiente de la velocidad, eso s, siempre acta en direccin contraria
al movimiento. En general, F = N siendo N la fuerza de contacto entre las dos
superficies, y el coeficiente de rozamiento supuesto constante. La caracterstica
ms importante de este tipo de movimiento es que conserva la oscilacin libre, es
decir, oscila con el mismo perodo T0 pero su amplitud se ve progresivamente
disminuida por la disipacin de energa en forma de calor.

6.14 Una masa m que desliza sobre una superficie horizontal rugosa, est
unida a un muelle de constante elstica k con uno de sus extremos fijo en la
pared. Determinar el movimiento de la masa m, la variacin de su amplitud al
cabo de un perodo, y su posicin final de equilibrio. El coeficiente de
rozamiento entre el suelo y el cuerpo es .
La ecuacin del movimiento debe incluir la fuerza elstica del muelle y la
fuerza de rozamiento. Ya que dicha fuerza de rozamiento es contraria al
movimiento, su direccin depender de la direccin del movimiento. Tenemos dos
ecuaciones del movimiento: cuando la masa se dirige de izquierda a derecha, su
velocidad x& es positiva y la fuerza de rozamiento es negativa, dirigida de derecha a
izquierda. La ecuacin de movimiento para esta direccin es

mx&& = kx mg
&&
x + 02 x = g

De forma anloga, cuando la masa se mueve de derecha a izquierda, la fuerza


de rozamiento va en la direccin x positiva, y la ecuacin de movimiento es
mx&& = kx + mg
&&
x + 02 x = g

Resolvemos ahora las ecuaciones de movimiento suponiendo que inicialmente


la masa se separa una distancia positiva x0 de la posicin de equilibrio, por lo que
inicialmente se mueve de derecha a izquierda. La solucin de la ecuacin del
movimiento es suma de una oscilacin libre ms una solucin x = cte. Cuando se
mueve de derecha a izquierda la solucin es
g
x = A cos ( 0t + ) + 2
0
Las condiciones iniciales son x = x0 , x& = 0, por lo que tenemos
g
x0 = A cos + 2
0
0 = A 0 sen

con la solucin
=0
A = x0

g
02

g
g
x = x0 2 cos 0t + 2
0
0

El movimiento de derecha a izquierda no termina en la posicin x = x0 tal y


como sera si no existiera rozamiento. El rozamiento hace perder energa al sistema,
y la amplitud del movimiento va decreciendo progresivamente. Para determinar el
punto donde termina el movimiento inicial de derecha a izquierda, averiguamos en
que punto se para, para iniciar as su movimiento de izquierda a derecha
completando su primer ciclo. El punto donde se para satisface x& = 0. Es decir,
sen 0t = 0 . La solucin cos 0t = 1 corresponde al punto de partida y la solucin
cos 0t = 1 corresponde al punto de retorno x1 < 0

g g
g
x1 = x0 2 + 2 = x0 2 2
0 0
0

Podemos decir entonces que el sistema ha perdido una amplitud 2 g


el viaje de ida en su primer ciclo.

02

al recorrer

Anlogamente se determina la ley de movimiento para el viaje de vuelta del


primer ciclo. La solucin general es
g
x = B cos ( 0t + ) 2
0
con las condiciones de salida cos 0t = 1,sen ? 0t = 0, x = x1 , x& = 0. Es importante
subrayar que el tiempo se mide siempre desde el instante inicial, es decir, el
movimiento de vuelta no comienza de nuevo en t = 0 sino en un tiempo tal que
cos 0t = 1. Obtenemos la solucin
=0
g
g
B = x1 2 = x0 3 2
0
0

g
g
x = x0 3 2 cos 0t 2
0
0

De nuevo, la posicin donde se para tras concluir el primer ciclo corresponde al


valor cos 0t = 1, ( x& = 0 ) , resultando

g
02
Podemos concluir entonces que la amplitud perdida en cada ciclo es igual a
4g 2 .
0
x2 = x0 4

Para determinar la posicin final de la masa, supondremos que el coeficiente


de rozamiento no slo es dinmico, es decir, genera una fuerza de rozamiento
cuando las dos superficies se mueven entre s, si no que tambin es un coeficiente de
rozamiento esttico, es decir, genera una fuerza de rozamiento que la masa debe
superar al ponerse en movimiento. Entonces, la masa se quedar en reposo en
cualquiera de sus puntos de retorno si en esa posicin la fuerza elstica no es capaz
de superar a la fuerza esttica mg . Es decir, se para cuando el punto de retorno nsimo satisfaga
kxn < mg
Esto es,

g
k x0 2n 2 < mg
0

1 kx
n > 0 1
2 mg
La fuerza de rozamiento dinmica no es capaz de hacer pararse a la masa m , slo
acta de forma progresiva quitndole energa que transforma en calor. Es la fuerza
de rozamiento esttica la que obliga a pararse a la masa, y esto slo puede suceder
en uno de los extremos del movimiento, cuando la masa retorna su movimiento.

Tercera Parte: Oscilaciones Forzadas


Las oscilaciones forzadas se producen cuando se aplican fuerzas exteriores
sobre un sistema vibratorio. Dicha fuerza exterior puede ser un simple impulso
instantneo, una oscilacin mantenida, o incluso puede estar causada por fuerzas de
inercia. Despus de un cierto tiempo, la oscilacin natural del sistema (rgimen
transitorio) desaparece por la presencia de fenmenos de resistencia, mientras que la
oscilacin estacionaria, debida a la fuerza exterior, tiene su misma frecuencia y
perdura en el tiempo. El rgimen transitorio tiene importancia prctica slo al
principio del movimiento, cuando la oscilacin natural del sistema no ha sido
amortiguada apreciablemente. Aunque en la prctica siempre existe cierto
amortiguamiento, es interesante estudiar el caso lmite de las oscilaciones forzadas
con amortiguamiento cero por su sencillez y porque las conclusiones que pueden
extraerse de este modelo matemtico son vlidas en presencia de amortiguamiento.

1. Oscilacin forzada no amortiguada


Segn la ley de Newton, la ecuacin de movimiento es
d 2x
m 2 = kx + F0 cos t
dt
Las fuerzas que actan sobre el sistema son la fuerza restauradora del muelle, y la
fuerza exterior de variacin armnica. Introduciendo la frecuencia del sistema no
perturbado,
0 =

k
m

escribimos esta ecuacin como un MAS


d 2x
F
+ 02 x = 0 cos t
2
dt
m
Solucin de esta ecuacin. En general, cuando el lado izquierdo depende de
la variable x y el derecho no, la solucin puede escribirse como la suma
x = xg + x p
siendo x g la solucin general de la ecuacin con el lado derecho igual a cero
d 2x
+ 02 x = 0
2
dt
y x p una solucin particular de la ecuacin general
d 2x
F
+ 02 x = 0 cos t
2
dt
m
En nuestro caso, sabemos que la solucin general es
x g = A cos ( 0t + )

donde las constantes A, dependen de las condiciones iniciales del movimiento.


Para encontrar la solucin particular, suponemos que la respuesta del sistema a la
fuerza exterior es proporcional a sta. Dicho de otra forma, esperamos que la

respuesta del sistema sea lineal con la perturbacin exterior que recibe. Con esto, la
solucin particular ser de la forma
x p = C cos t
Introduciendo x p en la ecuacin del movimiento, se satisface
2C cos t + C 02 cos t =

F0
cos t
m

Despejando la amplitud C ,
C=

F0
2 )

m( 02

que es la amplitud del movimiento forzado.


Por tanto, la masa m realiza el movimiento
F
1
x = A cos ( 0t + ) + 0 2
cos t
m 0 2
Es la composicin de una oscilacin libre (primer trmino) y una oscilacin de
arrastre debido a la fuerza exterior (segundo trmino).

2. Resonancia
Fijndonos en la solucin particular x p vemos que si la frecuencia de la
fuerza exterior coincide con la frecuencia natural 0 del sistema, la amplitud de la
oscilacin forzada tiende a infinito
C si 0
Es el fenmeno de la resonancia. Fsicamente expresa el hecho de que cuando
= 0 , toda la energa comunicada al sistema por la fuerza exterior es almacenada
por el sistema, con lo que la amplitud crece sin lmite. Veremos ms adelante que si
0 la potencia media (energa transferida por ciclo) es cero, la energa del
sistema se conserva y la amplitud del movimiento se mantiene constante.
Cualquier sistema fsico sufre algn tipo de amortiguamiento debido al
rozamiento. En ese caso, se mantiene el fenmeno de la resonancia, pero la amplitud
de la oscilacin forzada no tiende a infinito, llega a ser muy grande, pero se
mantiene finita.

3. Rgimen transitorio y permanente


En cualquier sistema real existen fuerzas de rozamiento que hacen que la
energa se disipe, se transforme en otras formas de energa (calor). Como la fuerza
exterior acta indefinidamente, es la oscilacin libre (primer trmino de la solucin)
la que pierde energa y tiende a remitir. Por tanto, en la etapa inicial, cuya duracin
depende de la intensidad de la amortiguacin, los dos movimientos (libre y forzado)
son importantes. Es el rgimen transitorio. Despus de un tiempo suficientemente

largo, el nico movimiento presente es la oscilacin forzada de frecuencia . Es el


rgimen permanente o estacionario.

4. Desfase respecto a la fuerza exterior


En el rgimen permanente,
x = C cos t =

F0
1
cos t
2
m 0 2

y de aqu se observa que


a) Si < 0 , la amplitud C del movimiento forzado es positiva. La partcula y
la fuerza exterior estn en fase, y oscilan en el mismo sentido.
b) Si > 0 , la amplitud C del movimiento forzado es negativa. La partcula y
la fuerza exterior estn en desfase, y oscilan en sentido contrario.

5. Potencia absorbida por un oscilador forzado


Durante el intervalo de tiempo dt , la fuerza exterior realiza un trabajo dW
sobre el sistema
dW = Fext dx
siendo dx el desplazamiento del sistema en ese tiempo. La potencia instantnea
suministrada al sistema por la fuerza exterior ser, entonces
dW
dx
P=
= Fext
= FextV
dt
dt
siendo V la velocidad del sistema.
Para un oscilador forzado, en el rgimen permanente, hemos visto que
Fext = F0 cos t
x=

F0
1
cos t
2
m 0 2

F0
1
sen t
2
m 0 2
con lo cual, la potencia instantnea recibida por el sistema es
F 2
1
P= 0
sen t cos t
2
m 0 2
Dicha potencia es positiva, es decir, el sistema absorbe energa por la accin de la
fuerza exterior, cuando sen t cos t < 0 . Y el sistema cede energa al exterior, en
contra del efecto de la fuerza exterior, cuando sen t cos t > 0 .
V =

La potencia media recibida en un ciclo de oscilacin de perodo T = 2

P=

1 T
Pdt = 0
T 0

es

ya que el valor medio del producto seno-coseno es nulo, donde el perodo de


oscilacin es. Esto quiere decir que en un ciclo completo, el sistema ni gana ni
pierde energa, y as la oscilacin mantiene una amplitud constante. Esto es cierto en
el rgimen permanente. En el rgimen transitorio, el amortiguamiento existente, por
pequeo que sea, produce una potencia negativa, y la oscilacin libre pierde
intensidad paulatinamente hasta anularse.

6. Movimiento generado por fuerzas de inercia


Un caso importante en el estudio de la oscilacin forzada ocurre cuando la
fuerza exterior aplicada al sistema es una fuerza de inercia. El sistema se ve
arrastrado por estar ligado a un punto exterior en movimiento. El movimiento ser la
combinacin de la oscilacin libre del sistema respecto de su punto de equilibrio y
del movimiento de arrastre del punto de equilibrio. Supongamos que el punto de
equilibrio se ve obligado a moverse segn xeq = cos t . Si m es la masa del
sistema, la fuerza de inercia debida al movimiento del punto de equilibrio es
Fext = mx&&eq = m 2 cos t
con lo cual el estudio de este tipo de movimiento es anlogo al caso general, con la
sustitucin directa
F0 = m 2
2
C= 2
0 2

Problemas Resueltos
6.15 Determinar las condiciones iniciales de movimiento para que un sistema
oscilatorio sometido a la fuerza exterior F0 cos Ot no tenga rgimen transitorio.
El movimiento general es suma de la oscilacin libre y la oscilacin forzada.
La primera corresponde el rgimen transitorio, y la segunda al rgimen estacionario.
Debemos determinar cules son las condiciones iniciales para la variable espacial y
la velocidad para que no exista oscilacin libre. Tenemos
F0
x ( t ) = A cos ( 0t + ) +
cos t
2
m( 0 2 )
F0
x& ( t ) = A sen ( 0t + )
sen t
m( 02 2 )
Las condiciones iniciales son
x0 = x ( 0) = A cos +

F0
2 )

m ( 02

V0 = x& ( 0) = A 0 sen

de donde despejamos el valor A que determina la amplitud del rgimen transitorio


(oscilacin libre)
2

V02
F0
A = x0
+
m ( 02 2 ) 2

Para que dicha amplitud sea cero, es necesario que


F0
x0 =
2
m( 0 2 )
V0 = 0

6.16 Una masa puntual m est ensartada en un anillo vertical de radio R, y


oscila libremente respecto de su punto de equilibrio situado en el punto ms
bajo del anillo. Se aplica un momento de fuerza exterior que hace oscilar al
anillo respecto de su centro en la forma
= 0 cos t
siendo el ngulo de giro. Determinar el movimiento general de la masa m, y
bajo que circunstancias dicho movimiento entra en resonancia, detallando
dicho fenmeno.
Los momentos de fuerza sobre la masa m se deben a su peso y a la fuerza de
inercia generada por el movimiento del punto de equilibrio, resultando
M = mgR sen mR 2&& cos
siendo el ngulo de giro de la masa respecto del punto de equilibrio. Suponiendo
que es pequeo,
sen
cos 1
la ley de Newton para la rotacin nos da la ecuacin de movimiento en la forma
mR 2&& = mgR mR 2&&
g
&& + = && = 0 2 cos t
R

La solucin general es combinacin de una oscilacin libre de frecuencia


g
0 =
R
y una oscilacin forzada, con la misma frecuencia que el movimiento forzado del
anillo,
2
= 0 cos ( 0t + ) + 2 0 2 cos t
0
El movimiento entra en resonancia cuando la frecuencia exterior es igual que
la frecuencia de la oscilacin libre. En este caso se produce cuando
g
= 2
R

y dicho fenmeno se observa por que la amplitud de la masa m crece


indefinidamente. Es decir, la masa deja de oscilar respecto del punto de equilibrio y
comienza un movimiento de libracin, dando vueltas completas alrededor del anillo
en una misma direccin.
Pregunta: En el caso de que se produzca, la resonancia la amplitud del
movimiento tiende a ser grande. Pero al deducir la ecuacin de movimiento
hemos supuesto que el ngulo de giro es pequeo. Tiene validez la conclusin
anterior sobre el fenmeno de resonancia cuando el ngulo de giro no se supone
pequeo?

6.17 Un pndulo, formado por una masa puntual m unida a una cuerda de
longitud L, se ve arrastrado en su punto de suspensin S por el movimiento
xS = cos t . Determinar bajo que condiciones el movimiento resultante del
pndulo puede interpretarse como un movimiento oscilatorio forzado segn la
descripcin terica desarrollada anteriormente, y deducir en ese caso, el valor
de la frecuencia exterior para que la amplitud de movimiento del pndulo
sea a) mayor que 2 , b) menor que .
2
En primer lugar, el movimiento forzado se produce cuando la fuerza exterior
est dirigida en la direccin de movimiento. La fuerza exterior se debe a la fuerza de
inercia debida al movimiento de S y est dirigida segn el eje x. Slo cuando el
movimiento del pndulo se encuentre en esta direccin podremos hablar de un
movimiento forzado, segn nuestro modelo terico. Y esto ocurre para ngulos
pequeos de giro. En ese caso, podemos aplicar la ley de Newton para la traslacin
del pndulo, resultando la ecuacin de movimiento
g
mx&& = m x mx&&S
L
que resolvemos como
g
&&
x + x = &&
xS = 2 cos t
L
La solucin general es suma de un movimiento de oscilacin libre (rgimen
transitorio) y un movimiento forzado (rgimen permanente), en la forma
2
x ( t ) = A cos ( 0t + ) + 2
cos t
0 2
siendo
g
0 =
L
la frecuencia de la oscilacin libre.
Para que el movimiento sea forzado exclusivamente es necesario que est
ausente el rgimen transitorio, es decir, A = 0 , condicin que equivale, como hemos
visto anteriormente, a

x0 =

2
02 2

V0 = 0

Cuando el movimiento es forzado, la amplitud de oscilacin del pndulo


(amplitud espacial, no angular) est dada por
2
C= 2
0 2
Para que sea mayor que 2 , sin especificar si el movimiento est en fase o en
desfase con el movimiento del punto de suspensin, se debe cumplir
2
>2
02 2
con la solucin
2
0 < < 2 0
3
Anlogamente, para el segundo caso, amplitud menor que , encontramos la
2
solucin
1
<
0
3

6.18 Un pndulo de masa m est unido a un dispositivo oscilador mediante un


muelle de constante k. Un extremo del muelle se fija al pndulo a una distancia
a del punto de suspensin, y el otro se fija al dispositivo. La oscilacin del
pndulo se considera horizontal (ngulo de giro pequeo). Si el dispositivo
genera un movimiento oscilatorio xd = A cos t , determinar el movimiento del
pndulo en el rgimen estacionario. Considrese que el muelle est en reposo
cuando el pndulo est en su posicin de equilibrio, y el dispositivo apagado.
Considerando slo el movimiento en direccin horizontal, aplicando la ley de
Newton para la traslacin, la ecuacin de movimiento es
g
mx&& = m x k ( xa xd )
L
siendo xa el desplazamiento horizontal del punto del pndulo donde est fijado el
muelle. Para ngulos pequeos,
a
xa = x
L
con lo cual podemos escribir
k
kA
g ka
&&
x+ +
x = xd =
cos t

m
m
L mL

En el rgimen estacionario, el movimiento del pndulo tiene la misma


frecuencia que el dispositivo, y es de la forma
x = C cos t
Introduciendo esta expresin en la ecuacin del movimiento, encontramos la
amplitud del movimiento forzado
k
m
C=A
g ka
+
2
L mL

6.19 Un muelle de constante k tiene un extremo unido a una pared y el otro a


una masa puntual m. El sistema oscila sobre un suelo horizontal sin friccin,
pero est sometido a una fuerza de rozamiento con el aire, de la forma
F = x& 2 . El sistema se perturba de la siguiente manera. Cuando la masa se
encuentra en el punto de retroceso ms alejado de la pared se le comunica
instantneamente la velocidad V0 , y esto ocurre en cada ciclo. Determinar el
valor de la velocidad comunicada para que la amplitud de oscilacin de la masa
se mantenga constante.
La amplitud del movimiento de la masa se conserva constante si en cada ciclo
la potencia media es cero. Es decir, en cada ciclo la suma de la potencia media de la
fuerza de rozamiento y la potencia media del impulso recibido debe ser cero.
Suponemos que el movimiento de la masa sigue la ley
x = A cos 0t
con lo cual, la potencia media debida a la fuerza de rozamiento es (calculada en el
tema de oscilaciones amortiguadas)
E
A202
Proz =
=
T
2
En cada impulso, la masa adquiere un momento mV0 , por lo que la fuerza
media en un ciclo, debida al impulso recibido, es
mV0
F=
T
Con esto, la potencia media debida al impulso recibido es
mV02
Pimp = FV0 =
T
y si la oscilacin mantiene una amplitud constante debe ser porque la potencia
media absorbida por el oscilador es nula
P = Proz + Pimp = 0
mV02 A2 02
=
T
2

Considerando que la frecuencia de oscilacin es igual que la frecuencia de la


oscilacin libre

k
m
obtenemos el valor de la velocidad del impulso necesario en la forma
02 =

V0 =

A2 kT
2m 2

Cuestin : Resolver este problema si el rozamiento slo se produce con el suelo


horizontal, y puede considerarse como un rozamiento seco.

7. Movimiento forzado amortiguado


Cuando existe rozamiento, slo estudiamos el rgimen permanente puesto que
el rgimen transitorio decae rpidamente. En este caso, la oscilacin del sistema
sigue teniendo la misma frecuencia que la fuerza exterior, pero se ve retrasada por la
presencia del rozamiento. En otras palabras, la cesin de energa al sistema por la
fuerza exterior no es instantnea, ya que para que el sistema pueda absorber la
energa necesaria para mantener su movimiento, la fuerza exterior debe suministrarla
en exceso, para contrarrestar la prdida por rozamiento. Por tanto, en el rgimen
permanente, la oscilacin del sistema sigue la ley
x = C cos ( t + )
Para encontrar C , introducimos este dato en la ecuacin de movimiento
d 2x
dx
F
+ 2
+ 02 x = 0 cos t
2
dt
dt
m

obteniendo
F0
cos t
m
Desarrollando las funciones trigonomtricas, e igualando sus coeficientes en cada
lado, obtenemos dos ecuaciones que al resolverse nos dan la amplitud del
movimiento
F
1
C= 0
2
m
2
02 2 + ( 2 )
C 2 cos ( t + ) 2C sen ( t + ) + 02C cos ( t + ) =

y el desfase entre el movimiento y la fuerza exterior


2
= arctan 2
02
Introduciendo, como se hizo anteriormente, el coeficiente de amortiguamiento dado
por

=
0
la amplitud del movimiento forzado amortiguado adquiere la expresin

C=

F0
m 02

1
2

2
2

1 + 2
0 0

para distintos valores

cuya grfica representamos


amortiguamiento

del

coeficiente

=0

0 .1
4

m02 C
F0

0 .2
2

0.5
0

0.5

1.5

0
El desfase con la fuerza exterior tiene la expresin

2
0
= arctan
2

1
0
y su grfica correspondiente es

p
0 .1

3
4p

0 .5

=0

0 .2

2
0 .5

4
0

0 .2

=0
0.5

0.1

1.5

de

Resonancia
De forma anloga cuando no hay rozamiento, la resonancia establece que la
respuesta del sistema al impulso exterior es mxima. Cuando existe rozamiento, la
resonancia puede ser en amplitud, cuando C alcanza su valor mximo, o en potencia,
cuando la potencia media suministrada por la fuerza exterior es mxima. En este
ltimo caso se introduce el concepto ya conocido del factor de calidad Q, que se
interpreta como el parmetro que define la transmisin efectiva de energa al
sistema. Cuanto mayor sea Q, la anchura de la resonancia en potencia ser menor
(ver despus) y la transferencia de energa ser ms eficaz.
Resonancia en amplitud
La condicin corresponde al mnimo del denominador de C. Esto es,
= 0 1 2 2
C=

F0
1
2
m 0 2 1 2

Potencia cedida al sistema


La potencia instantnea cedida por la fuerza impulsora es
P = FextV = C F0 cos t sen ( t + )
y su valor medio en un ciclo de oscilacin es
1
P = C F0 sen
2
Cuestin: Demostrar que la potencia absorbida por la fuerza de rozamiento es
igual a la potencia cedida y de signo contrario, de forma que la potencia total es
cero, y el sistema mantiene su movimiento.
Resonancia en potencia
La potencia media cedida al sistema puede escribirse tambin

F02
1
P=


3/2
m 0
2 2
2
0

+ 2
0 0

representada en la siguiente grfica


=0

14
12
10

m 0 P
F02

0.1

8
6
4

0.2
2

0.6

0.8

1.2

1.4

La frecuencia a la que la potencia media cedida al sistema se hace mxima es


1

max = 0 1 2 2 + 9 4 2 + 4 4
2

El caso que ms nos interesa corresponde a una fuerza de rozamiento dbil, cuando
el factor de calidad se hace grande. En este caso, la frecuencia del mximo es
prcticamente igual a la frecuencia de la oscilacin libre
max = 0
y dicho mximo satisface
F02
Pmax =
8m 0 2
Para caracterizar la curva de potencia de la manera ms simple posible, se toman dos
parmetros, uno de ellos es la altura o valor mximo. El otro parmetro caracteriza
la anchura de la campana que se define como la distancia entre las frecuencias a las
cuales la potencia media cedida se hace la mitad del valor mximo. Dichas
frecuencias en el caso de una fuerza de rozamiento dbil valen

= 0 1 22 / 3 1

y expresado en funcin del factor de calidad

Q= 0

obtenemos

= 0 1

Ambas frecuencias se encuentran a ambos


ausencia de rozamiento = 0 , que es

22 / 3 1

lados de la frecuencia de resonancia en


la misma en esta aproximacin a la

frecuencia de la potencia mxima. Queda claro as que la cantidad

2 2 2 / 3 1 0

Q
pueda definirse como la anchura de resonancia, o intervalo de frecuencias para las
cuales la potencia cedida al sistema es importante.
Cuestin: Demostrar que el valor mximo de la potencia es proporcional al factor
de calidad para rozamientos pequeos, y entonces, cuanto mayor sea el factor de
calidad ms picuda ser la curva de potencia cerca de la resonancia.

Tema 6. Oscilaciones de sistemas con varios


grados de libertad
Primera parte: Sistema de dos masas y un muelle
1. Ecuaciones del movimiento
Nuestro sistema est formado por dos masas, en general diferentes, m1 y m2 ,
unidas por un muelle de constante recuperadora k y longitud en reposo L0 .
Definimos sus posiciones a travs de las coordenadas x1 y x2 referidas a un sistema
de ejes fijos, no estando definidas respecto al punto de equilibrio como se realizaba
hasta ahora. Las ecuaciones del movimiento son
d 2 x1
m1 2 = k ( x1 x2 L0 ) + F1
dt
d 2 x2
m2 2 = k ( x1 x2 L0 ) + F2
dt
donde F1 , F2 son las fuerzas exteriores que actan sobre cada una de las masas.
Vamos a suponer que el campo exterior es constante, es decir, la aceleracin
producida por la fuerza exterior es la misma para cada masa
F1 F2
=
=g
m1 m2
Entonces, el movimiento del centro de masa del sistema y el movimiento relativo de
las masas quedan desligados entre s, y pueden estudiarse independientemente. El
movimiento del sistema ser la suma de los movimientos del centro de masa y
relativo.

2. Movimiento del centro de masa


La posicin del centro de masa es
m x + m2 x 2
xcm = 1 1
m1 + m2
y satisface la ecuacin del movimiento
d 2 xcm
1 d 2 x1
d 2 x2
=
m
+
m
1 2
2
m1 + m2
dt 2
dt
dt 2

1
1
( F1 + F2 ) =
( m1g + m2 g ) = g
m1 + m2
m1 + m2
Es decir, el centro de masa se mueve bajo la accin exclusiva del campo exterior. No
se ve influido por la fuerza elstica del muelle.
=

Integrando esta ecuacin obtenemos la velocidad del centro de masa


Vcm = Vcm,0 + gt
y su posicin
1
xcm = xcm,0 + Vcm,0t + gt 2
2
una vez conocidos los datos iniciales de velocidad y posicin
m x + m2 x2,0
xcm,0 = 1 1,0
m1 + m2
mV + m2V2,0
Vcm,0 = 1 1,0
m1 + m2
Como el campo exterior es conservativo, la energa del centro de masa se
conserva en el tiempo, definida por la suma de la energa cintica del centro de masa
y de su energa potencial
1
2
Ecm = ( m1 + m2 )Vcm
( m1 + m2 ) gxcm
2
tomando como referencia de la energa potencial el origen de coordenadas.

3. Movimiento relativo
La posicin relativa de las masas queda definida por la coordenada x
x = x1 x2
y satisface la ecuacin del movimiento
k
d 2 x d 2 x1 d 2 x2
k
=
2 = +
( x1 x2 L0 )
2
2
dt
dt
dt
m1 m2
k
( x L0 )

donde es la masa reducida del sistema, definida por


1
1
1
=
+
m1 m2
=

La solucin general del movimiento relativo es un MAS


x = L0 + A cos ( 0t + )
de frecuencia de oscilacin
k

amplitud A y fase inicial , que quedan definidos por las condiciones iniciales
x0 = x1,0 x2,0 = L0 + A cos
y
dx
= V1,0 V2,0 = A 0 sin
dt 0
0 =

de donde se obtiene fcilmente


A = ( x1,0 x2,0 L0 )
2

V V
+ 1,0 2,0
0

y
= arccos

x1,0 x2,0 L0
A

Como todo MAS, el movimiento relativo conserva su energa, suma de la


energa cintica relativa y energa potencial elstica
2

1 dx 1
1
2
E = + k ( x L0 ) = kA2
2 dt 2
2

4. Conclusin
Para estudiar un sistema de dos masas y un muelle en un campo exterior
constante, lo ms conveniente es dividir el movimiento en dos contribuciones que
conservan la energa por separado, el movimiento del centro de masa en el campo
exterior constante (movimiento uniformemente acelerado) y el movimiento relativo
sobre el que acta slo la fuerza elstica (MAS). Una vez hecho esto, el movimiento
general del sistema se obtiene como suma de estos dos de la forma siguiente.
Tenemos que despejar las coordenadas x1 y x2 de las definiciones de centro de masa
y posicin relativa
( m1 + m2 ) xcm = m1 x1 + m2 x2
x = x1 x2
resultando para la posicin de la masa m1
x1 = xcm +

m2
x
m1 + m2

x2 = xcm

m1
x
m1 + m2

y para la posicin de la masa m2

Por ltimo, introduciendo la dependencia temporal para xcm y x obtenemos


x1 = xcm ,0 + Vcm,0t +

1 2
m2
gt +
( L + A cos (0t + ) )
2
m1 + m2 0

x2 = xcm,0 + Vcm,0t +

1 2
m1
gt
( L + A cos (0t + ) )
2
m1 + m2 0

Problemas Resueltos
7.1

Un muelle ideal de masa despreciable, constante recuperadora k y


longitud natural L0 unido en sus extremos a dos masas puntuales m y M , se
encuentra suspendido del techo por el extremo de la masa m . En el instante
t = 0 , se suelta la masa m de forma que el sistema cae por accin de la
gravedad permaneciendo siempre el muelle en posicin vertical.
a) Escribir las ecuaciones del movimiento de cada una de las masas, la
ecuacin de movimiento del centro de masa, y la ecuacin que describe
la variacin temporal de la elongacin del muelle
b) Calcular la frecuencia de oscilacin del sistema
c) Resolver las ecuaciones para las posiciones de las masas

t=0

t >0

x2

M
M

x1

Tomamos un sistema de referencia ligado al techo, siendo x1 y x2 las


posiciones de las masas M y m respecto al techo. As, la gravedad acta en la
direccin del movimiento y se toma con signo positivo. Las ecuaciones del
movimiento son
d 2 x1
M 2 = k ( x1 x2 L0 ) + Mg
dt
d 2 x2
m 2 = k ( x1 x2 L0 ) + mg
dt
de donde obtenemos, de acuerdo con lo expuesto en la parte terica, la ecuacin para
el movimiento del centro de masa
d 2 xcm
1 d 2 x1
d 2 x2
=
+m 2
M
dt 2
m+M
dt 2
dt
1
=
( mg + Mg ) = g
m+ M
con la solucin ya conocida
1
xcm = xcm,0 + Vcm,0t + gt 2
2
una vez conocidos los datos iniciales de velocidad y posicin

xcm,0 =
Vcm,0 =

Mx1,0 + mx2,0
M +m
MV1,0 + mV2,0
M +m

La ecuacin de movimiento para la elongacin del muelle


x = x1 x2
est dada por
d 2 x d 2 x1 d 2 x2
k
k
k
=
2 = + ( x1 x2 L0 ) = ( x L0 )
2
2

dt
dt
dt
M m
donde es la masa reducida del sistema, definida por
1 1 1
=
+
M m
con la solucin conocida
x = L0 + A cos ( 0t + )
siendo 0 la frecuencia de oscilacin de las masas respecto del centro de masa, dada
por la expresin
k
k
k
0 =
=
+

M m
Adems, la amplitud A y fase inicial estn relacionadas con los datos iniciales en
la forma
A = ( x1,0 x2,0 L0 )
2

V V
+ 1,0 2,0
0

y
= arccos

x1,0 x2,0 L0
A

Las posiciones de las masas pueden expresarse en funcin de la coordenada


del centro de masa y de la coordenada de la posicin relativa, a travs de las
ecuaciones
m
x1 = xcm +
x
M +m
y
M
x2 = xcm
x
M +m
Introduciendo las soluciones temporales halladas anteriormente para xcm y x ,
obtenemos la solucin del sistema
1
m
x1 = xcm ,0 + Vcm,0t + gt 2 +
( L0 + A cos (0t + ) )
2
M +m
y
1
M
x2 = xcm,0 + Vcm,0t + gt 2
( L0 + A cos (0t + ) )
2
M +m

Slo nos falta calcular cules son las condiciones iniciales para las posiciones de las
dos masas, y el problema quedar resuelto.
Inicialmente, la masa m est en reposo unida al techo, por lo cual
x2,0 = 0
V2,0 = 0

y la masa M se encuentra en equilibrio en el otro extremo del muelle. Es decir, la


fuerza elstica debe contrarrestar inicialmente su peso:
k ( x1,0 L0 ) = Mg
con lo cual
Mg
x1,0 = L0 +
k
V1,0 = 0
De aqu, obtenemos
M
Mg
L0 +

M +m
k
=0

xcm,0 =
Vcm,0

y
Mg
k
= arccos1 = 0
A=

Por tanto, las posiciones de las masas en el instante t arbitrario estn dadas
por sus coordenadas de posicin respecto del techo
M
Mg 1 2
m
Mg

x1 =
cos 0t
L0 +
+ gt +
L0 +
M +m
k 2
M +m
k

= L0 +

g M
1 2
+ cos 0t + gt
k m
2

y
M
Mg 1 2
M
Mg

cos 0t
L0 +
+ gt
L0 +
M + m
k 2
M +m
k

g M
1
=
(1 cos0t ) + gt 2
k m
2

x2 =

Dos masas m1 = 3m / 4 y m2 = m unidas por un muelle de constante


elstica k se encuentran en equilibrio y reposo sobre un suelo horizontal. Se
lanza una masa m3 = m / 4 con velocidad V0 en la direccin del eje que une las
masas de manera que choca con m1 y se adhiere a ella. Despreciando el
rozamiento, calcular la amplitud y el perodo con que oscilan las masas despus
del choque.

7.2

En el choque se conserva el momento lineal, pero no la energa cintica. Sin


embargo, la energa cintica incidente se transforma en energa cintica del sistema
(energa cintica del centro de masa) y en energa interna (energa elstica
almacenada por el muelle). Por tanto, despus del choque, el sistema se mover con
una velocidad del centro de masa Vcm y oscilar en torno al centro de masa con una
amplitud A , debindose satisfacer la ley de conservacin del momento lineal
1
3
1

mV0 = m + m + m Vcm
4
4
4

y la ley de conservacin de la energa del sistema


11
11
3
2 1 2
mV02 = m + m + m Vcm
+ kA
24
24
4
2

Obtenemos
V
Vcm = 0
8
7 m
A=
V0
32 k
Por ltimo, de la teora general, sabemos que la frecuencia de oscilacin de un
sistema de dos masas y un muelle est dada por
k
0 =

siendo la masa reducida del sistema


1 1 1 2
= + =
m m m
con lo cual
2k
0 =
m
y el perodo del movimiento de oscilacin respecto del centro de masa es
2
2m
T=
=
0
k

Dos masas m 1 = m y m2 = 2m , unidas por un muelle de constante elstica


k pueden deslizar sin rozamiento por un plano horizontal estando la masa m2
apoyada sobre una pared vertical. Manteniendo la masa m2 apoyada contra la
pared vertical se desplaza la masa m1 hasta que el muelle se comprime una
distancia d . Si desde esta posicin se libera al sistema, calcular
a) La distancia que se ha desplazado la masa m1 cuando la masa m2 se
empieza a mover
b) La velocidad del centro de masa a partir de ese instante
c) La amplitud de oscilacin de la posicin relativa de las masas a partir de
ese instante

7.3

En el instante inicial el muelle est comprimido y la fuerza elstica creada


tiende a separar las dos masas. La masa m1 se aleja de la pared, y sobre la masa m2
acta la normal en la pared, compensando la fuerza elstica de forma que m2 se
mantenga apoyada contra la pared. Una vez que la masa m1 se mueve una distancia
d alejndose de la pared, el muelle empieza a estirarse y as, la fuerza elstica
tiende a acercar a las masas, provocando el movimiento de m2 . Por tanto, cuando
m1 se mueve una distancia d , la masa m2 empieza a moverse.
Una vez que ambas masas estn en movimiento, podemos utilizar los
resultados obtenidos para el movimiento de dos masas unidas a un muelle, sin la
presencia de un campo externo. La velocidad del centro de masa es constante, el
movimiento del centro de masa es uniforme, y la oscilacin respecto del centro de
masa se produce con una frecuencia
k
0 =

siendo la masa reducida del sistema


1 1 1
3
= +
=
m 2m 2 m

2
= m
3
Sea t0 el instante en el que la masa m2 se pone en movimiento. Esto es, es el
instante a partir del cual el movimiento del sistema puede definirse como la
combinacin del movimiento uniforme del centro de masa ms la oscilacin libre
respecto del centro de masa. Si en este tiempo, las posiciones y velocidades de las
masas respecto a un sistema ligado a la pared son respectivamente, x1,0 , x2,0 y
V1,0 ,V2,0 , entonces la velocidad del centro de masa y la amplitud de oscilacin estn
dadas por (segn lo visto en la parte terica)
mV1,0 + 2mV2,0
Vcm,0 =
3m
y

A = ( x1,0 x2,0 L0 )
2

V V
+ 1,0 2,0
0

Por tanto, para resolver el problema slo tenemos que determinar las
posiciones y velocidades de las masas en el instante t0 . Para la masa m2 es fcil ya
que se encuentra en reposo junto a la pared:
x2,0 = 0
V2,0 = 0

Para la masa m1 , debemos conocer su ley de movimiento antes de que se mueva m2 .


Antes del instante t0 , sobre m1 acta la fuerza elstica de un muelle de constante k ,
con un extremo fijo en la pared, a travs de m2 . Como m1 parte del reposo, cuando
el muelle est comprimido una distancia d , su posicin respecto de la pared antes de
t0 viene dada por
x1 = L0 d cos 1t
siendo 1 la frecuencia de oscilacin libre del muelle cuando slo acta sobre m1
1 =

k
k
=
m1
m

Su velocidad viene dada por


dx1
= 1d sin 1t
dt
De aqu, ya podemos obtener la posicin y velocidad de la masa m1 en el instante t0 ,
cuando el muelle alcanza su longitud natural, y comienza a estirarse. En ese instante
x1,0 = L0
con lo cual
cos 1t0 = 0
V1 =

sen 1t0 = 1
V1,0 = 1d

Conocidos ya los datos iniciales, la velocidad del centro de masa es


1
1 k
Vcm =
m1d =
d
3m
3 m
y la amplitud de oscilacin es

A= 1 d
0
Como se puede comprobar en este ejemplo, la energa no se conserva.
Inicialmente, la energa es igual a la energa elstica de compresin
1
Ei = kd 2
2
y cuando la masa m2 se pone en movimiento, la energa es suma de la energa
cintica del centro de masa, y la energa elstica del centro de masa

1 1 k 1 12 2 1 2 1 2 5 2
E f = 3 m
d + k
d = kd + kd = kd
2 3 m 2 02
3
2
6
El defecto de energa (cintica en este caso) es igual al trabajo de la fuerza normal
en la pared sobre el sistema, necesario para mantener en reposo a la masa m2
durante el movimiento inicial de la masa m1 .

7.4

Un muelle de constante k , al que est enganchado una masa M cuelga


verticalmente estando el sistema en equilibrio. Una partcula de masa m con
una velocidad V0 golpea desde abajo a la masa M . Determinar la amplitud del
movimiento subsiguiente despus del choque suponiendo que ste es elstico o
totalmente inelstico (las masas quedan unidas).

Cuando el choque es elstico se conserva el momento lineal y la energa. Si V


y v son las velocidades de M y m despus del choque, se satisface
1
1
1
mV02 = MV 2 + mv 2
2
2
2
mV0 = mv + MV
Resolviendo estas ecuaciones obtenemos la velocidad con que sube inicialmente la
masa M
2m
V=
V0
m+M
Ya que se produce en el punto ms bajo de la trayectoria, esta es la velocidad
mxima del movimiento de M . La energa cintica correspondiente ser la energa
cintica mxima del movimiento y debe coincidir con la energa potencial elstica
mxima (nos olvidamos de la energa potencial gravitatoria, ya que nos referimos a
desplazamientos respecto del punto de equilibrio). Por tanto, la amplitud A satisface
1 2 1
kA = MV 2
2
2

M
M 2m
A=
V=
V0
k
k M +m
Cuando el choque es totalmente inelstico, slo se conserva el momento
lineal, y la velocidad V de ambas masas despus del choque satisface
mV0 = ( m + M )V
m
V0
M +m
Sin embargo, ahora la posicin de equilibrio no es la misma antes y despus del
choque. Tomando como referencia el nuevo punto de equilibrio, la energa total
despus del choque, es decir, la energa cintica despus del choque ms la energa
potencial elstica respecto del nuevo punto de equilibrio, es igual a la energa
potencial elstica mxima. Por un lado, el punto de equilibrio se encuentra a una
distancia
V=

mg
k

del punto de equilibrio inicial

k ( l L0 ) = Mg
Entonces, la amplitud de oscilacin de ambas masas satisface en este caso
2

1
1 mg
1
( m + M ) V 2 + k = kA2
2
2 k 2

de donde obtenemos
A=

mg
kV02
1+

k
m + M )g2
(

Problema Propuesto
Dos masas m1 = m y m2 = 2m , unidas por un muelle de constante elstica
k y longitud natural L0 , se sitan de forma vertical. En el equilibrio, la masa
m2 est en contacto con el plano horizontal, y la masa m1 se encuentra en
equilibrio a una cierta altura H e . Se desplaza hacia abajo la masa m1 hasta que
se encuentra a una altura H 0 , y se abandona el sistema sin velocidad inicial.
Determinar
a) La altura inicial de equilibrio H e de la masa m1
b) El valor mximo de H 0 para que la masa m2 despegue del suelo
c) La posicin y velocidad de la masa m1 cuando despega la masa m2 ,
suponiendo que se satisface el apartado b
d) La posicin del centro de masa en cualquier instante, una vez que
despega m2

7.5

Solucin:
mg
k
4mg
b) H 0 < L0
k
2mg
z1,0 = L0 +
k

a) H e = L0

1/2

c) V1,0

2 2
9m g
k
mg

=
H

L
+
1

0
0
2

m
k
mg

2
k H 0 L0 +

1
1
1
d) zcm = z 1,0 + V1,0t gt 2
3
3
2

Segunda parte: Modos de vibracin


Objetivo: Estudiar el movimiento general de un sistema oscilatorio de varios
grados de libertad
Mtodo: Determinar los modos de vibracin del sistema. El movimiento
general ser combinacin lineal de los modos de vibracin. El movimiento particular
se calcula teniendo en cuenta las condiciones iniciales de movimiento.
Modo de vibracin: Vibracin armnica colectiva de todas las variables del
sistema, en la forma
xi = Ai cos ( t + i )

donde es la frecuencia de vibracin del sistema, y ( Ai , i ) es la amplitud y la fase


de la oscilacin de la coordenada xi . En ausencia de rozamiento, todas las variables
vibran en fase, i = .
Por tanto, un modo de vibracin est caracterizado por una frecuencia de vibracin,
y por la relacin numrica entre las amplitudes de vibracin de las coordenadas del
sistema.
Resolucin del problema
Supongamos que nuestro sistema tiene N grados de libertad. Aplicando las leyes de
Newton tenemos N ecuaciones de movimiento. Definiendo las coordenadas xi como
los desplazamientos respecto del punto de equilibrio xi = 0 , y despreciando
trminos de orden mayor que uno en xi , las ecuaciones de movimiento sern
lineales en las coordenadas, y el sistema oscilar solamente de forma armnica.
Para buscar los modos de vibracin del sistema introducimos las expresiones
xi = Ai cos ( t + ) en las ecuaciones del movimiento. Eliminando la funcin coseno
que aparece en todos los trminos, nos encontramos con un sistema de N ecuaciones
lineales para las N amplitudes de vibracin de cada modo, Ai . Segn el teorema de
Frobenius, para que exista una solucin no trivial, es decir, distinta del estado de
reposo Ai = 0 , es necesario que el determinante de la matriz asociada sea nulo. Est
condicin nos da una ecuacin caracterstica de grado N en 2 , que tiene N
soluciones , cada una de ellas correspondiendo a un modo de vibracin. Una vez
determinada la frecuencia de vibracin de cada modo, hallamos la relacin entre las
amplitudes a partir del sistema de ecuaciones lineales para las N variables Ai .

Movimiento del sistema: Un sistema de N grados de libertad tiene N modos de


vibracin independientes, y el movimiento general ser combinacin lineal de los
modos de vibracin. Ya que el sistema de ecuaciones lineales es homogneo, la
solucin para las amplitudes de vibracin queda indeterminada, en funcin de una
amplitud A genrica. Por tanto, cada modo de vibracin tiene asociadas dos
constantes ( A, ) que deben determinarse para cada movimiento particular. En total,
debemos determinar 2N constantes, dos por cada modo, a partir de las condiciones
iniciales para las coordenadas xi y sus velocidades x&i .
Coordenadas normales: La coordenada normal Qn asociada al modo normal
de frecuencia n es la combinacin lineal de las coordenadas xi que satisface el
movimiento armnico simple
&& + 2Q = 0
Q
n
n n
cuya solucin corresponde al modo de vibracin n-simo
Qn = A cos ( n t + )
Es decir, cuando el sistema oscila segn dicho modo de vibracin la oscilacin es
equivalente al movimiento armnico simple de un sistema de un grado de libertad
Qn . Otra forma de encontrar el movimiento general del sistema es buscar por
inspeccin las N coordenadas normales que corresponden a los modos de vibracin.
Entonces, invirtiendo la relacin encontramos las coordenadas del sistema xi en
funcin de las coordenadas normales Qn . En el caso de un sistema de dos grados de
libertad, cuando actan las mismas fuerzas sobre cada masa, es fcil ver que una de
las coordenadas normales corresponde a la coordenada del centro de masas, y la otra
corresponde a la coordenada relativa.

Movimiento forzado: De la misma forma que en el caso unidimensional, la


oscilacin libre del sistema, suma de los N modos de vibracin, tiene a remitir
rpidamente por efecto del rozamiento, presente siempre en la prctica. Pasado el
rgimen transitorio, el sistema ejecuta una oscilacin conjunta forzada con la misma
frecuencia que la fuerza exterior, y con la misma fase, si prescindimos del
rozamiento. La forma ms sencilla de buscar dicha oscilacin forzada es determinar
el movimiento forzado para cada una de las coordenadas normales, y de aqu,
invirtiendo la relacin, determinar el movimiento forzado de cada coordenada xi .

Problemas Resueltos
7.6

Tenemos el sistema

2m

y1
k

y2

Hallar los modos normales. Si en t = 0 , la masa 2m se desplaza una distancia d


hacia abajo, determinar el movimiento de la masa m si el sistema se abandona
sin velocidad inicial.
Tomando las coordenadas yi respecto de la posicin de equilibrio, las
ecuaciones del movimiento son
2my&&1 = ky1 k ( y1 y 2 )
my&&2 = k ( y1 y2 )

Buscamos las coordenadas normales, combinacin lineal de las coordenadas


del sistema. Ya que las fuerzas que actan son diferentes para cada masa, no
podemos esperar que las coordenadas normales correspondan al centro de masas y a
la posicin relativa. Escribimos la coordenada normal genrica en la forma
a ( 2my1 ) + b ( my2 )
Qi =
3m
La ecuacin de movimiento para esta variable es
&& = a ( 2my&&1 ) + b ( my&&2 ) = ( 2a b ) k y ( b a ) k y
Q
i
1
2
3m
3m
3m
Para que Qi sea coordenada normal su ecuacin de movimiento debe corresponder a
un movimiento armnico simple. Para ello debe satisfacerse
2a b b a
=
2a
b
2
2
Es decir, 2a = b . Eligiendo a = 1, obtenemos b = 2 .
Por tanto, la primera coordenada normal es
2 y + 2 y2
Q1 = 1
= A cos (1t + )
3

que corresponde al primer modo normal de frecuencia


2 2 k
12 =

2 m
y relacin de amplitudes
y1
2
=
y2
2
La segunda coordenada normal est dada por
2 y 2 y2
Q2 = 1
= B cos ( 2t + )
3
que corresponde al segundo modo normal de frecuencia
2+ 2 k
22 =

2 m
y relacin de amplitudes
y1
2
=
y2
2
De la relacin de amplitudes vemos que el primer modo corresponde a la oscilacin
en fase de las dos masas, mientras que el segundo modo corresponde a su oscilacin
en desfase (movimiento en sentido contrario).
Invirtiendo la relacin entre las coordenadas normales y las coordenadas del
sistema, encontramos la solucin ms general al movimiento
3
3
3
y1 = ( Q1 + Q2 ) = A cos (1t + ) + B cos ( 2t + )
4
4
4
3
3
3
y2 =
A cos (1t + )
B cos ( 2t + )
( Q1 Q2 ) =
2 2
2 2
2 2
Nuestro movimiento particular corresponde a las condiciones iniciales en
t =0,
y1 = d
y&1 = 0

y2 = 0
y&2 = 0
De aqu debemos encontrar las constantes ( A, B, , ) . Obtenemos las cuatro
ecuaciones
3
3
d = A cos + B cos
4
4
0 = A cos B cos
0 = A1 sen + B 2 sen

0 = A1 sen B 2 sen

con la solucin

= = 0
2
A= B = d
3
Por tanto, nuestro movimiento particular sigue la ley
d
y1 = ( cos (1t ) + cos ( 2t ) )
2
d
y2 =
( cos (1t ) cos ( 2t ) )
2

7.7

Sabiendo la relacin de amplitudes para los modos de vibracin del


sistema anterior, determinar las frecuencias de vibracin utilizando la
dependencia funcional de . (Este tipo de ejercicio se estudi en el tema del
movimiento armnico simple)
La dependencia funcional de la frecuencia de vibracin es
fuerza
2 =
masa desplazamiento
En el primer modo de vibracin los desplazamientos respecto del punto de equilibrio
son
y2 = 2 y1
Nos fijamos en la masa 2m. Cuando se desplaza una distancia y1 , la fuerza que
recibe se debe al estiramiento de los dos muelles. El muelle superior se estira una
cantidad y1 , y el muelle inferior se estira una cantidad y2 y1 =

2 1 y1 . Por

tanto, de la dependencia funcional obtenemos


12 =

ky1 + k

2 1 y1

2my1

k 2 2

m 2

De forma anloga, para el segundo modo de vibracin, encontramos


y2 = 2 y1
y de aqu
22 =

ky1 k

2 + 1 y1

2my1

k 2+ 2

m 2

7.8

Tres masas estn ensartadas en un aro de radio R, y ligadas entre s por


muelles de constante k. Determinar los modos normales del sistema.
Definimos las coordenadas s1 , s 2 , s3 como las distancias sobre el aro respecto
al estado de equilibrio. Las ecuaciones del movimiento para distancias s pequeas
(de forma que la fuerza ejercida por los muelles sea siempre tangente al aro), son

ms&&1 = k ( s1 s2 ) k ( s1 s3 )

ms&&2 = k ( s2 s1 ) k ( s2 s3 )
ms&&3 = k ( s3 s2 ) k ( s3 s1 )

Ya que cada masa recibe las mismas fuerzas elsticas, las coordenadas
normales correspondern al centro de masas, y a dos posiciones relativas. Para el
centro de masa, la coordenada normal es
ms + ms2 + ms3
Q1 = 1
3m
cuya ecuacin de movimiento
&& = 0
Q
1
corresponde a una rotacin uniforme, sin oscilacin, en la que los muelles
permanecen siempre en reposo, sin estirarse ni encogerse. Es el primer modo de
vibracin, movimiento uniforme del centro de masas, sobre el que no acta ninguna
fuerza exterior.
Para las posiciones relativas, podemos tomar la coordenada normal
s s
Q2 = 1 2
2
cuya ecuacin de movimiento es
&& + 3k Q = 0
Q
2
2
m
Esta coordenada normal corresponde al segundo modo de vibracin, de frecuencia
3k
2 =
m
en el que la masa de coordenada s3 se mantiene en su posicin de equilibrio,
mientras las otras dos masas oscilan en desfase.
Por ltimo, el tercer modo tiene la misma frecuencia que este ltimo modo,
debido a la degeneracin producida por la simetra del problema. Obtenemos
s s
Q3 = 1 3
2
cuya ecuacin de movimiento es
&& + 3 k Q = 0
Q
3
3
m
Esta coordenada normal corresponde al tercer modo de vibracin, de frecuencia
3k
3 =
, en el que la masa de coordenada s 2 se mantiene en su posicin de
m
equilibrio, mientras que las otras dos masas oscilan en desfase.

7.9 Determinar los modos normales del sistema formado por una barra de
masa m y longitud L, sujeta al techo por sus extremos mediante dos muelles de
constante k.

m
Tenemos dos grados de libertad, el desplazamiento vertical y del centro de
masas respecto de su posicin de equilibrio, y el ngulo de giro de la barra
respecto de su centro de masa, tomando = 0 como el ngulo cuando la barra se
encuentra en posicin horizontal. Obviamente, el movimiento del centro de masa y
el movimiento de rotacin deben estar desacoplados, y por tanto, corresponden a las
coordenadas normales del sistema.

Para el movimiento del centro de masa, la coordenada normal es


Q1 = y
Ya que el centro de masa est sometido a la fuerza elstica de los dos muelles de
constante k, la ecuacin de movimiento
my&& = 2ky
movimiento armnico simple de frecuencia 1 = 2k

Para el movimiento de rotacin respecto del centro de masa, la coordenada


normal que representa la distancia vertical respecto de la posicin de equilibrio
= 0 , para ngulos pequeos, es
L
Q2 =
2
y su ecuacin de movimiento corresponde a la rotacin de una barra respecto de su
centro de masa, debido al par de fuerzas generado por la fuerza elstica de los
muelles respecto al centro de la barra
1
L L
mL 2&& = 2k
12
2 2
que es un movimiento armnico simple
&& + 6 k Q = 0
Q
2
2
m
de frecuencia 2 = 6k .
m

Finalmente, los desplazamientos verticales de los muelles se obtienen a partir


de las coordenadas normales
L
y1 = y + = A cos (1 t + ) + Bcos ( 2 t + )
2
L
y1 = y = A cos (1 t + ) Bcos ( 2 t + )
2
Observando esta expresin deducimos que, respecto de las coordenadas de los
muelles, el primer modo de vibracin corresponde a la vibracin en fase de los
muelles, mientras que el segundo modo corresponde a la vibracin en desfase de los
dos muelles.

7.10 Tenemos el sistema de masas

x1

x2

x3

que simula el comportamiento de una molcula triatmica. Determinar los


modos normales del sistema, y sus coordenadas normales.
Sean x1, x2 , x3 los desplazamientos respecto del equilibrio. Las ecuaciones del
movimiento son
Mx&&1 = k ( x1 x2 )
mx&&2 = k ( x1 x2 ) k ( x2 x3 )

Mx&&3 = k ( x2 x3 )
Introduciendo las dos frecuencias de vibracin
k
m2 =
m
k
M2 =
M
obtenemos
&&
x1 + M2 ( x1 x2 ) = 0

&&
x2 + m2 ( 2x2 x1 x3 ) = 0
&&
x3 + M2 ( x3 x2 ) = 0

Buscamos los modos normales en la forma


x1 = A cos ( t + )

x2 = B cos ( t + )
x3 = C cos ( t + )

Introduciendo estas funciones en las ecuaciones del movimiento, eliminando las


funciones coseno, obtenemos el sistema de ecuaciones lineales para las amplitudes
A, B, C en forma matricial
M2 2

2
m

A 0

m2 B = 0

M2 2 C 0

M2

2 m2 2
M2

Para la solucin no trivial, la relacin entre las amplitudes es


A
2
= 2 M 2
B M
C
M2
=
B M2 2
donde las frecuencias de los modos normales son solucin de la ecuacin
caracterstica (determinante de la matriz igual a cero),

2
M

) ( 2
2

2
m

2 2 m2 M2 M2 2 = 0

Desarrollando esta ltima ecuacin, obtenemos


2 2 2 m2 M2 M2 2 = 0

)(

De aqu, ya podemos describir los tres modos de vibracin:


Modo 1

12 = 0

A= B =C

Es el modo de traslacin uniforme con velocidad constante, ya que en este caso,


&&
x1 = &&x2 = &&x3 = 0 . Es el movimiento del centro de masa, en ausencia de fuerzas
exteriores. La coordenada normal correspondiente es
x1

Q1 = ( A B C) x2 = A ( x1 + x2 + x3 )
x
3
Modo 2

2 = M

A = C

B=0

En este modo la masa central no se mueve, y las masas exteriores oscilan en desfase,
por la accin de cada muelle exterior. La frecuencia corresponde a la frecuencia de
oscilacin de una masa M bajo la accin de un muelle de constante k.

La coordenada normal es
x1

Q2 = ( A B C) x2 = A ( x1 x3 )
x
3

Modo 3

A= C

32 = 2 m2 + M2

B=

2M
A
m

Las masas exteriores oscilan en fase entre s, y en desfase con la masa central.
La coordenada normal es
x1
2M


Q3 = ( A B C) x2 = A x1
x2 + x3
m

x
3

7.11 Sobre el muelle del lado izquierdo acta una fuerza exterior, de manera
que ste se mueve segn x = x0 cos t . Determinar los desplazamientos de las
masas en el rgimen permanente.
x = x0 cos t

x1

x2

Las ecuaciones del movimiento son


mx&&1 = k ( x1 x2 ) k ( x1 x )
mx&&2 = k ( x1 x2 ) kx2
Introduciendo la frecuencia de oscilacin libre
k
m2 =
m
podemos escribir
&&
x1 + m2 ( 2 x1 x2 ) = m2 x0 cos t
&&
x2 + m2 ( 2 x2 x1 ) = 0

Ya que las fuerzas que actan sobre ambas masas son las mismas, en ausencia
de la fuerza exterior, las coordenadas normales para la oscilacin libre del sistema
corresponden al movimiento del centro de masas, y al movimiento relativo. Para el
centro de masas,
x +x
Q1 = 1 2
2

cuya ecuacin de movimiento es


&& + 2 Q = 1 2 x cos t
Q
1
m 1
m 0
2
y para el movimiento relativo la coordenada normal es
Q2 = x1 x2
cuya ecuacin de movimiento es
&& + 3 2 Q = 2 x cos t
Q
1
m 1
m 0

En el rgimen permanente la oscilacin libre del sistema es despreciable, y


slo se mantiene la oscilacin forzada de la misma frecuencia que el movimiento
exterior. Por tanto, esperamos la solucin a los dos modos de vibracin en la forma
Q1 = C1 cos t
Q2 = C 2 cos t
Introduciendo estas expresiones en las ecuaciones de movimiento correspondientes,
obtenemos las amplitudes de las oscilaciones forzadas
1
2
C1 = x0 2 m 2
2 m
C2 = x0

m2
3 m2 2

Por ltimo, invirtiendo la relacin entre las coordenadas normales y las


coordenadas del sistema, encontramos los desplazamientos de las masas en el
rgimen permanente

1
1 m2
m2
x1 = Q1 + Q2 = x0 2
+
cos t
2
2 m 2 3 m2 2

1
1 2
2
x2 = Q1 Q2 = x0 2 m 2 2 m 2 cos t
2
2 m 3 m

Tema 7. Vibraciones en sistemas continuos.


Anlisis de Fourier
Primera parte: Oscilaciones transversales
1. Oscilaciones acopladas en una cuerda con masas
Consideramos una cuerda ligera que soporta N masas puntuales iguales m
espaciadas entre s una distancia a , a lo largo de su longitud. Los dos extremos de la
cuerda estn fijos, su longitud total es L = ( N + 1) a , y est sometida a una tensin T
constante en todos sus puntos.
Dejando que el sistema evolucione libremente, las masas ejecutan
oscilaciones armnicas en la direccin y perpendicular a la posicin de la cuerda en
equilibrio. El problema es determinar las frecuencias de los modos normales de
vibracin, y los desplazamientos de las masas en cada modo particular. Suponemos
que la tensin que sufre la cuerda es T independientemente de la posicin de las
masas.

1
yn

yn1

yn+1

a
La ecuacin de movimiento en la direccin y para la masa m situada a la
distancia na de uno de los extremos es
d 2y
m 2n = T ( sen1 + sen 2 )
dt
y yn 1 yn y n+1
= T n
+

a
a

que escribimos en la forma


d 2 yn
= 02 ( 2 y n yn 1 y n+1 )
2
dt
donde 0 =

T
es una frecuencia de oscilacin libre caracterstica del sistema.
ma

En un modo de vibracin determinado las masas oscilan armnicamente con


la misma frecuencia por lo que podemos suponer
yn = An cos t

Introduciendo esta descripcin del movimiento en la ecuacin anterior, obtenemos


las relaciones entre las amplitudes de vibracin para masas vecinas, en la forma
An 2 = 02 ( 2 An An1 An+1 )
vlido para n = 1,.., N , junto con las condiciones frontera que aseguran que los
extremos se mantienen fijos en cualquier tiempo
A0 = 0
AN +1 = 0

Para resolver este sistema de ecuaciones, en vista de las condiciones en los


extremos, tomamos las amplitudes en la forma
An = C sen ( nka )
siendo k el llamado vector de onda del modo de vibracin dado, que tenemos que
determinar utilizando las condiciones frontera. Hallamos
sen ( ( N + 1) ka ) = 0
cuya solucin es el conjunto de N valores
s 1
ks =
( s = 1,.., N )
N +1 a
Cada valor corresponde a un modo de vibracin determinado, cuya relacin
de amplitudes est dada por
ns

Ans = C sen ( nk s a ) = C sen

N +1
cuyos signos relativos dependen exclusivamente del valor de la funcin seno.
Tenemos por tanto, N modos de vibracin en cada cual hemos hallado la relacin
entre las N amplitudes de las masas puntuales.
Nos queda por determinar la frecuencia angular de cada modo de vibracin,
que depender como hemos visto de la variable s que caracteriza los distintos
modos. De la ecuacin
Ans s2 = 02 ( 2 Ans An 1, s An+1, s )
para n = 1 , obtenemos

s
s

sen
s2 = 02 2sen
sen 2

N +1
N +1
N +1

y de aqu,

s2 = 2 02 1 cos

N +1

Ejemplo. N = 1
En este caso, slo existe una frecuencia de vibracin

T
1
2 = 2 02 1 cos = 2
ma
2

que corresponde a la frecuencia de oscilacin libre de una masa ensartada en el


punto medio de una cuerda tensa con extremos fijos.
Ejemplo. N = 2
En este caso, las frecuencias de los modos de vibracin satisfacen

1
12 = 2 02 1 cos = 02
3

2
22 = 2 02 1 cos = 302
3

Para el primer modo, la relacin de amplitudes es


3
1
A11 = C sen ( k1a ) = C sen =
C
2
3
3
2
A21 = C sen ( 2k1a ) = C sen =
C
2
3
con lo cual, A11 = A21 , ambas masas oscilan en fase. El movimiento asociado a este
modo de vibracin corresponde al movimiento del centro de masas. Corresponde a
la vibracin libre de un sistema con una masa neta 2m colocada en el punto medio
de una cuerda tensa, con extremos fijos.
Para el segundo modo, la relacin de amplitudes es
3
2
A12 = C sen ( k2 a ) = C sen =
C
2
3
3
4
A22 = C sen ( 2k 2a ) = C sen = C
2
3
con lo cual, A11 = A21 , ambas masas oscilan en desfase. Este modo de vibracin
corresponde al movimiento relativo.

2. Lmite continuo. Ecuacin de ondas


Queremos determinar en qu se transforman estas oscilaciones acopladas en el
lmite continuo, cuando la distancia a entre las masas se hace muy pequea, as
como la propia magnitud m de cada masa, pero manteniendo la densidad de masa
= m constante. Es decir, a partir de un conjunto de masas puntuales queremos
a
construir una cuerda continua con masa. Como coordenada del sistema tomamos la
distancia horizontal de un punto de la cuerda al extremo izquierdo, con lo cual
x = na
x dx = ( n 1) a
dx = a
m = dx

En este lmite, la ecuacin de movimiento del sistema se transforma en


2 y (x , t )
y ( x, t ) y ( x dx , t ) y (x , t ) y ( x + dx, t )
dx
= T
+

2
t
dx
dx

Utilizando el concepto de derivada la expresin entre parntesis resulta ser


2 y
y y
= 2 dx
x x
x x + dx
x x
y la ecuacin de movimiento se transforma en la ecuacin de ondas
2 y T 2 y
=
t 2 x 2
El cociente T

tiene dimensiones del cuadrado de una velocidad, la

velocidad de propagacin de la onda en la direccin x. En cada punto x, la


oscilacin segn el eje y se realiza de forma armnica, y es la forma de la oscilacin
la que se propaga en la direccin del eje x, con velocidad T .

Solucin general de la ecuacin de ondas


La solucin general puede escribirse como la suma de dos funciones arbitrarias
y (x , t ) = f1 ( x + Vt ) + f 2 ( x Vt )
La primera funcin representa una onda viajera que se propaga con velocidad V
hacia la izquierda, y la segunda funcin representa otra onda viajera que se propaga
hacia la derecha con la misma velocidad. En general, la velocidad de propagacin de
una onda es la velocidad a la que vara la fase de la onda. Con esto, para
determinarla basta la condicin
d
( x Vt ) = 0
dt
y obtenemos las velocidades
dx
= V
para la onda con fase ( x + Vt )
dt
dx
=V
para la onda con fase ( x Vt )
dt
As, la primera fase se propaga hacia la izquierda y la segunda fase hacia la derecha.
La naturaleza de las funciones f1 , f 2 es arbitraria. Pueden ser funciones
sinusoidales o pueden describir pulsos de ondas. De hecho, las dos funciones
siempre pueden escogerse de forma que su suma represente cualquier estado inicial
de desplazamiento y ( x ,0 ) y velocidad y& ( x ,0 ) , siendo y& ( x, t ) la velocidad de
oscilacin vertical del punto de la cuerda situado en la coordenada x.

3. Soluciones armnicas simples


La solucin ms general es de la forma
y (x , t ) = A sen ( kx t ) + B cos ( kx t )

+ C sen ( kx + t ) + D cos ( kx + t )

para una oscilacin de frecuencia en el tiempo y de frecuencia k en el espacio.


Los perodos de oscilacin estn dados por
2
T=

2
=
k
El primero corresponde a la oscilacin vista en el tiempo t, y el segundo, la longitud
de onda, corresponde a la oscilacin vista en el espacio x.
La relacin entre la frecuencia de oscilacin temporal y la frecuencia de
oscilacin espacial se llama relacin de dispersin
= (k )
En el caso de la cuerda vibrante, la relacin de dispersin es
= Vk
T

La relacin de dispersin determina la dispersin de la energa entre los distintos


modos de vibracin, definidos por el vector de onda k. Si la relacin de dispersin es
lineal en k, como ocurre en la cuerda tensa, todos los modos se propagan con la
misma velocidad, y la energa mecnica de la onda se propaga de forma homognea.
Se trata de un medio no dispersivo. Si la relacin no es lineal, la velocidad de
distintos modos es diferente y la energa no se propaga homogneamente, sino que
se divide en paquetes de energa cada uno propagndose con la velocidad del modo
de vibracin asociado. La energa se ha visto dispersada y el medio se llama
dispersivo.
V=

4. Ondas estacionarias (cuerda con dos extremos fijos)


Consideramos una cuerda de longitud L con dos extremos fijos en x = 0, L .
Las condiciones de contorno
y ( 0, t ) = y ( L, t ) = 0
para cualquier tiempo exigen que la solucin general de la oscilacin vertical de la
cuerda sea de la forma
y (x , t ) = ( A sen t + B cos t ) sen kx
Dicha solucin determina una onda estacionaria, ya que la velocidad de
propagacin se hace cero. Tampoco existe propagacin de energa mecnica. Para

determinar los modos de vibracin, utilizamos la condicin de contorno en el


extremo derecho y ( L, t ) = 0 , que exige que
sen kL = 0
La solucin es un conjunto de valores caracterizados por un ndice n. Se habla as de
los modos armnicos de vibracin
k n L = n
( n = 1,2,.. )
Segn la relacin de dispersin anterior, a cada modo de vibracin le corresponde
una frecuencia angular de oscilacin en el tiempo dada por
V
n = Vk n = n
L
Por tanto, la oscilacin de la cuerda vibrante con dos extremos fijos segn un
modo definido de vibracin se caracteriza por una periodicidad en el tiempo con
perodo
2 2L 1
Tn =
=
n V n
y una periodicidad en el espacio con longitud de onda
2 2L
n =
=
kn
n
En resumen, el modo n-simo de vibracin tiene la solucin
nVt
nVt
n x

yn ( x, t ) = A n sen
sen
+ Bn cos

L
L
L

La mejor forma de comprender la oscilacin estacionaria de la cuerda en cada


modo particular es estudiar el comportamiento espacial de la oscilacin, ya que, al
ser la onda estacionaria, se conserva su forma espacial con una amplitud
( A sen t + B cos t ) en cada punto dependiente del tiempo.
a) De la dependencia de la longitud de onda en n, obtenemos la relacin
n
L = , lo que significa que en el modo n-simo la cuerda en su longitud L
2
contiene un nmero entero de semilongitudes de onda.
b) Existen puntos en los que la vibracin mantiene valores caractersticos.
Tenemos por un lado los nodos de vibracin, puntos donde no existe vibracin en
ningn momento. Corresponden a la condicin
sen k n xnodo = 0
y la solucin es el conjunto de valores
m m
xnodo =
= L ( m = 0,.., n )
kn
n
Vemos as que en el modo n-simo existen n + 1 nodos de vibracin, dos de los
cuales coinciden con los puntos extremos de la cuerda, x = 0, x = L . La separacin
espacial entre los nodos de vibracin es igual a una semilongitud de onda

xnodo =

m +1
m
L
L L= = n
n
n
n 2

c) Otros puntos caractersticos son los vientres de vibracin, puntos en los


cuales la vibracin transversal se hace mxima en cada instante de tiempo. Es
importante notar que el valor mximo de esta vibracin no tiene el mismo valor
siempre, sino que para cada tiempo tenemos un valor mximo. Lo caracterstico es
que en los vientres se produce siempre dicho mximo. Corresponde a la condicin
de mximo en x de la curva y ( x, t )
y
=0
x
cos k n xvientre = 0
y la solucin es el conjunto de valores
( 2m + 1) = 2m + 1 L
xvientre =
kn
2
2n

( m = 0,.., n 1)

Tenemos por tanto, n vientres de vibracin cuya separacin espacial tambin es


igual a una semilongitud de onda
2m + 3
2 m +1
L
xvientre =
L
L= = n
2n
2n
n 2
y la separacin espacial entre un nodo y un vientre sucesivo, y viceversa, es igual a
un cuarto de longitud de onda.
d) Por ltimo, existe una sucesin armnica en las frecuencias de vibracin e
la cuerda vibrante con dos extremos fijos. Tomando como referencia los datos del
primer modo, o modo fundamental, n = 1 ,

1 = 2 L
V
1 =
2L

obtenemos la relacin armnica para las frecuencias naturales


1
Tn = T1
n
1
n = = n1
Tn
Una vez estudiados con detalle los distintos modos de vibracin,
determinamos la forma ms general de vibracin de la cuerda vibrante. Esto
constituye el llamado anlisis de Fourier, que bsicamente describe la solucin
general como la superposicin lineal de distintos modos de vibracin, en la forma
n Vt
n Vt
n x

y (x , t ) =
yn ( x, t ) =
+ Bn cos
A n sen
sen
L
L
L

El objetivo del anlisis de Fourier es determinar los coeficientes An , Bn en funcin de


los perfiles iniciales de desplazamiento y ( x ,0 ) y velocidad y& ( x ,0 ) . Para ello,
utilizamos las propiedades de las funciones, que forman un conjunto completo
ortogonal en la recta 0 x L , con la relacin de ortogonalidad
0 si m n
L
n x
m x
dx =
sen
sen

L
0
L
L
2 si m = n

De la condicin inicial,
y ( x ,0 ) =

Bn sen

n x
L

multiplicando ambos miembros por la funcin sen


0 x L , obtenemos

m x
, e integrando en la recta
L

n x
L
dx = Bn
0
L
2
con lo cual, el coeficiente Bn est dado por
n x
2 L
Bn =
y ( x,0 ) sen
dx
L 0
L

y ( x ,0 ) sen

De forma anloga, de la condicin inicial para la velocidad


n V
n x
y& ( x ,0 ) =
A n sen
L
L

obtendramos el coeficiente An en la forma


L
n x
2
An =
y& ( x ,0 ) sen
dx
n V 0
L

5. Energa mecnica en una cuerda vibrante


Una cuerda posee tanto energa cintica, debida a su movimiento como
energa potencial debida a su deformacin. La energa cintica de un elemento de
longitud dx y masa dx es
1
y
dEc = dx
2
t
y la energa cintica total de la cuerda vibrante ser

L y
1
dx
2 0 t
y depender del tiempo. La energa potencial del elemento de longitud dx es igual al
trabajo realizado por la tensin de la cuerda para conseguir que dicho elemento
adquiera la longitud deformada

Ec =

ds =

( dx )

+ ( dy )

Es decir,

dE p = T ( ds dx )
Si el elemento de longitud se toma lo suficientemente pequeo, podemos aproximar
la deformacin por su valor en primer orden en dx
1 y 2
2
2
ds dx = ( dx ) + ( dy ) dx 1 + dx dx
2 x

1 y
= dx
2 x
Suponiendo que la tensin de la cuerda sea constante, cierto si el grado de
deformacin de la cuerda es pequeo, la energa potencial de la cuerda est dada por

1
Ep = T
2

y
x dx

En el caso de que la cuerda vibrante oscile segn el modo estacionario nsimo de vibracin, la energa total tiene el valor, utilizando las frmulas anteriores
1
En = L n2 An2 + Bn2
4
2
2
donde An + Bn representa el cuadrado de la mxima amplitud de vibracin en ese
modo determinado. Cuando el movimiento de la cuerda vibrante sea combinacin
lineal de distintos modos de vibracin, la energa total ser suma de las energas de
dichos modos de vibracin

E=

En

6. Ejemplo adicional: cuerda con un extremo libre


De forma resumida, establecemos las caractersticas de las ondas estacionarias
producidas en este caso. La condicin en el extremo fijo es la misma que en el caso
anterior. En el extremo libre, la tensin de la cuerda debe ser la misma que cuando
se encuentra en reposo, ya que hemos supuesto que la cuerda no es flexible en la
direccin de su longitud, y slo puede oscilar de forma perpendicular a su longitud.
Como adems la deformacin es pequea de manera que la tensin de la cuerda
puede suponerse constante, la posicin del extremo libre debe coincidir tanto en
reposo como en movimiento. Esto es, el extremo libre debe estar dispuesto de forma
paralela al estado de reposo, y = 0. La condicin correspondiente es
y
= 0 en x = L
x
Desarrollando los pasos que se hicieron en el caso de la cuerda vibrante con dos
extremos fijos, en este modelo obtenemos
La solucin para el modo n-simo de vibracin

n + 1 Vt
n + 1 Vt
n+ 1 x
2
2
2

yn ( x, t ) = A n sen
+ Bn cos
sen

L
L
L

donde n = 0,1,...

Las frecuencias de oscilacin espacial y temporal para el modo n-simo


n + 12
kn =
L
n + 1 2 V
n =
L

La relacin entre la longitud de la cuerda y la longitud de onda


2n + 1
L=
n
4
La posicin de los n + 1 nodos de vibracin
m
2m
xnodo =
=
L ( m = 0,.., n )
kn
2n + 1
y la posicin de los n + 1 vientres de vibracin
2m + 1
xvientre =
L ( m = 0,.., n )
2n + 1
De nuevo la separacin entre nodos y vientres sucesivos es igual a una cuarto de
longitud de onda.
El anlisis de Fourier es similar al caso de dos extremos fijos, resultando las
frmulas finales
2
Bn =
L

n + 1 ) x
(
2
y ( x, 0) sen
dx

2
An =
(n + 1 )V
2

(n + 1 ) x
2
y& ( x ,0 ) sen
dx
L

Como se observa en todas las formulas anteriores, basta el cambio de ndice


n n + 1 , para obtener a partir de las soluciones para dos extremos fijos, las
2
soluciones para un extremo libre. Es como si desplazramos las soluciones con dos

extremos fijos, desde el extremo derecho una distancia n .


4

7. Pulsos de forma constante


Otra solucin posible en el movimiento de oscilacin de una cuerda vibrante
es la propagacin de un pulso de forma constante, de longitud espacial finita, en la
forma
y (x , t ) = f ( x Vt )
si por ejemplo el pulso viaja hacia la derecha con velocidad V. Lo que caracteriza
un pulso es su velocidad de oscilacin, ya que generalmente no tiene una estructura
espacial peridica. Ya que la funcin f no cambia en el tiempo, dicha velocidad es
y f ( x Vt )
=
=
f ' (x Vt ) = Vf ' ( x Vt )
t t
t
y es proporcional a la pendiente de la curva en el punto y el instante en cuestin, y
de signo contrario. Esto quiere decir que cuando la pendiente del pulso es positiva,
(creciente), la velocidad de oscilacin es negativa. Los puntos de la cuerda se dirigen
hacia la posicin de equilibrio, y = 0 . Y del mismo modo, cuando la pendiente es
negativa (decreciente) los puntos se alejan de la posicin de equilibrio. As es como
se consigue que el pulso se propague a lo largo de la cuerda con velocidad V.

Problemas Resueltos
7.2

Una cuerda uniforme de 2.5 metros de longitud y 0.01 kg de masa se


somete a una tensin de 10 N. Calcular la frecuencia de su modo fundamental.
Si se pulsa transversalmente la cuerda y luego se toca en un punto a 0.5 metros
de su extremo, determinar las frecuencias de vibracin que persisten en el
movimiento.
El modo fundamental corresponde a un nmero de onda
k1 L =

k1 =
L
Ya que la propagacin de las ondas en una cuerda vibrante no es dispersiva, la
velocidad de propagacin no depende del nmero de onda y vale

T
TL
V= =
=
k

M
siendo M la masa de la cuerda y L su longitud. De esta relacin, obtenemos la
frecuencia natural ( = 2 ) del modo fundamental
1 =

1
k
1 T
=V 1 =
= 10 Hz
2
2 2 ML

Cuando se pulsa transversalmente la cuerda, el movimiento ser combinacin


lineal de los modos de vibracin, cada uno con una amplitud y fases determinadas
mediante el anlisis de Fourier a partir de los perfiles iniciales de velocidad y
desplazamiento. Pero si a continuacin tocamos la cuerda en un punto generamos un
nodo en dicho punto, y los modos que persistirn sern aquellos compatibles con la

presencia de un nodo de vibracin en esa posicin. En nuestro caso, los modos que
persisten son los que tienen un nodo en x = 0,5 metros . Para el modo n-simo la
posicin de sus nodos es
m
xnodo = L
n
m = 0,.., n
Introduciendo los datos de la longitud y de la posicin del nodo, el modo n-simo
persiste si para algn entero m se satisface
m 0,5 1
=
=
n 2,5 5
n = 5m
Es decir, el ndice n debe ser mltiplo de 5. Con esto, las frecuencias que persisten
son
n = n 1 = 5m1 = 50, 100, 150,.. Hz

7.2

Una cuerda estirada de masa m, longitud L y tensin T se ve impulsada


por dos fuentes, una en cada extremo. Ambas fuentes tienen la misma
frecuencia y la misma amplitud de vibracin pero estn desfasadas 180
entre s. Determinar el valor ms pequeo posible de consistente con la
vibracin estacionaria de la cuerda.
La perturbacin de la forma A cos t que se produce en el extremo izquierdo,
que tomamos como x = 0 , se propaga hacia la derecha con velocidad V = TL
m
por lo que la perturbacin generada en la posicin x de la cuerda estar dada por la
funcin A cos ( kx t ) siendo k = V el vector de onda. De la misma forma, en el

extremo derecho, x = L , la perturbacin generada A cos ( t + ) = A cos t se


propaga hacia la izquierda con velocidad V, y en la posicin x la perturbacin ser
de la forma A cos ( k ( L x ) t ) = A cos ( kx + t kL ) .
Por tanto, la perturbacin total de la cuerda en la posicin x debido a la presencia
de las dos fuentes es
y (x , t ) = A cos ( kx t ) cos ( kx + t kL )

con la forma de una onda estacionaria slo cuando kL sea mltiplo de . La


frecuencia mnima necesaria para generar tal onda estacionaria ser igual entonces a
la frecuencia del primer modo de vibracin de una cuerda con dos extremos fijos.
En este caso,
V
V
T
= = =
k
L
mL

7.3 Hallar el movimiento general de una cuerda tensa de longitud L, con dos
extremos fijos, con las condiciones iniciales
y ( x ,0 ) = Ax ( L x)
y
=0
t t = 0

Segn el anlisis de Fourier, el movimiento general est dado por


n x
y (x , t ) =
( An sin n t + Bn cos nt ) sen
L

Los coeficientes de este desarrollo en serie estn dados por las integrales
2
Bn =
L

y ( x, 0) sen

2
An =
L n

n x
dx
L

y& ( x ,0 ) sen

n x
dx
L

En nuestro caso, ya que la velocidad inicial es cero, todos los coeficientes An


son cero. Para evaluar los coeficientes Bn , es til considerar la simetra del
problema. El perfil inicial del desplazamiento y ( x ,0 ) es una funcin par respecto
del punto medio de la cuerda, x = L . Esto indica que los coeficientes Bn sern
2
n x
que
distintos de cero slo para aquellos valores n asociados a las funciones sen
L
sean pares respecto del punto medio de la cuerda. Es decir, aquellos valores que
satisfagan
n ( L x )
n x
n x
n x
sen
= sen
= sen ( n ) cos
cos ( n ) sen
L
L
L
L
La solucin es n impar. Por tanto, B2k = 0 . Desarrollando la integral obtenemos
B2k +1

2A
=
L

= 2 AL

x ( L x )sen

( 2k + 1) x
L

dx

u(1 u)sen ( ( 2k + 1) u )du

2 AL2

( 2k + 1)

y de aqu, la solucin general es

y (x , t ) =

(
k =0

2k +1 =

4 AL2
2k + 1)

( 2k + 1)
L

sen
3

( 2k + 1) x cos
L

2k +1t

7.4 Una cuerda tensa con dos extremos fijos se desplaza desde su posicin
central una distancia h segn la direccin vertical, y se suelta sin velocidad
inicial. Determinar la energa mecnica de la oscilacin, y su perodo temporal
de oscilacin.
Ya que inicialmente la cuerda est en reposo, la energa mecnica ser igual a la
energa potencial elstica que adquiere la cuerda en su deformacin inicial. De la
teora, sabemos que la energa potencial tiene la expresin en este caso
1
E = E p (t = 0 ) = T
2

y
dx
x t = 0

y
es igual a la pendiente de la curva inicial. Para el tramo que va desde
x
x = 0 hasta x = L , dicha pendiente es
2
y
h
2h
=
=
L
x L
2
y para el segundo tramo, desde x = L hasta x = L , la pendiente tendra el mismo
2
valor con signo contrario. Por tanto,

La funcin

4h 2
y
=

2
x t = 0 L
y la energa mecnica de la onda ser
1
4h2
h2
E = TL 2 = 2T
2
L
L

7.5

Una cuerda de longitud L, y masa M cuelga verticalmente de un extremo


fijo en el techo. La tensin que sufre la cuerda se debe exclusivamente a su
peso. Determinar la tensin de la cuerda y la velocidad de propagacin de las
ondas en la cuerda. En t=0, se produce un pulso en el extremo inferior que viaja
hacia el extremo superior. Determinar el tiempo que tarda el pulso en volver al
extremo inferior si se ve reflejado en el extremo superior.
La tensin sufrida por un elemento de cuerda situado a una distancia x del techo
es igual al peso que debe soportar, correspondiente a un trozo de cuerda de longitud
L x . Por tanto, la tensin de la cuerda es funcin de la distancia x en la forma
T ( x) = g ( L x )
y la velocidad de propagacin de los pulsos de onda sobre la cuerda es
T ( x)
= g ( L x)

El pulso tarda el mismo tiempo es subir que en bajar la cuerda. Por tanto, el
tiempo de viaje sera
V (x) =

t =2

dx
1
=2
V ( x)
g

1
=
Lx

L
g

7.6 Resolver de nuevo este problema si ahora la cuerda se fija por un


extremo a un eje que gira sobre s mismo con velocidad angular . Por efecto
del giro, la cuerda se coloca de forma horizontal, de manera que el peso de la
cuerda puede despreciarse.
En este caso, la tensin de la cuerda a una distancia r del eje de giro debe
soportar la fuerza centrfuga generada por el giro del trozo de cuerda de longitud
L r . Tenemos
T (r) =

dm x =
2

2 xdx =

y la velocidad de propagacin de los pulsos sera


1 2 2 2
V (r) =
L r
2

1
2 L2 r 2
2

El tiempo que tarda el pulso en recorrer dos veces la longitud de la cuerda es


t =2

dr
2 2
=
V (r )

dr
L2 r 2

7.7

Determinar cmo se refleja un pulso sobre el extremo fijo de una cuerda


y sobre el extremo libre de una cuerda.
En la reflexin un pulso no cambia de forma pero si de sentido de propagacin.
Supongamos que el pulso que llega al punto de reflexin tiene la forma f ( x Vt ) .
Entonces, tras la reflexin la combinacin del pulso incidente y del pulso emergente
ser de la forma
y (x , t ) = f ( x Vt ) f ( x + Vt )
Nuestro objetivo es decidir que signo debemos tomar en cada caso. En un extremo
fijo, la posicin de la cuerda corresponde a y = 0 , y esto es posible slo si en la
frmula anterior tomamos el signo menos. Corresponde entonces al hecho de que el
pulso reflejado invierte su posicin respecto al pulso incidente. Si el incidente llega
por arriba el reflejado sale por debajo.
Para un extremo libre, la condicin es y = 0 . Utilizando la ecuacin anterior
t
obtenemos
y
= Vf ' ( x Vt ) Vf ' ( x + Vt )
t
y en el extremo libre debemos tomar entonces el signo ms. El pulso reflejado no
invierte su posicin respecto del pulso incidente.

Segunda parte: Ondas de sonido


La sensacin que llamamos sonido se produce por las variaciones en la presin
del aire, que detectamos por su efecto mecnico sobre nuestro sistema auditivo. Las
vibraciones de los cuerpos se transmiten, en forma de ondas que se propagan en el
aire, al tmpano y despus al odo interno donde se encuentran las terminaciones del
nervio auditivo.
Los sonidos se distinguen por tres caractersticas: la altura, o gravedad del
sonido, vinculada a la frecuencia de las vibraciones, la intensidad, cualidad de un
sonido fuerte o dbil, vinculada a la amplitud de las vibraciones, y timbre cualidad
que distingue los sonidos emitidos por instrumentos diferentes, y que tiene que ver
con la complejidad del sonido, superposicin de armnicos superpuestos.

La capacidad auditiva humana se extiende en el rango de frecuencias entre


20 Hz y 20 kHz , aunque la sensitividad decrece substancialmente por debajo de
100 Hz o por encima de 10 kHz . Este rango de frecuencias est obviamente
relacionada con el habla humana, ya que la energa emitida se concentra
principalmente entre 100 Hz y 10 kHz , con los sonidos vocales concentrados en el
rango 300 Hz-3 kHz , y los sonidos de las consonantes principalmente por encima de
1 kHz . Los sonidos musicales han evolucionado para estimular el sentido auditivo
sobre todo su rango, pero la mayora se encuentran en el rango 100 Hz-3 kHz .

1. Ondas sonoras
Las ondas se propagan en cualquier medio que tenga masa y elasticidad. Los
materiales slidos, que tienen elasticidad bajo tensin y bajo compresin, permiten
la propagacin de ondas de tensin (transversales) y de compresin (longitudinales).
Los fluidos, y en particular los gases como el aire, debido a su viscosidad, (friccin
entre elementos vecinos del fluido) slo tienen un comportamiento elstico cuando
son comprimidos, no cuando sufren tensin. Debido a esto, slo las ondas
longitudinales pueden propagarse en un fluido, con el movimiento de las partculas
del fluido en la misma direccin que la direccin de propagacin de la propia onda.
Cuando se generan ondas sonoras por una fuente puntual, se extienden en
todas direcciones. En la forma ms simple pueden verse como ondas planas, con
frentes de onda en forma de planos normales a la direccin de propagacin.
Suponiendo que dichos planos puedan extenderse hasta el infinito, el problema de la
propagacin de la onda slo implica una coordenada x que mida la distancia
recorrida por la onda en la direccin de propagacin.
Supongamos que ( x) es el desplazamiento del aire durante el paso de una
onda sonora. Tomamos un elemento de volumen de aire de espesor dx, localizado
entre dos planos de rea S perpendiculares a la direccin de propagacin, situados en
las coordenadas x y x + dx . El volumen antes de que pase la onda sonora es
V = Sdx .

dx

( x)

( x + dx )

Al pasar la onda sonora, el plano izquierdo se desplaza una distancia ( x ) , y el


plano derecho se desplaza una distancia

( x + dx ) ( x ) +
dx
x
Por tanto, el incremento de volumen de este elemento ser

dV = S ( ( x + dx ) ( x ) ) S
dx
x
Ahora estudiamos el efecto de este incremento de volumen sobre la presin
del aire. Debemos recordar que el aire, siendo un gas, se encuentra en equilibrio
antes de que pase la onda sonora. Esto significa que su volumen y su presin estn
relacionados por una ecuacin de estado de equilibrio. Para estudiar los
movimientos que no se producen en equilibrio, nos fijamos en la compresibilidad
del gas
dp
K = V
dV
donde dp es la variacin de la presin del aire respecto de su valor en el equilibrio.
En general, K es positiva demostrando que bajo una compresin ( dV < 0 ) , la

presin del aire aumenta, y bajo una dilatacin ( dV > 0 ) , la presin del aire
disminuye. El efecto mecnico de estos cambios de volumen supone la existencia de
una fuerza restauradora, que tiende a llevar al aire a su estado de equilibrio. Cuando
el volumen disminuye, la presin aumenta, generando una fuerza que tiende a
expandir el gas sobre sus alrededores, y cuando el volumen aumenta, la presin
disminuye, permitiendo que los alrededores ejerzan una fuerza sobre el gas que lo
comprime. Por tanto, hemos constatado la existencia de la fuerza restauradora
generada por el incremento de presin dp. Dicho incremento depende del
incremento de volumen, y as del desplazamiento del aire en cada punto

S
dx
dV

dp = K
= K x
= K
V
Sdx
x
La fuerza generada por el desplazamiento del aire en el elemento seleccionado
sobre sus alrededores es igual a la variacin de la presin por el rea normal S. Sobre
el plano izquierdo, la fuerza es

( x )
x

Fizdo = Sdp = SK

y sobre el plano derecho,


Fdcho = Sdp = SK

( x + dx )
x

SK

( x )
x

+ SK

2 ( x )
x 2

dx

Finalmente, aplicando la ecuacin de Newton, encontramos la ecuacin de


movimiento para la masa de aire que se encuentra encerrada en nuestro elemento en
cuestin. Escribimos
m ( x ) a ( x ) = Fizdo + Fdcho
siendo m ( x ) la masa de aire del elemento

m ( x ) = Sdx

Aqu, es la densidad del aire en su estado de equilibrio. La aceleracin del


movimiento del aire ser la segunda variacin temporal de su desplazamiento
respecto del equilibrio
2
a ( x) = 2
t
Con esto, obtenemos
2 K 2
=
t 2 x 2
Por tanto, la ecuacin de movimiento resulta ser la ecuacin de ondas,
suponiendo que la compresibilidad no depende de x, es decir, si el aire puede
considerarse un medio homogneo. Adems, la ecuacin de evolucin de la
variacin de presin tambin corresponde a la ecuacin de ondas
2 ( dp ) K 2 ( dp )
=
x 2
t 2
y hablamos en general de la propagacin de dos ondas, una onda de desplazamiento
y una onda de presin. La velocidad de propagacin de las ondas sonoras es
K

teniendo el valor aproximado c 340 m/s, a temperatura ambiente.


csonido =

Para determinar la compresibilidad del gas debemos estudiar el


comportamiento del aire bajo las compresiones y expansiones sucesivas. En general,
la temperatura aumenta en las zonas de compresin y disminuye en las zonas de
expansin. Pero la velocidad de la onda sonora es tan grande que no hay tiempo
suficiente para que la temperatura se equilibre entre las regiones de expansin y
compresin. Es decir, la propagacin de la onda sonora se realiza de forma
adiabtica, sin transferencia de energa trmica. En tal caso, de la ecuacin de
equilibrio de una gas bajo una transformacin adiabtica

pV = cte
dp
dV
+
=0
p
V

donde

Cp

es el cociente de calores especficos, obtenemos la


CV
compresibilidad del aire
dp
K = V
= p
dV
siendo p la presin en el equilibrio. Por tanto, la velocidad del sonido
p
pV
nRT
RT
=
=
=

m
m
M
es proporcional a la raz cuadrada de la temperatura del gas. Aqu, M es la masa de
un mol de gas.
csonido =

2. Modos normales. Ondas estacionarias


De la misma forma que las ondas que se generan sobre una cuerda tensa
pueden ser ondas estacionarias, las ondas sonoras que se generan en el interior de
cavidades tambin pueden adquirir la forma estacionaria. Estudiamos el caso de las
ondas sonoras en el interior de un tubo finito de longitud L con sus extremos
abiertos o cerrados.
Determinamos primero las condiciones frontera que deben satisfacer las ondas
de desplazamiento y de presin. Un extremo cerrado no permite el paso del aire y en
l, el aire ejerce su fuerza mxima. Un extremo abierto corresponde a una variacin
de presin nula, y a un mximo en el desplazamiento (en el extremo abierto, la
fuerza que se opone al paso de la onda sonora es nula, por estar equilibradas las
presiones a ambos lados).
a) Con un extremo abierto
Por las condiciones de contorno, dp es mxima en el extremo cerrado, x = 0 , y nula
en el extremo abierto, x = L . El desplazamiento es nulo en el extremo cerrado y es
mximo en el extremo abierto. La forma ms simple de onda estacionaria que
satisface dichas condiciones es
dp = A cos kx cos t
= B sen kx cos t
De la dependencia en x, vemos que la onda de presin est desfasada un cuarto de
longitud de onda respecto de la onda de desplazamiento. Esto quiere decir que
cuando la variacin de presin es mxima el desplazamiento es nulo, y que cuando
la variacin de presin es nula, el desplazamiento es mximo.
Cada modo de vibracin est representado por un valor de k, dentro de todos los
valores posibles compatibles con la condicin
cos kL = 0
Es decir,


2
con n = 0,1,.. Para el modo n-simo, la longitud de onda es
2
4
n =
=
L
kn 2n + 1
lo que equivale a decir que en el modo n-simo, en la longitud L del tubo sonoro las
ondas sonoras describen 2 n + 1 cuartos de longitud de onda

L = ( 2 n + 1) n
4
La frecuencia de los modos de vibracin es
2n + 1 csonido
c
n = sonido =
4
L
n
Para las ondas de desplazamiento los ( n + 1) nodos estn en los puntos
k n L = ( 2n + 1)

n
2
para m = 0,.., n , y los ( n + 1) vientres en los puntos
xnodo = m

n n
+
2
4
Para las ondas de presin, los nodos son los vientres del desplazamiento, y los
vientres son los nodos del desplazamiento.
xvientre = m

b) Con dos extremos abiertos


Por las condiciones de contorno, dp es nula en x = 0 , y en x = L . El desplazamiento
es mximo en ambos extremos. La forma ms simple de onda estacionaria que
satisface dichas condiciones es
dp = A sen kx cos t
= B cos kx cos t
De nuevo, la onda de presin est desfasada un cuarto de longitud de onda respecto
de la onda de desplazamiento.
Cada modo de vibracin est representado por un valor de k, dentro de todos los
valores posibles compatibles con la condicin
sen kL = 0
Es decir,
k n L = n
con n = 1,2,.. Para el modo n-simo, la longitud de onda es
2 2L
n =
=
kn
n
lo que equivale a decir que en el modo n-simo, en la longitud L del tubo sonoro las
ondas sonoras describen n semilongitudes de onda

L=n n
2
La frecuencia de los modos de vibracin es

csonido
c
= n sonido
n
2L
y la sucesin de modos es armnica. Para las ondas de desplazamiento los n nodos
estn en los puntos

xnodo = m n + n
2
4
para m = 0,.., n 1, y los ( n + 1) vientres en los puntos
n =

xvientre = m

n
2

donde ahora m = 0,.., n .


Para las ondas de presin, los nodos son los vientres del desplazamiento, y los
vientres son los nodos del desplazamiento.

Problemas Resueltos
7.8

Una habitacin tiene dos paredes opuestas que estn alicatadas. Las
paredes restantes, el suelo y el techo estn recubiertas de un material
absorbente del sonido. La frecuencia ms baja para la cual es acsticamente
resonante la habitacin es 50 Hz . Se produce un ruido complejo en la
habitacin que slo excita a los dos modos de vibracin inferiores, de tal forma
que cada modo tiene su mxima amplitud en t = 0 .
a) Hacer un esquema del desplazamiento para cada modo separadamente
1
1
en funcin de x para los tiempos t = 0,
,
seg.
200 100
b) Se observa que el desplazamiento mximo de las partculas del polvo
en el aire (que no se producen necesariamente en el mismo instante en la misma
posicin) en diversos puntos entre las paredes es el siguiente:
x

max

+ 10

+ 10

3L

10

Determinar las amplitudes de vibracin de cada uno de los modos por


separado.
El sonido que se produce en la habitacin genera ondas estacionarias (modos
de vibracin) slo cuando se propaga en la direccin horizontal x, puesto que las
paredes restantes al estar recubiertas de material absorbente, no reflejan el sonido
que les llega. Los modos de vibracin para el desplazamiento corresponden a las
ondas estacionarias que se anulan en dos extremos, localizados en cada una de las
paredes alicatadas. Para ello, el vector de onda debe satisfacer
sen kL = 0
siendo L la distancia entre las dos paredes alicatadas. Por tanto,

n
L
y el conjunto de soluciones para los modos de vibracin de las ondas de sonido tiene
la expresin general
n x
n ( x , t ) = An sen
cos ( nt + n )
L
siendo n la frecuencia del modo de vibracin,
c
n = k nc sonido = n sonido
L
kn =

Segn los datos del problema, la frecuencia ms baja del modo de vibracin
es de 50 Hz. Obtenemos la relacin
c
1 = 2 1 = sonido = 100 ( rad / s )
L
con lo cual, la longitud entre las paredes debe ser
c
L = sonido 3.4 m
2 1
Se genera un ruido complejo combinacin de los dos modos de vibracin
inferiores, ( n = 1,2 ) , de forma que en t = 0 su amplitud sea igual a su valor
mximo, An . Esto ocurre si la fase inicial se toma igual a cero, n = 0 . Por tanto, el
ruido generado tiene la expresin matemtica para el desplazamiento del aire
x
2 x
( x, t ) = A1 sen
cos 1t + A2 sen
cos 2t
L
L
x
2 x
= A1 sen
cos (100 t ) + A2 sen
cos ( 200 t )
L
L
Las grficas de los desplazamientos para cada modo por separado y para los tiempos
considerados son

t =0

1 ( x ,0 ) = A1 sen

x
L

2 ( x, 0 ) = A2 sen

2 x
L

1 (x, 0)

A1

0.5

x
0.2

-0.5

-1

0.4

2 ( x,0 )

0.6

A2

0.8

t=1

200

2 ( x, t ) = A2 sen

1 ( x, t ) = 0

seg

2 x
L

0.5

1 ( x, t )
0.2

0.4

0.6

2 ( x, t )

-0.5

0.8

A2

-1

1 ( x, t ) = A1 sen

t=1
seg
100

x
L

2 ( x, t ) = A2 sen

2 x
L

2 ( x, t )
0.5

A2

x
0.2

0.4

0.6

0.8

-0.5

1 (x, t )
-1

A1

Evaluamos ahora el desplazamiento del aire, (y por tanto, el desplazamiento


de las partculas de polvo que flotan libremente en l) en los puntos indicados.
Tenemos

L
, t = A1 sen cos 1t + A2 sen cos 2t
4
2
4

L
, t = A1 sen cos 1t + A2 sen cos 2t
2
2
3
3
3L
, t = A1 sen
cos 1t + A2 sen
cos 2t
4
2
4

y determinando sus valores mximo, (que se producen a distintos tiempos), las


amplitudes de vibracin deben satisfacer las ecuaciones

2
L
, t = +10 = A1 sen + A2 sen =
A1 + A2
4
2
2
4 max
L
, t = +10 = A1
2 max
3
3
2
3L
, t = 10 = A1 sen
+ A2 sen
=
A1 A2
4
2
2
4 max

con la solucin
A1 = 10

2
A2 = 10 1

7.9

Calcular la energa total de vibracin de una cuerda con dos extremos


fijos si est vibrando en la siguiente superposicin de modos normales
x
3 x

y (x , t ) = A1 sen
cos 1t + A2 sen
cos 3t
4
L
L

La energa total es suma de la energa cintica de la cuerda debida a su


movimiento y la energa potencial debida a su deformacin. La energa cintica es

1
=
2

1
Ec =
2

y
dx
t
2

x
3 x

sen 1t A23 sen


sen 3t dx
A11 sen
4
L
L

1
= A1212 sen 2 1t
2

x
1

sen
dx + A3212 sen 2 3t
L
2
4

+ A1 A213 sen 1t sen 3t


4

sen

sen 2

3 x
dx
L

x
3 x
sen
dx
L
L

n x

Utilizando el hecho que las funciones base sen


son ortogonales en el
L n

intervalo 0 x L , y que

cuerda vibrante
Ec =

sen 2

n x
L
dx = , obtenemos la energa cintica de la
L
2

1
1

LA1212 sen 2 1t + LA3212 sen 2 3t


4
4
4

Por otro lado, la energa potencial de la cuerda est dada por


1
Ep = T
2

y
x dx


x

3
3 x

A1 L cos L cos 1t + A2 L 3 cos L cos 3t 4 dx

0
n x

De nuevo, utilizando el hecho de que las funciones base cos


son
L n

1
= T
2

ortogonales en el intervalo 0 x L , y que

cos 2

n x
L
dx = , obtenemos la
L
2

energa potencial de la cuerda vibrante


2
2
1
1

2
2
2 9
E p = TLA1 2 cos 1t + TLA3 2 cos 2 3t
4
4
4
L
L

Por ltimo, teniendo en cuenta la relacin entre la tensin de la cuerda y la


frecuencia de los modos de vibracin
T
L2
n = kn
T = n2 2 2

n
obtenemos la energa potencial en la forma
1
1

E p = LA1212 cos 2 1t + LA32 32 cos2 3t


4
4
4

Sumando las dos contribuciones, vemos que la energa total de la cuerda es


constante, e igual a la suma de las energas totales de cada modo por separado
1
1
E = LA1212 + LA32 32 = E1 + E3
4
4

7.10 Un altavoz puede considerarse como un tubo sonoro de seccin

transversal variable S ( x ) , donde x es la coordenada en la direccin de


propagacin de las ondas de sonido. Determinar la ecuacin que gobierna la
distribucin de presin dentro del altavoz, suponiendo que las ondas sonoras
sean de tipo estacionario. Generamos un pulso de voz en el extremo del altavoz.
Definir las condiciones para la variacin de presin en dicho extremo y estudiar
como vara la fuerza transmitida por la onda sonora en funcin de S ( x ) ,
determinando cul sera la forma del altavoz ms efectivo.

Cuando la seccin transversal depende de la coordenada x, al determinar la


fuerza que acta sobre el elemento de volumen (ver teora) encontramos que

Fizdo = S ( x ) dp = K S

x x

Fdcho = S ( x + dx ) dp = K S

x x + dx
con lo cual la fuerza neta sera


S
dx
x x x
y la ecuacin de movimiento nos llevara a la ecuacin de ondas modificada para el
perfil de desplazamiento
1 2 1
=
S

2
c sonido
t 2 S x x
F = Fizdo + Fdcho K

De manera anloga, el perfil de la variacin de presin satisface


2
1 ( dp ) 1 ( dp )
=
S

2
c sonido
t 2
S x
x
y si las ondas sonoras son de tipo estacionario, dp debe ser de la forma
dp = P ( x ) cos t
donde la funcin P ( x ) satisface

2
2
csonido

P=

1 d dP
S

S dx dx

Introduciendo como es habitual el vector de onda k del modo de vibracin


estacionario

k=
csonido
podemos escribir esta ltima ecuacin en la forma
d dP
S
+ k 2 PS = 0

dx dx
Ahora generamos un pulso de voz en el extremo del altavoz, generando una
variacin de presin constante en el aire que se encuentra en el extremo del altavoz.
Las condiciones para la variacin de presin sern
P = P0
dP
=0
dx
siendo P0 la fuerza por unidad de rea que realizamos al generar el pulso de voz. Si
S0 es el rea transversal en el extremo del altavoz, la velocidad V del flujo de aire
emitido sera
V 2 S0 = P0
P0
S0
(Si dicha velocidad fuese mayor que la velocidad del sonido, se producira una onda
de choque, fenmeno que se conoce como la ruptura del sonido, y que aparece
habitualmente en los vuelos supersnicos)
V=

La fuerza transmitida por la onda sonora a una distancia x del extremo del
altavoz es F ( x ) = S ( x ) P ( x ) . Nuestro altavoz ser el ms eficaz cuando las
prdidas por difusin del sonido sean despreciables, en cuyo caso la fuerza de la
onda sonora debe mantenerse constante en el interior del altavoz. Obtenemos la
condicin adicional
F ( x ) = S ( x ) P ( x ) = S 0 P0
con lo cual, para encontrar S ( x ) , P ( x ) , debemos resolver las dos ecuaciones
d dP 2
S
+ k P0 S0 = 0
dx dx
dP
dS
S
+P
=0
dx
dx

Eliminando la funcin P( x ) , obtenemos


d 1 dS
= k2

dx S dx
que se integra directamente, resultando
1

S = exp k 2 x 2 + Bx + C
2

Las condiciones que exigimos en el extremo del altavoz, S ( 0) = S0 , dS


nos determinan el valor de las constantes de integracin
B=0
exp C = S0
Por tanto, nuestro altavoz ptimo ser aquel que tenga una seccin
1
S ( x ) = S0 exp k 2 x 2
2

S0

6
4
2

kx
0.5

-2
-4
-6

1.5

dx (

0) = 0 ,

Como datos tpicos, = 200 Hz , con lo cual

kx =

2
x
csonido

csonido
340 ms -1
x=
kx =
kx = 27 kx cm
2
400s -1
7.11 Un lazo cerrado de una cuerda uniforme se hace girar rpidamente a

una velocidad angular constante . La masa de la cuerda es M y el radio es R.


Como resultado de la rotacin la cuerda sufre una tensin T. Determinar el
valor de dicha tensin. Se deforma la cuerda en un punto, haciendo que
aparezca una protuberancia. Determinar bajo qu condicin la distorsin de la
cuerda permanece estacionaria respecto a un sistema de ejes fijo, cuyo origen se
site en el centro del lazo.
Cuando la tensin de la cuerda equilibra las fuerzas centrfugas generadas por
la rotacin, la cuerda tensa rota manteniendo el radio del lazo constante. Desde un
punto de vista energtico, la conservacin de la energa mecnica del sistema nos
dice que el incremento de energa cintica que adquiere la cuerda debe ser igual a la
energa potencial que gana al realizar la tensin de la cuerda un trabajo para
deformarla (extendiendo el radio del lazo).
Al tratarse de un cuerpo continuo, si el radio del lazo aumenta desde el valor
inicial R al valor final R + dR , el trabajo producido en la deformacin es,
suponiendo que la tensin es constante a lo largo de toda la cuerda,
E p = T ( 2 ( R + dR ) 2 R ) = 2 dR T
Esta suposicin siempre es cierta si la deformacin puede suponerse pequea,
dR << R .
En el movimiento de rotacin, todos los elementos de la cuerda estn a la
misma distancia del centro y se mueven con la misma velocidad lineal. El
incremento de energa cintica de la cuerda al deformarse es
1
1
2
Ec =
2 ( R + dR ) dm
2 R 2 dm
2
2
1
= M 2 2R
dR + dR 2
2
Por tanto, la conservacin de la energa mecnica nos lleva a
1
M 2 2 RdR + dR 2 = 2 T dR
2

Cuando la deformacin de la cuerda es pequea, dR << R , podemos


despreciar el trmino en ( dR ) , obteniendo as el valor de la tensin de la cuerda
2

T=

1
M 2 R
2

La velocidad de propagacin, a lo largo de la cuerda, de la perturbacin que se


produce en un punto determinado corresponde a la velocidad de propagacin de las
ondas en una cuerda vibrante
T
2 RT
V=
=

M
la densidad de la cuerda, masa por unidad de longitud. Teniendo
siendo = M
2 R
en cuenta el valor de la tensin, la velocidad de propagacin de la perturbacin
producida es
T
M 2R 2
=
= R

M
y coincide con la velocidad lineal de desplazamiento de los puntos materiales de la
cuerda. Esto quiere decir que si la perturbacin viaja por la cuerda en direccin
contraria al sentido de giro, entonces, respecto a un sistema fijo de ejes, tal
perturbacin siempre permanecera estacionaria.
V=

7.12 Dos pulsos triangulares simtricos de altura 0,4 metros y longitud total
1,0 metros se generan en los dos extremos fijos de una cuerda vibrante, de 4
metros de longitud. Ambos pulsos viajan con la velocidad de onda de 1 m/seg, al
encuentro uno de otro. En el instante inicial estn en la posicin indicada en la
figura. Determinar la disposicin de la cuerda en los tiempos t=1 seg, t=1,5 seg,
t=2 seg.
En el instante inicial, la disposicin de la cuerda es

y
f1 ( x )
0

f 2 (x )

combinacin de dos pulsos, que se expresan matemticamente mediante las


funciones de x
4

x
si
0 < x < 0.5

5
f1 ( x ) =

0.8 4 x

si
0.5 < x < 1

si
3 < x < 3.5
5 ( x 3)

f2 ( x ) =

0.8 + 4 ( x 3)

si 3.5 < x < 4

A partir del instante inicial, el pulso superior se propaga con una velocidad de
1 m/seg, hacia el extremo derecho, y el pulso inferior se propaga con la misma
velocidad hacia el extremo izquierdo. Ya que los pulsos no cambian de forma, su
posicin en cualquier instante del tiempo se obtiene directamente de la funcin que
define su posicin inicial con la sustitucin x x Vt . Obtenemos
4

si
Vt < x < 0.5 + Vt
( x Vt )

5
f1 ( x Vt ) =

0.8 4 ( x Vt )

si
0.5 + Vt < x < 1 + Vt

5
4

si
3 Vt < x < 3.5 Vt

( x + Vt 3)

5
f 2 ( x + Vt ) =

0.8 + 4 ( x + Vt 3)

si 3. 5 Vt < x < 4 Vt

y el desplazamiento de la cuerda en la posicin x en el instante t es directamente


y (x , t ) = f1 ( x Vt ) + f2 ( x + Vt )
En t = 1 seg , la disposicin de la cuerda es
y ( x ,1) = f1 ( x 1) + f 2 ( x + 1)
4

si
1 < x < 1.5
( x 1)

0.8 4 ( x 1)
si 1.5 < x < 2

5
=

4 ( x + 1 3)
si
2 < x < 2.5
5

4
0.8 + ( x + 1 3)
si 2.5 < x < 3

con la grfica

y
f1 ( x 1)
0

2 f 2 (x + 1)

Para t = 1.5 seg , de forma anloga, obtenemos


y ( x ,1.5) = f1 ( x 1.5) + f 2 ( x + 1.5)
4
4

x
1.5

(
)
( x + 1.5 3)

5
5
=
0.8 4 ( x 1.5) 0.8 + 4 ( x + 1.5 3)

5
5
con la grfica,

1.5 < x < 2

si

2 < x < 2.5

si

y
1

x
y por ltimo, para t = 2 seg , los dos pulsos se han cruzado y la cuerda toma la
disposicin

y
f1( x2)
0

f2( x+2) 2

Tema 8. Ondas progresivas


1. Solucin general de la ecuacin de ondas
La solucin general puede escribirse como la suma de dos funciones
arbitrarias
y (x , t ) = f1 ( x + Vt ) + f 2 ( x Vt )
La primera funcin representa una onda progresiva que se propaga con velocidad V
hacia la izquierda, y la segunda funcin representa otra onda progresiva que se
propaga hacia la derecha con la misma velocidad. En general, la velocidad de
propagacin de una onda es la velocidad a la que vara la fase de la onda. Con esto,
para determinarla basta la condicin
d
( x Vt ) = 0
dt
y obtenemos las velocidades
dx
= V
para la onda con fase ( x + Vt )
dt
dx
=V
para la onda con fase ( x Vt )
dt
As, la primera fase se propaga hacia la izquierda y la segunda fase hacia la
derecha. La naturaleza de las funciones f1 , f 2 es arbitraria. Pueden ser funciones
sinusoidales o pueden describir pulsos de ondas. De hecho, las dos funciones
siempre pueden escogerse de forma que su suma represente cualquier estado inicial
de desplazamiento y ( x ,0 ) y velocidad y& ( x ,0 ) , siendo y& ( x, t ) la velocidad de
oscilacin vertical del punto del medio material que soporta la onda situado en la
coordenada x.

2. Soluciones armnicas simples


La solucin ms general es de la forma
y (x , t ) = A sen ( kx t ) + B cos ( kx t )

+ C sen ( kx + t ) + D cos ( kx + t )
para una oscilacin de frecuencia en el tiempo y de frecuencia k en el espacio.
Los perodos de oscilacin estn dados por
2
T=

2
=
k
El primero corresponde a la oscilacin vista en el tiempo t, y el segundo, la longitud
de onda, corresponde a la oscilacin vista en el espacio x.

La relacin entre la frecuencia de oscilacin temporal y la frecuencia de


oscilacin espacial se llama relacin de dispersin

= (k )

En el caso de la cuerda vibrante, la relacin de dispersin es

= Vk
V=

La relacin de dispersin determina la dispersin de la energa entre los distintos


modos de vibracin, definidos por el vector de onda k. Si la relacin de dispersin es
lineal en k, como ocurre en la cuerda tensa, todos los modos se propagan con la
misma velocidad, y la energa mecnica de la onda se propaga de forma homognea.
Se trata de un medio no dispersivo. Si la relacin no es lineal, la velocidad de
distintos modos es diferente y la energa no se propaga homogneamente, sino que
se divide en paquetes de energa cada uno propagndose con la velocidad del modo
de vibracin asociado. La energa se ha visto dispersada y el medio se llama
dispersivo.

3. Velocidad de fase, dispersin


Nos limitamos al caso de ondas progresivas de tipo armnico, con una
frecuencia angular bien definida. La velocidad de fase V = k es la velocidad a la
que se propaga la fase de la onda, el contorno de la onda. Slo puede hablarse de
esta velocidad cuando la funcin de onda tenga la misma forma a lo largo de toda su
longitud. Si la onda cambia de forma en funcin del tiempo o de la distancia a lo
largo de su direccin de propagacin, la medida de la velocidad de fase no dara
siempre el mismo resultado. Cuando V = V (k ) en un medio dado, se dice que el
medio es dispersivo y que la onda presenta dispersin.

4. Propagacin de un grupo de ondas,


velocidad de grupo
Nuestro objetivo ahora es averiguar si existe alguna definicin de velocidad
de propagacin de ondas que sea til cuando no se cumplan las condiciones que nos
permiten definir la velocidad de fase de forma unvoca.

a) Superposicin de dos ondas armnicas

Estudiamos por simplicidad la propagacin de dos ondas armnicas en la


misma direccin con la misma amplitud pero con frecuencias angulares y vectores
de onda ligeramente diferentes. Las conclusiones que se pueden obtener de este
modelo simple son aplicables a la propagacin de cualquier grupo de ondas.
La onda que se propaga en nuestro medio es la superposicin de dos ondas
armnicas
y (x , t ) = A cos ( kx t ) + A cos ( ( k + dk ) x ( + d ) t )

donde los diferenciales se toman como cantidades pequeas respecto del valor de las
variables k , . De la relacin trigonomtrica suma de dos funciones coseno,
obtenemos
d
dk
y (x , t ) = 2 A cos
x
t cos ( kx t )
2
2
con la aproximacin
dk << k
d <<
Al representar dicha onda compuesta debemos fijarnos en la frecuencia
espacial y temporal de los dos factores. En el primer factor dichas frecuencias son
diferenciales respecto de los valores correspondientes al segundo factor. Esto quiere
decir que el primer factor tiene una variacin mucho ms lenta que el segundo. De
otra forma, el primer factor acta de envolvente del movimiento de frecuencia
mayor del segundo factor. Hablamos entonces de una pulsacin de amplitud de
variacin suave
d
dk
A ( x , t ) = 2 A cos
x
t
2
2
frente a la variacin ms rpida de la fase armnica
cos ( kx t )
Representamos grficamente esta superposicin de ondas en el caso concreto
dk = k , en el instante inicial t = 0 . Esto nos dar suficiente informacin sobre la
10
propagacin de la onda compuesta. Tenemos entonces
y ( x ,0 ) = A ( x ,0 ) cos kx
k
A ( x ,0 ) = 2 A cos x
20

y( x,0 )

k
cos x
20

2A
0.5

kx
20
-0.5
-1

k
cos x
20

40

60

80

100

120

Se ve claramente cmo la oscilacin se produce en el interior de la envolvente


generada por la amplitud A( x ,0) . Se habla as de la amplitud modulada del grupo de
ondas.
En cuanto a la propagacin de la onda, tenemos dos velocidades bien
definidas. Una de ellas es la velocidad de fase de la onda armnica cos ( kx t ) ,
cuyo valor coincide con la velocidad de fase de las ondas primitivas, V = . La
k
otra velocidad, la llamada velocidad de grupo, est asociada con la velocidad de
variacin no de la fase de la onda sino de la amplitud de la onda. Como la amplitud
de la onda tambin tiene una dependencia armnica en sus argumentos, la velocidad
de grupo es directamente
d
2 = d
Vg =
dk
dk
2
Para encontrar el significado fsico de la velocidad de grupo, basta recordar la
relacin entre la energa mecnica de una onda y la amplitud de su movimiento: la
energa es proporcional a la amplitud de movimiento al cuadrado. Por tanto, la
velocidad de grupo adems de ser la velocidad con que vara la amplitud modulada
de la onda compuesta es la velocidad con que se propaga la energa transportada por
la onda. De esta forma es un concepto ms fundamental que la velocidad de fase. En
ausencia de superposicin de ondas, la velocidad de grupo carece de sentido.
Adems, las dos velocidades tienen el mismo valor cuando el medio no es
dispersivo.

b) Caso general
Cuando un medio pueda transportar ondas progresivas con la relacin de
dispersin = ( k ) , la velocidad de grupo es
Vg ( k ) =

y la velocidad de fase es

d ( k )
dk

(k)
k
siendo k el vector de onda que define la propagacin espacial de la onda o grupo de
ondas (define la frecuencia espacial, y la direccin de propagacin).
V (k) =

Problemas Resueltos
8.1

Determinar la velocidad de grupo y de fase de las ondas que se propagan


en una cuerda discreta, con vector de onda k.
Segn vimos en el tema anterior, la cuerda discreta tiene las siguientes
frecuencias espacial y temporal para cada modo de vibracin
s 1
ks =
( s = 1,.., N )
N +1 a

s2 = 2 02 1 cos

N +1

con lo cual, la relacin de dispersin para la propagacin de ondas es


2 = 202 (1 cos ( ka ) )
Por tanto, la velocidad de grupo sera
d ( k ) 02 k sen ka 02 sen ka
=
=
Vg ( k ) =

dk
V (k)
siendo la velocidad de fase
V (k) =

2 02 (1 cos ka )

k
Cuando sen ka = 0 , la velocidad de grupo y de fase son nulas, mostrando que para
esos valores de k no existe la propagacin de la energa ni de la fase armnica. Las
nicas ondas que pueden recorrer la cuerda discreta son las ondas estacionarias,
correspondientes a los valores ka = n .

8.2

El movimiento de la onda de longitud de onda corta en el agua est


controlado por la tensin superficial. La velocidad de fase de estas ondas viene
dada por
2 S
V ( ) =

donde es la densidad del agua, S la tensin superficial y la longitud de


onda. Determinar la velocidad de grupo para un grupo de ondas con longitudes
de onda prximas a , y explicar cmo se observa el movimiento de este grupo
de ondas en la superficie del agua.
Podemos utilizar la siguiente relacin entre la velocidad de grupo y la
velocidad de fase
d d
dV
Vg =
= ( kV ) = V + k
dk dk
dk
kS
Como k = 2 , tenemos V ( k ) =
y de aqu, la velocidad de grupo es

1
3
Vg = V + V = V
2
2

En el movimiento de un grupo de ondas sobre la superficie del agua, las ondas


individuales se mueven con velocidad V mientras que el grupo de ondas lo hace con
velocidad 3 V . Esto quiere decir que el grupo est adelantando constantemente a
2
las ondas individuales, es decir, se observa como se producen ondas individuales
que el grupo absorbe al cabo de un cierto tiempo. Es el fenmeno observado en mar
abierto.

8.3

La relacin de dispersin en un cierto medio se indica en la siguiente


figura. Explicar de forma cualitativa los valores relativos de las velocidades de
grupo y de fase, en el intervalo representado.

k
Para valores pequeos de k, la relacin de dispersin es lineal
= Vk
con lo cual, la velocidad de grupo es igual a la velocidad de fase, y el medio se
comporta como un medio no dispersivo.
Para valores un poco mayores de k, la recta se va inclinando hacia el eje k,
indicando que la potencia en k es menor que uno. Supongamos una relacin de la
forma = Ck n , n < 1 . Entonces,
V = Ck n 1
dV
= nV
Vg = V + k
dk
En este caso, la velocidad de grupo es menor que la velocidad de fase. Las ondas
individuales adelantan al grupo.
Para valores ms grandes de k, la frecuencia angular tiende a un valor
constante. La velocidad de fase y de grupo se hacen cada vez ms pequeas,
indicando la existencia de ondas estacionarias para valores grandes de k.

8.4

Se superponen en un medio las dos ondas siguientes:


y1 = A sen ( 5x 10t )

y2 = A sen ( 4x 9t )
Determinar la perturbacin combinada, la velocidad de grupo y la distancia
entre los puntos de amplitud nula.

Por la relacin trigonomtrica de la suma de dos funciones seno, la


perturbacin combinada es
19
9
y = 2 A cos ( x t ) sen x t
2
2
El primer factor corresponde a la amplitud modulada, y el segundo a la fase
armnica. La velocidad de grupo es la velocidad con la que vara la fase del primer
factor. Tenemos
1
Vg = = 1 ( m / s )
1
La distancia entre los puntos de amplitud nula es la distancia entre dos nodos
consecutivos de la amplitud modulada, e igual a su semilongitud de onda,

= = (m )
2 k

5. Caso prctico: Ondas de gravedad


Investigamos las ondas que se producen en la frontera aire-agua bajo la
influencia de la gravedad, como una idealizacin del movimiento superficial que se
produce en un lago o en el mar. Nuestro volumen de agua se representa como un
canal indefinido en la direccin x y confinado en la direccin vertical y entre dos
superficies, el fondo en y = 0 , y la superficie libre en y = h . Nos interesan aqu las
ondas progresivas que se propagan en la direccin x, y que generan una oscilacin
vertical en el agua. Nuestro objetivo es encontrar la relacin de dispersin, la
velocidad de propagacin de dichas ondas y el movimiento del agua al pasar la
onda.
En la direccin vertical, las fuerzas que actan sobre el fluido son la fuerza de
gravedad y la fuerza originada por la variacin de presin con la profundidad. Para
un elemento de volumen dV y masa dm=dV, situado entre las posiciones verticales
y e y+dy y seccin horizontal S, la ecuacin de movimiento es
vy
dVg Py+ dy Py S = dV
t
Al ser dV = Sdy , podemos escribirla en la forma
vy
P
dVg
dV = dV
y
t
con lo cual, como dV es arbitrario, la ecuacin que debemos resolver es
v y
P
g
=
y
t

Adems, el agua es un fluido incompresible. Esto quiere decir que la cantidad


de agua que entra en una regin debe ser la misma que la cantidad que sale de ella.
En trminos de flujo, el flujo neto de agua que atraviesa cualquier superficie cerrada
debe ser cero. Matemticamente,

v ndS = 0
S

donde v es la velocidad del agua y n el vector unitario normal a la superficie S. Por


el teorema de la divergencia esto es cierto cuando
v = 0
que constituye nuestra segunda ecuacin fundamental para la resolucin del
problema. En nuestras coordenadas x,y se escribe
v x vy
+
=0
x
y
El mtodo de resolucin es bastante estndar. Se busca una nica funcin
desconocida , que satisfaga simultneamente

vx =
x

vy =
y
con lo cual la segunda ecuacin a resolver se transforma en
2 2
+
=0
x 2 y 2
Buscamos soluciones que sean ondas progresivas que se propaguen en la direccin
x, en la forma
= A ( y ) cos ( kx t )
Introduciendo esta suposicin en la ecuacin anterior, vemos que la funcin A
satisface la ecuacin diferencial
d2A
k 2A = 0
2
dy
con la solucin general
A ( y ) = Ce ky + De ky
Por tanto, nuestra solucin al problema est dada por la funcin
= Ce ky + De ky cos ( kx t )

La funcin queda determinada una vez que apliquemos las condiciones


frontera que definen nuestro modelo de propagacin. La primera nos dice que la
velocidad vertical debe anularse en el fondo. Es decir,

vy =
= 0 en y = 0
y
Evaluando esta derivada encontramos que C = D . Entonces, la solucin por ahora
puede escribirse en la forma

= 2C cosh ky cos ( kx t )

La condicin frontera en la superficie libre del agua corresponde a un valor de


la presin igual a la presin atmosfrica, valor constante en el tiempo. Por tanto,
debemos obtener la relacin directa entre la funcin y la presin P. De la
ecuacin de movimiento se obtiene

P = g

y t

ecuacin que integrada en la variable y nos da el valor de la presin

P=
+ gy + cte
t
De aqu, la condicin de presin constante en el tiempo en la superficie libre se
escribe
P
= 0 en y = h
t
2
y
0 = 2 + g
en y = h
t
t
De la definicin de la velocidad del fluido, v y = y = , la segunda
t
y
condicin frontera para la funcin queda finalmente
0=

g
2
y
t

en y = h

Esta condicin se satisface para la funcin tipo onda progresiva slo cuando k
y sean dependientes, es decir, cuando estn relacionadas por la llamada relacin
de dispersin. Encontramos
= gk tanh kh
y de aqu, la velocidad de la onda es

g
V= =
tanh kh
k
k
Distinguimos dos tipos de ondas en la superficie del agua
Ondas en aguas profundas
Este caso corresponde a un gran valor de la profundidad h, de forma que
kh >> 1
tanh kh 1
La velocidad de la onda es en esta aproximacin
g
g
V=
=
k
2
Las olas que se observan en mar abierto corresponden a este tipo. La velocidad de
grupo en este caso es
dV 1
Vg = V + k
= V
dk 2

menor que la velocidad de fase, de forma que las crestas de ola que aparecen son
capaces de adelantar al grupo, para desaparecer delante de l.
Ondas en aguas poco profundas
En este caso,
kh << 1
tanh kh kh
con lo cual, la velocidad de la onda es
V = gh
El agua se comporta como un medio no dispersivo en este caso, y todas las ondas
viajan a la misma velocidad. Este tipo de comportamiento se observa en las olas que
se acercan a la orilla. Pero no explicara la existencia de la zona de ruptura de olas
en la orilla. Para entender este hecho, vemos que la velocidad de fase depende de la
profundidad del fondo. Esto quiere decir que las ondas sumergidas en el agua viajan
ms despacio que las ondas que se encuentren en la superficie del agua. Esto
provoca la tpica forma rompiente de una ola al llegar a la orilla. Adems, las olas al
llegar a la orilla ven reducida globalmente su velocidad (por haber disminuido la
profundidad del fondo del mar), pero no as su momento que se conserva constante.
Igual momento pero menor velocidad equivale a una mayor cantidad de agua en las
olas que llegan a la orilla, y a una mayor fuerza descargada.

Tema 9. Formulacin lagrangiana


1. Lagrangiano
Se define como la diferencia entre la energa cintica del sistema T y su
energa potencial V
L = T V
y ser funcin en general de las coordenadas x, de las velocidades x& , y del tiempo t.
Aqu, x representa el conjunto de coordenadas necesario para definir de forma
unvoca la posicin del sistema, y no siempre corresponden a coordenadas de
longitud.

2. Integral de accin S
Se define como la integral respecto al tiempo del lagrangiano
S=

t2

L( x, x&, t) dt

t1

Es una funcional (funcin de funciones) de la trayectoria del sistema x ( t ) . Para


cada trayectoria particular, la integral de accin entre los tiempos considerados, tiene
un valor determinado.

3. Principio de mnima accin


Establece que la trayectoria real del sistema, entre todas las trayectorias
compatibles con las condiciones frontera x ( t1 ) = x1 y x ( t2 ) = x2 , es aquella que
corresponde al valor mnimo de la funcional accin. Es decir, la variacin de la
accin es nula para la trayectoria real
S =0

4. Ecuaciones de Lagrange
Estudiamos la variacin de la accin S, correspondiente a una variacin
arbitraria de la trayectoria x ( t ) . Ya que el tiempo no se trata como una variable
independiente para cada trayectoria, la variacin de la accin es directamente
S =

t2

L( x, x&, t) dt

t1

Al variar la trayectoria, el lagrangiano vara segn


L
L
L = x + x&
x
x&
y adems, como t no acta como variable independiente, la derivada temporal y la
variacin son operadores intercambiables, con lo cual

dx d
= x
dt dt
Por tanto, la variacin de la accin queda expresada en la forma
x& =

S =

t2

t1

L d
L

x x + x& dt x dt

Integramos el segundo trmino por partes

t2

2
L d
L

t2
d L

x
dt

x
x dt
=

t1

&
&
x&

t1 x dt
t1 dt x
con lo cual, la variacin de la accin toma la forma definitiva

S =

t2

t1

L
L d L
t2
x dt x& x dt + x& x t1

El principio de accin mnima establece que dicha variacin es cero para la


trayectoria real, elegida entre todas las trayectorias compatibles con x ( t1 ) = x1 y
x ( t2 ) = x2 . Es decir, se exige que S = 0 , para x ( t1 ) = 0 y x ( t2 ) = 0 , con la

variacin de la trayectoria x ( t ) , funcin arbitraria del tiempo por lo dems.


Concluimos que el segundo sumando es nulo, y as la trayectoria real descrita por el
sistema satisface las ecuaciones de Lagrange
L d L

=0
x dt x&

5. Ecuaciones del movimiento


En la formulacin de Lagrange, la ecuacin que caracteriza la evolucin del
sistema es la ecuacin de Lagrange
d L L

=0
dt x& x
con
L = T V
y existe una ecuacin de Lagrange para cada coordenada del sistema.

6. Evaluacin de la energa cintica


En la mayora de los casos,
1
T = mv 2
2
por lo que la dificultad estriba en determinar v 2 en funcin de las derivadas
temporales de las coordenadas del sistema. En coordenadas cartesianas es fcil
v 2 = x& 2 + y&2 + z&2

En coordenadas cilndricas, la velocidad tiene tres componentes, z& en la


direccin vertical, r& en la direccin radial y r& en la direccin tangente al crculo
de radio r,

z&

r&

r&

y
x
con lo cual

v 2 = r&2 + r 2& 2 + z& 2

En coordenadas esfricas, la velocidad lineal de rotacin en torno al eje z es


r sen & , la velocidad lineal de rotacin respecto al eje perpendicular al eje z es r& ,
y la velocidad radial es r&

r&

r sen

r&

r sen &

r
y

x
con lo cual

v 2 = r& 2 + r 2& 2 + r 2 sen 2

Por ltimo, destacar que en el caso de un cuerpo continuo en movimiento, la


energa cintica ser la suma de la energa cintica de rotacin y de la energa
cintica de traslacin.

7. Evaluacin de V
Si F es la fuerza ejercida sobre el sistema en la direccin del eje x, la energa
potencial viene dada por
V =

Fdx

x0

integrando desde un punto arbitrario x0 . Si la coordenada del sistema es un ngulo


la energa potencial satisface
V =

Fr d

siendo r el radio de giro.

8. Fuerzas de ligadura
Las ligaduras son las condiciones adicionales que deben satisfacer las
coordenadas del sistema, y que limitan el nmero de coordenadas independientes del
sistema. Una partcula de masa m que se mueve por el interior de una aro de radio R
tiene dos coordenadas ( r , ) , una ligadura r = R , y una sola coordenada
independiente .
Las fuerzas de ligadura son las fuerzas que obligan al sistema a seguir una
ligadura. En un pndulo, la tensin T del hilo es la fuerza de ligadura que hace que
la longitud del hilo sea constante.

9. Metodologa
Si no nos interesa conocer las fuerzas de ligadura, desde un principio escribimos
el lagrangiano en funcin slo de las coordenadas independientes y resolvemos las
ecuaciones del movimiento.
Si nos interesa conocer las fuerzas de ligadura, escribimos el lagrangiano como
funcin de todas las coordenadas del sistema, y definimos el lagrangiano modificado
L ' = L + ( ligadura )
donde es la fuerza de ligadura que multiplica a la ecuacin de la ligadura
expresada en una forma que sea idnticamente nula. Con esto el lagrangiano
modificado es estrictamente igual al lagrangiano inicial. Entonces, la condicin de
accin mnima exigiendo el cumplimiento de la ligadura da lugar a las ecuaciones de
Lagrange para el lagrangiano modificado L, ms la condicin L ' = 0 , que nos

da de nuevo la ecuacin de la ligadura. As resolvemos las ecuaciones de


movimiento y la fuerza de ligadura. Es anlogo al clculo de extremos
condicionados en Anlisis.

Como regla general, debe multiplicar a la ecuacin de ligadura resuelta para


las coordenadas, no para el cuadrado de las coordenadas, etc. Es decir, si la ligadura
es x 2 + y 2 = a 2 , el lagrangiano modificado se escribe
L' = L +

x2 + y 2 a

y as tiene las dimensiones correctas de fuerza.

Problemas Resueltos
9.1

Determinar por el mtodo lagrangiano la posicin de equilibrio de una


varilla rgida de masa M y longitud L, con un extremo fijo a un eje vertical que
gira con velocidad angular constante .

M, L

Sea el ngulo que forma la varilla con el eje vertical. Definimos x como la
coordenada de posicin de un elemento de masa dm de la varilla, respecto del punto
fijo. La energa cintica de la varilla es
1
T=
V 2 ( x ) dm
2
siendo V ( x ) la velocidad del elemento de masa dm. Dicha velocidad tiene dos
componentes,

x sen

x&

x sen

una perpendicular a la varilla con valor x& , en la direccin creciente del ngulo ,
y otra en direccin horizontal, tangente a la trayectoria circular de dm, con valor
r = x sen , siendo r = x sen el radio de giro del elemento dm. La primera
componente se debe al giro de la varilla sobre el punto fijo, y la segunda al giro de la
varilla respecto al eje vertical. Por tanto,

( ) + ( x sen )

V 2 ( x ) = x&

= x 2 & 2 + 2 sen 2

Por ser la varilla un cuerpo homogneo, el elemento de masa es dm = M

dx ,

con lo cual, la energa cintica toma el valor


1
T=
2

(&

1 M &2
=
+ 2 sin 2
2 L
=

M
dx
L

+ 2 sin 2 x 2

1
ML2 & 2 + 2 sin 2
6

x 2dx

Medimos la energa potencial gravitatoria respecto de la posicin horizontal de la


varilla ( = 90 )
V=

gy dm = g

ydm = Mgycm

siendo ycm la posicin vertical del centro de masas respecto del plano horizontal de
referencia,
L
ycm = cos
2
con lo cual
L
V = Mg cos
2
El lagrangiano de la varilla es
L
1
L = T V = ML2 & 2 + 2 sen 2 + Mg cos
6
2
y la ecuacin de Lagrange resulta
d L L
=0

dt &

d 1
1
1
ML2& = ML2 2 sen cos MgL sen

dt 3
2
3
y para la posicin de equilibrio, && = 0 , obtenemos la condicin

1 2 2
1
ML sen cos = MgL sen
3
2
con dos soluciones, una estable
3 g
cos =
2 L 2
y otra inestable,
sen = 0

9.3 Determinar la forma que adquiere la superficie libre de un lquido


dentro de una vasija cilndrica en rotacin.

Por simetra, la forma ser z = f ( r ) , donde z es la altura de un punto material de


la superficie libre y r su distancia radial al eje de giro. Sin tener en cuenta la
ligadura, la velocidad de un elemento de masa m tiene tres componentes
perpendiculares, una componente radial r& , una componente vertical z& , y una
componente en la direccin tangente a la circunferencia de radio r, de valor r .
Entonces,
v 2 = r& 2 + z& 2 + 2r 2
la energa cintica del elemento de masa es
1
T = m r& 2 + z& 2 + 2 r 2
2
y la energa potencial respecto de la posicin de equilibrio ( z = 0 ) es
V = mgz

Para tener en cuenta la ligadura, definimos el lagrangiano modificado


L ' = L + ( z f ( r ))

1
m r& 2 + z& 2 + 2r 2 mgz + ( z f ( r ) )
2
Las ecuaciones de Lagrange son
d L ' L '
= 0 mr&& = m 2 r f ' ( r )
&
dt r r
=

d L ' L '
=0

dt z& z
L '
=0

mz&& = mg
z = f (r )

Para calcular la posicin de equilibrio del elemento de masa, basta tomar las
condiciones &&
z = 0, r&& = 0 , en las ecuaciones anteriores. Obtenemos la solucin
m 2 r = f ' ( r )
= mg
z = f (r )

De las dos primeras, eliminando la fuerza de ligadura , encontramos la pendiente


de la curva de equilibrio
2r
f '(r ) =
g
Integrando esta ltima ecuacin con la condicin z = 0 si r = 0 , es decir, f ( 0 ) = 0 ,
obtenemos
1 2r 2
f (r ) =
2 g
y la forma de la superficie libre resulta ser la de un paraboloide
1 2r 2
z=
2 g

9.3

Determinar la posicin de equilibrio de una masa m unida a una cuerda


de longitud L, a su vez unida a un volante de radio r que gira con velocidad
angular constante .

L
m

La velocidad de la masa m tiene dos componentes perpendiculares entre s, L& ,


en direccin perpendicular a la cuerda y ( r + L sen ) en direccin tangente a la
circunferencia de giro en el plano horizontal de radio r + L sen . Con esto, la
energa cintica de la masa m es
1
2
T = m L2&2 + 2 ( r + L sen )
2
y la energa potencial respecto al plano horizontal ( = 90 ) es
V = mgL cos

El lagrangiano del sistema es


1
2
L = m L2& 2 + 2 ( r + L sen ) + mgL cos
2
y la ecuacin del movimiento
d L L
=0

dt &
resulta ser
d
mL2& = mL ( r + L sen ) 2 cos mgL sen
dt

Para el equilibrio, && = 0 , obtenemos


2
tan =
( r + L sen )
g

9.4

Determinar la ecuacin del movimiento de un sistema formado por una


cuerda de longitud L, con una masa m en cada extremo, de forma que una de
ellas se mueve sobre el eje x, y la otra sobre el eje y. Inicialmente la cuerda se
encuentra en la posicin horizontal ( = 0 )
x

y
L
m

Si no tenemos en cuenta la ligadura del sistema, la nica coordenada del sistema


es el ngulo que forma la cuerda con el eje x. La energa cintica del sistema es
1
1
T = mx& 2 + my& 2
2
2
donde
x = L cos
y = L sen
y de aqu, derivando respecto del tiempo, encontramos
x& = L& sen
y& = L& cos
con lo cual la energa cintica tiene la expresin
1
T = mL2& 2
2
La energa potencial respecto a la posicin de equilibrio horizontal ( = 0 ) es igual
a la energa potencial de la masa que desliza por el eje y
V = mgy = mgL sen
El lagrangiano es
1
L = mL2& 2 + mgL sen
2
y la ecuacin de movimiento resultante es
d L L

= 0 mL2&& = mgL cos

&
dt

g
&& = cos
L

9.5

Determinar la fuerza de ligadura en el caso anterior.

Las variables del sistema son ( x , y ) y la ligadura L2 = x2 + y 2 . El lagrangiano


modificado es
1
L ' = m x& 2 + y& 2 + mgy + x 2 + y 2 L
2
siendo la fuerza de ligadura. Las ecuaciones de Lagrange son
d L ' L '
x

= 0 mx&& =

2
dt x& x
x + y2

d L ' L '
=0

dt y& y

my&& = mg +

L '
=0

y
x2 + y2

x 2 + y2 = L

Utilizando la tercera ecuacin, las dos primeras quedan


mx&& =

x
L

my&& = mg +

y
L

Para integrar esta ecuacin, nos ayudamos de una ecuacin adicional, de


conservacin de la energa
1
E = m x& 2 + y& 2 mgy
2
y como inicialmente la cuerda est en reposo horizontalmente, E = 0 . Tenemos as
las tres ecuaciones
x
mx&& =
L
y
my&& = mg +
L
2
2
x& + y& = 2 gy
de donde podremos obtener la fuerza de ligadura.

Multiplicando la primera por x, la segunda por y, sumando el resultado,


obtenemos
m ( xx&& + yy&&) = L + mgy
Derivando dos veces la condicin de ligadura L2 = x2 + y 2 , encontramos

xx&& + yy&& = x& 2 + y& 2

con lo cual, con ayuda de la conservacin de energa, la ecuacin anterior queda en


la forma
m x& 2 + y& 2 = L + mgy

= 3 mg

y
= 3mg sen
L

9.6

Determinar la tensin de la cuerda y la aceleracin de cada de las masas


en una mquina de Atwood simple, mediante el mtodo lagrangiano.

x2

x1
m2
m1

La tensin de la cuerda es la fuerza de ligadura que mantiene unidas las masas a


la cuerda de longitud L. Tomamos nuestro origen de coordenadas en el punto ms
alto de la trayectoria. Sean ( x1, x2 ) las coordenadas de ambas masas respecto del
punto de referencia. La energa cintica del sistema es
1
1
T = m1x12 + m2 x22
2
2
la energa potencial se mide respecto al punto de referencia resultando ser
V = m1gx1 m2 gx 2
Por ltimo, la ecuacin de la ligadura es x1 + x2 = L
El lagrangiano modificado es
1
1
L ' = m1x12 + m2 x22 + m1 gx1 + m2 gx2 + ( x1 + x2 L )
2
2
y las ecuaciones de Lagrange son
d L ' L '
= 0 m1&&
x1 = m1 g +

dt x&1 x1
d L ' L '
=0

dt x&2 x2
L '
=0

m2 &&
x2 = m2 g +
x1 + x2 = L

x1 = &&
x2 . Restando las dos primeras
De la condicin de ligadura obtenemos &&
ecuaciones obtenemos la aceleracin de cada de las masas
( m1 + m2 ) &&x1 = ( m1 m2 ) g

m m2
&&
x1 = 1
g
m1 + m2

Entonces, de la primera ecuacin se obtiene la fuerza de ligadura, la tensin


de la cuerda,
2m1m2
=
g
m1 + m2
El signo menos indica que la tensin est dirigida en el sentido contrario al aumento
de las coordenadas x1, x2 .

9.7

Una masa puntual m se encuentra en reposo en el punto ms alto de un


esfera fijada al suelo de radio R. Determinar por el mtodo lagrangiano la
ecuacin de su movimiento de cada cuando se separa ligeramente del punto de
equilibrio, y establecer el punto en el que la masa se separa de la superficie
esfrica.

m
r

La fuerza que mantiene a la masa sobre la superficie esfrica es igual a la


reaccin normal N de la esfera sobre ella. Tomamos coordenadas polares centradas
en el origen de la esfera en el plano de la trayectoria de cada. Las coordenadas de la
masa m sern ( r , ) . La energa cintica de la masa m est dada por

1
m r&2 + r 2 2
2
La energa potencial respecto de la posicin de equilibrio est dada por
V = mg ( R r cos )
y la condicin de ligadura es r = R .
T=

El lagrangiano modificado es
1
L ' = m r&2 + r 2& 2 + mg ( R r cos ) + N ( r R )
2
y las ecuaciones de Lagrange son
d L ' L '
= 0 mr&& = mg cos + N + mr&2
&
dt r r

d L ' L '
=0

dt &
L '
=0
N

mr 2&& = mgr sen


r=R

Con ayuda de la condicin de ligadura, de la segunda ecuacin obtenemos la


ecuacin del movimiento para la trayectoria de cada
g
&& = sen
R
y de aqu, para la primera ecuacin
N = mg cos mR&2
necesitamos encontrar & en funcin de . Partimos de la relacin
d& & d&
&& =
=
dt
d
con lo cual, la ecuacin de movimiento puede escribirse
d& g
&
= sen
d R
Realizamos la integral con la condicin inicial, & = 0 si = 0 , resultando
g
& 2 = 2 ( cos 1)
R
con lo cual
mR& 2 = 2mg ( cos 1)
y as, la normal adquiere el valor
N = 3mg cos 2mg
La partcula abandona la superficie de la esfera cuando la normal se haga cero.
Esto es, en la posicin angular
2
cos =
3

Apndice
A) Principio de mnima accin
Establece que la trayectoria real del sistema, entre todas las trayectorias
compatibles con las condiciones frontera x ( t1 ) = x A y x ( t2 ) = xB , es aquella que
corresponde al valor mnimo de la funcional accin. Es decir, la variacin de la
accin es nula para la trayectoria real
S =0
Para enunciar de forma matemtica precisa lo que implica este principio de
accin mnima, vamos a considerar el cambio producido en la accin al pasar de una
trayectoria a otra vecina, teniendo las dos sus extremos en los puntos A y B, segn
muestra la figura

x (t ) + x (t )

B
x (t )

Ya que las trayectorias comparten extremos comunes, la variacin de la accin debe


satisfacer la condicin adicional
x ( t1 ) = x ( t2 ) = 0
Suponemos que la variable de tiempo t no vara de una trayectoria a otra, y no
se considera una coordenada independiente. Por tanto, la accin S slo es funcional
de la trayectoria considerada x ( t ) . En consecuencia, su variacin es
S =

t2

L( x, x&, t) dt

t1

La variacin del lagrangiano es


L
L
x + x&
x
x&
y en virtud que la coordenada temporal es independiente de la trayectoria elegida,
podemos intercambiar el operador de derivada temporal y el operador de variacin
con lo que
dx d
x& =
= x
dt dt
con lo cual, la variacin de la accin queda expresada en la forma
L( x, x&, t) =

S =

t2

t1

L d
L

x x + x& dt x dt

Integrando por partes el ltimo factor, vemos que

t2

t1

2
L d
L

x
dt

x
=

x& dt
x&
t1

t2

t1

d L
dt x& xdt

y ya que todas la trayectorias tienen extremos comunes x ( t1 ) = x ( t2 ) = 0 , el


primer trmino es nulo. As, la variacin de la accin para un cambio arbitrario en la
trayectoria, tiene la forma final
S =

t2

t1

L d L
x dt x& xdt

Segn establece el principio de mnima accin, esta variacin debe ser nula
para cualquier cambio x ( t ) , respecto de la trayectoria real correspondiente al
movimiento real del cuerpo. Esto ser as si la trayectoria real es aquella que
satisface las ecuaciones de Lagrange
d L L

=0
dt x& x

B) Teorema de E. Noether
Este teorema relaciona las cantidades conservadas (energa, momento, etc) del
movimiento con el tipo de simetra que se conserva en el sistema. Supongamos que
la transformacin de las coordenadas
t t = t + t
x ( t ) x ( t )

no vara la integral de accin para una trayectoria dada. Veamos que consecuencias
podemos extraer de esta simetra.
La variacin de la accin debido a esta transformacin de coordenadas es
x
x
S = L( x ,
, t ) dt L( x , , t ) dt
t
t
Calculamos las variaciones de las coordenadas por separado. Respecto a la variable
de tiempo, tenemos
dt
d t
dt =
dt = 1 +
dt
dt
dt

Para la variable espacial, su variacin es doble, primero por variar la propia


coordenada y segundo por variar su dependencia en el tiempo. Obtenemos
x
x = x ( t ) x ( t ) = x + t
t
siendo x = x (t ) x ( t ) es la variacin de la coordenada manteniendo fijo el tiempo.
De la misma forma, la variacin del lagrangiano se compone de dos partes

L = L+

L
L
L
L
dt = x + &x + dt
t
x
&x
t

La variacin de la accin se escribe


x
d t
x
x

S = L( x ,
, t)
dt + L( x ,
, t ) L( x , , t ) dt
dt
t
t
t

t
L
L
L
= L
dt + x + x& +
dt dt
t
x&
t
x
( L t )
L
L
=
dt + x + x& dt
t
x&
x

De la misma forma que en el apartado anterior, el operador de derivada


temporal puede intercambiarse con el operador de variacin a tiempo constante, por
lo que

x& = x
t
con lo cual, podemos escribir el ltimo trmino en el integrando en la forma
L
L
L
x& = x x
x&
t x&
t x&
y as, la variacin de la accin tiene la expresin final
( L t )
L
L
L
S =
dt + x + x x dt
t
t x&
t x&

S =

L
L t + & x dt +
t
x

L L
x dt

x t x&

Con la condicin de que la variacin de la accin sea nula y que el


movimiento recorra la trayectoria real, esto es, que se satisfagan las ecuaciones de
Lagrange, obtenemos el teorema de E. Noether: Si S es invariante bajo la
transformacin de las coordenadas t t, x x , entonces

L
S =
L t + x dt = 0

t
x&
Es decir,

L
L t + x = 0

t
x&
lo que es lo mismo
L
L t + x
x&
x
es una constante del movimiento. Por ltimo, al tener x = x t , dicha
t
constante del movimiento se escribe en la forma
L
L

L x& x& t + x& x

1. Simetra frente a traslacin temporal


La transformacin temporal es

t = a
x=0

y en consecuencia, segn el teorema de Noether, existe una cantidad conservada en


el movimiento en la forma
L
x& L
x&
Para encontrar el significado a esta constante, basta utilizar el lagrangiano habitual
1 2
& +V ( x)
L = mx
2
y llegamos a
L
1
1
x& L = mx& 2 mx& 2 + V ( x ) = mx& 2 + V ( x ) = E
x&
2
2
que es la energa total del sistema. Por tanto, cuando la evolucin de un sistema no
dependa del tiempo, su energa total se conserva.

2. Simetra frente a traslacin espacial


La traslacin espacial es

t = 0
x=a

Se conserva el momento conjugado px definido por


L
px =
x&
Si la coordenada x mide una longitud, px = mx& es constante. Es decir, se conserva el
momento lineal en la direccin del eje que mantiene invariante al sistema. Si la
coordenada x es un ngulo, p x = I& es constante. Es decir, se conserva el momento
angular de giro respecto al eje que mantiene invariante al sistema.

C) Formulacin hamiltoniana
Para describir el estado fsico de un sistema no es necesario escoger las
coordenadas rectangulares usuales. Es posible escoger cualquier conjunto de parmetros
que especifiquen de forma completa el estado del sistema. Estas cantidades no tienen
por qu tener dimensin de longitud, ya que dependiendo del problema puede ser ms
adecuado elegir algn parmetro sin dimensin, como puede ser un ngulo, etc. Por
coordenadas generalizadas se entiende dicho conjunto de parmetros que describen
completamente el estado del sistema. La finalidad que se busca al elegir uno u otro
conjunto de coordenadas es obtener las ecuaciones de movimiento resultantes en la
forma ms simple posible, que permita una interpretacin del movimiento lo ms
sencillamente posible.

Se definen los momentos generalizados segn la frmula


L
p=
&x
con lo cual, las ecuaciones de Lagrange pueden escribirse en la forma
d
L
p=
dt
x
Una consecuencia directa es que si el lagrangiano no depende de la coordenada
generalizada x, su momento generalizado se conserva constante en el tiempo
La descripcin del movimiento utilizando las coordenadas generalizadas y sus
momentos asociados de forma independiente se basa en la formulacin hamiltoniana.
Con esto el estado del sistema queda descrito no slo por su posicin sino tambin por
el conjunto de velocidades generalizadas que definen su movimiento. Se introduce el
hamiltoniano del sistema en la forma
& )
H ( x,p,t ) = &xp L ( x,x,t
donde debe entenderse que las velocidades generalizadas se introducen como funciones
de x,p y t. Es decir, de la ecuacin p = L &x , se despejan las velocidades generalizadas
respecto de las coordenadas y momentos generalizados.
Tomando la diferencial total del hamiltoniano, se obtiene
L
L
L
& dx dx
& dt
dH = &xdp + pdx
x
&x
t
y en virtud de las ecuaciones de Lagrange y de la definicin de momento generalizado,
se puede escribir
L
dH = &xdp &pdx dt
t
de donde se obtienen las ecuaciones del movimiento en la formulacin hamiltoniana
H
&x =
p

&p =

H
x

dH H
L
=
=
dt
t
t
La ltima ecuacin expresa el hecho de que si el lagrangiano no depende explcitamente
del tiempo, el hamiltoniano se conserva constante en el tiempo. De nuevo se obtiene
uno de los resultados deducidos por el teorema de Noether.

Ejemplo
Consideramos una partcula de masa m que se mueve sobre la superficie
interior del cilindro vertical x2 + y 2 = R 2 , de radio R. La partcula sufre la
accin de una fuerza central dirigida hacia el origen de coordenadas,
F = kr . Determinar las ecuaciones del movimiento utilizando el mtodo
hamiltoniano.
En coordenadas cilndricas, tomando el eje z en la direccin del eje del cilindro,
la velocidad de la masa m satisface
v2 = R2& 2 + &z2
puesto que la coordenada radial siempre es constante, r = R . La energa cintica del
sistema es, por tanto
1
1
T = mv 2 = m R2& 2 + &z 2
2
2

La energa potencial de la partcula en presencia de la fuerza central es


1
1
V = F dr = kr 2 = k R2 + z 2
2
2
con lo cual, el lagrangiano del sistema queda escrito en la forma
1
1
L = T V = m R2& 2 + &z2 k R2 + z2
2
2

Las coordenadas generalizadas que describen completamente el estado del


sistema son y z. Los momentos generalizados son
L
p = = mR2&
&
L
&
p z = = mz
&z
de donde deben despejarse las velocidades generalizadas, obtenindose
p
& = 2
mR
p
&z = z
m

El hamiltoniano est definido en la forma

p2

p2z 1
+ k R2 + z 2
2
2 mR 2 m 2
Las ecuaciones del movimiento correspondientes a este hamiltoniano son cuatro
H
&p =
=0

H
&p z =
= kz
z
p
H
& =
= 2
p mR
H pz
&z =
=
pz m
La solucin completa es
p = mR2& = cte
por lo que el momento angular alrededor del eje z es constante. Este hecho estaba
asegurado por ser el eje z el eje de simetra del sistema. Por otro lado, la ecuacin de
movimiento para la coordenada z
k
&&z + z = 0
m
representa un movimiento armnico simple.
H = & p + &zpz L =

Potrebbero piacerti anche